CSAT FLT – 4 – PRELIMS 2024
Time limit: 0
0 of 80 questions completed
Questions:
- 1
- 2
- 3
- 4
- 5
- 6
- 7
- 8
- 9
- 10
- 11
- 12
- 13
- 14
- 15
- 16
- 17
- 18
- 19
- 20
- 21
- 22
- 23
- 24
- 25
- 26
- 27
- 28
- 29
- 30
- 31
- 32
- 33
- 34
- 35
- 36
- 37
- 38
- 39
- 40
- 41
- 42
- 43
- 44
- 45
- 46
- 47
- 48
- 49
- 50
- 51
- 52
- 53
- 54
- 55
- 56
- 57
- 58
- 59
- 60
- 61
- 62
- 63
- 64
- 65
- 66
- 67
- 68
- 69
- 70
- 71
- 72
- 73
- 74
- 75
- 76
- 77
- 78
- 79
- 80
Information
.
You have already completed the Test before. Hence you can not start it again.
Test is loading…
You must sign in or sign up to start the Test.
You have to finish following quiz, to start this Test:
Your results are here!! for” CSAT FLT – 4 – PRELIMS 2024 “
0 of 80 questions answered correctly
Your time:
Time has elapsed
Your Final Score is : 0
You have attempted : 0
Number of Correct Questions : 0 and scored 0
Number of Incorrect Questions : 0 and Negative marks 0
-
Not categorized
You have attempted: 0
Number of Correct Questions: 0 and scored 0
Number of Incorrect Questions: 0 and Negative marks 0
maximum of 200 points Pos. Name Entered on Points Result Table is loading No data available
Your result has been entered into leaderboardLoading
- 1
- 2
- 3
- 4
- 5
- 6
- 7
- 8
- 9
- 10
- 11
- 12
- 13
- 14
- 15
- 16
- 17
- 18
- 19
- 20
- 21
- 22
- 23
- 24
- 25
- 26
- 27
- 28
- 29
- 30
- 31
- 32
- 33
- 34
- 35
- 36
- 37
- 38
- 39
- 40
- 41
- 42
- 43
- 44
- 45
- 46
- 47
- 48
- 49
- 50
- 51
- 52
- 53
- 54
- 55
- 56
- 57
- 58
- 59
- 60
- 61
- 62
- 63
- 64
- 65
- 66
- 67
- 68
- 69
- 70
- 71
- 72
- 73
- 74
- 75
- 76
- 77
- 78
- 79
- 80
-
Answered
-
Review
-
Question 1 of 80
1. Question
DIRECTION : READ THE FOLLOWING PASSAGE AND ANSWER THE QUESTION THAT FOLLOW. YOUR ANSWER TO THIS QUESTION SHOULD BE BASED ON THE PASSAGE ONLY.
It's easy to forget that most of the world's languages are still transmitted orally with no widely established written form. While speech communities are increasingly involved in projects to protect their languages – in print, on air and online – orality is fragile and contributes to linguistic vulnerability. But indigenous languages are about much more than unusual words and intriguing grammar: They function as vehicles for the transmission of cultural traditions, environmental understandings and knowledge about medicinal plants, all at risk when elders die and livelihoods are disrupted.
Both push and pull factors lead to the decline of languages. Through war, famine and natural disasters, whole communities can be destroyed, taking their language with them to the grave, such as the indigenous populations of Tasmania who were wiped out by colonists. More commonly, speakers live on but abandon their language in favor of another vernacular, a widespread process that linguists refer to as “language shift” from which few languages are immune. Such trading up and out of a speech form occurs for complex political, cultural and economic reasons – sometimes voluntary for economic and educational reasons, although often amplified by state coercion or neglect. Welsh, long stigmatized and disparaged by the British state, has rebounded with vigor.
Many speakers of endangered, poorly documented languages have embraced new digital media with excitement. Speakers of previously exclusively oral tongues are turning to the web as a virtual space for languages to live on. Internet technology offers powerful ways for oral traditions and cultural practices to survive, even thrive, among increasingly mobile communities. I have watched as videos of traditional wedding ceremonies and songs are recorded on smartphones in London by Nepali migrants, then uploaded to YouTube and watched an hour later by relatives in remote Himalayan villages . . .Globalization is regularly, and often uncritically, pilloried as a major threat to linguistic diversity. But in fact, globalization is as much process as it is ideology, certainly when it comes to language. The real forces behind cultural homogenization are unbending beliefs, exchanged through a globalized delivery system, reinforced by the historical monolingualism prevalent in much of the West.
Monolingualism – the condition of being able to speak only one language – is regularly accompanied by a deep-seated conviction in the value of that language over all others. Across the largest economies that make up the G8, being monolingual is still often the norm, with multilingualism appearing unusual and even somewhat exotic. The monolingual mindset stands in sharp contrast to the lived reality of most the world, which throughout its history has been more multilingual than unilingual. Monolingualism, then, not globalization, should be our primary concern.
Multilingualism can help us live in a more connected and more interdependent world. By widening access to technology, globalization can support indigenous and scholarly communities engaged in documenting and protecting our shared linguistic heritage. For the last 5,000 years, the rise and fall of languages was intimately tied to the plow, sword and book. In our digital age, the keyboard, screen and web will play a decisive role in shaping the future linguistic diversity of our species.
We can infer all of the following about indigenous languages from the passage EXCEPT that:
(A) they are repositories of traditional knowledge about the environment and culture.
(B) people are increasingly working on documenting these languages.
(C) they are in danger of being wiped out as most can only be transmitted orally.
(D) their vocabulary and grammatical constructs have been challenging to document.
-
-
-
-
Correct
Incorrect
This question is pertaining to indigenous languages. From third paragraph, we can infer options A,B and C. Option D cannot be inferred because if many speakers of poorly documented languages have embraced new digital media with excitement, there would not be many challenges in it. Moreover, there is no evidence of any challenges faced.
Unattempted
This question is pertaining to indigenous languages. From third paragraph, we can infer options A,B and C. Option D cannot be inferred because if many speakers of poorly documented languages have embraced new digital media with excitement, there would not be many challenges in it. Moreover, there is no evidence of any challenges faced.
-
Question 2 of 80
2. Question
DIRECTION : READ THE FOLLOWING PASSAGE AND ANSWER THE QUESTION THAT FOLLOW. YOUR ANSWER TO THIS QUESTION SHOULD BE BASED ON THE PASSAGE ONLY.
Today we can hardly conceive of ourselves without an unconscious. Yet between 1700 and1900, this notion developed as a genuinely original thought. The “unconscious” burst the shell of conventional language, coined as it had been to embody the fleeting ideas and the shifting conceptions of several generations until, finally, it became fixed and defined in specialized terms within the realm of medical psychology and Freudian psychoanalysis.
The vocabulary concerning the soul and the mind increased enormously in the course of the nineteenth century. The enrichments of literary and intellectual language led to an altered understanding of the meanings that underlie time-honored expressions and traditional catchwords. At the same time, once coined, powerful new ideas attracted to themselves a whole host of seemingly unrelated issues, practices, and experiences, creating a peculiar network of preoccupations that as a group had not existed before. The drawn-out attempt to approach and define the unconscious brought together the spiritualist and the psychical researcher of borderline phenomena (such as apparitions, spectral illusions, haunted houses, mediums, trance, automatic writing); the psychiatrist or alienist probing the nature of mental disease, of abnormal ideation, hallucination, delirium, melancholia, mania; the surgeon performing operations with the aid of hypnotism; the magnetizer claiming to correct the disequilibrium in the universal flow of magnetic fluids but who soon came to be regarded as a clever manipulator of the imagination; the physiologist and the physician who puzzled oversleep, dreams, sleepwalking, anesthesia, the influence of the mind on the body in health and disease; the neurologist concerned with the functions of the brain and the physiological basis of mental life; the philosopher interested in the will, the emotions, consciousness, knowledge, imagination and the creative genius; and, last but not least, the psychologist.
Significantly, most if not all of these practices (for example, hypnotism in surgery or psychological magnetism) originated in the waning years of the eighteenth century and during the early decades of the nineteenth century, as did some of the disciplines (such as psychology and psychical research). The majority of topics too were either new or assumed hitherto unknown colors. Thus, before 1790, few if any spoke, in medical terms, of the affinity between creative genius and the hallucinations of the insane .
Striving vaguely and independently to give expression to a latent conception, various lines of thought can be brought together by some novel term. The new concept then serves as a kind of resting place or stocktaking in the development of ideas, giving satisfaction and a stimulus for further discussion or speculation. Thus, the massive introduction of the term unconscious by Hartmann in 1869 appeared to focalize many stray thoughts, affording a temporary feeling that a crucial step had been taken forward, a comprehensive knowledge gained, a knowledge that required only further elaboration, explication, and unfolding in order to bring in a bounty of higher understanding. Ultimately, Hartmann's attempt at defining the unconscious proved fruitless because he extended its reach into every realm of organic and inorganic, spiritual, intellectual, and instinctive existence, severely diluting the precision and compromising the impact of the concept.
All of the following statements may be considered valid inferences from the passage, EXCEPT:
(A) New conceptions in the nineteenth century could provide new knowledge because of the establishment of fields such as anaesthesiology.
(B) Unrelated practices began to be treated as related to each other, as knowledge of the mind grew in the nineteenth century.
(C) Without the linguistic developments of the nineteenth century, the growth of understanding of the soul and the mind may not have happened.
(D) Eighteenth century thinkers were the first to perceive a connection between creative genius and insanity.
-
-
-
-
Correct
Incorrect
This question apparently looks tough because it has the phrase “valid inferences… except”. Option B can be inferred from the entire second paragraph. From the first paragraph we can infer C. From the last sentence of second last paragraph we can infer D. Thus A is the best choice, as we don't have any evidence for A.
Unattempted
This question apparently looks tough because it has the phrase “valid inferences… except”. Option B can be inferred from the entire second paragraph. From the first paragraph we can infer C. From the last sentence of second last paragraph we can infer D. Thus A is the best choice, as we don't have any evidence for A.
-
Question 3 of 80
3. Question
DIRECTION : READ THE FOLLOWING PASSAGE AND ANSWER THE QUESTION THAT FOLLOW. YOUR ANSWER TO THIS QUESTION SHOULD BE BASED ON THE PASSAGE ONLY.
As high-level visits go, Prime Minister Narendra Modi’s visit to Israel was bound to attract superlatives like ‘historic’ and ‘groundbreaking’. Still, it is clear that the buzz in the relationship is on account of Mr. Modi’s personal diplomatic style and his host’s equally warm response.
Israel’s Prime Minister Benjamin Netanyahu set the tone when he welcomed Mr. Modi at the airport together with the spiritual leaders of all the major faiths in the region, an honor traditionally reserved for the U.S. President and the Pope. Mr. Modi’s trademark bear hugs were reciprocated, three at the airport, and by the time the visit ended, the TV commentators had lost count!
Mr. Netanyahu gushingly described the relationship between the two countries as “a marriage made in heaven”, but behind the success was a receptive political backdrop as well as the careful planning undertaken by both sides.
Marking 25 years of establishing diplomatic relations between the two countries added to the historic character of an Indian Prime Minister’s first visit to a country that had quietly emerged as a strong defense partner. There had been high-level exchanges but the Indian response was cautious. Foreign Minister Shimon Peres first visited India in 1993, and Jaswant Singh reciprocated in 2000; President Ezer Weizman came in 1997, while President Pranab Mukherji’s visit only took place in 2015. The first Israeli Prime Minister to visit India was Ariel Sharon in 2003, and from the time Mr. Modi came to power, a return visit was a certainty. The two leaders had met on the margins of the UN General Assembly and continued their relationship by tweeting their greetings on Hanukkah and Diwali.
Yet, official-level exchanges between the two countries have been intensive, beginning with the visit of Foreign Secretary J.N. Dixit in early 1993. While relations between Mossad and Research and Analysis Wing had existed earlier, the strategic partnership got cemented when National Security Adviser-level dialogue was established in 1999 between Brajesh Mishra and Gen. (retd.) David Ivry. Gen. Ivry was a former Air Force chief who had led the air raid on Osirak, the Iraqi nuclear reactor, in 1981. Incidentally, Israel was one of the few countries that showed a complete understanding of India’s decision to undertake the nuclear tests in 1998. This reinforced both the defense and the counter-terrorism cooperation relationship.
By 2000, India was acquiring surface-to-air missiles (Barak 1) and UAVs (unmanned aerial vehicles) from Israel. Subsequently, the refurbishing of MiG-21 aircraft employed Israeli avionics. During the 1999 Kargil war, Israel assisted with laser-guidance kits mated with gravity bombs, carried by the Mirage 2000 aircraft. With U.S. concurrence, Israel sold India the Phalcon airborne early warning system and mounted on the Russian Il-76, provided AWACS capability. Subsequent acquisitions have included Spike anti-tank guided missiles and the long range surface-to-air missiles in both the naval and land versions. Today, Israel has emerged as the third-largest defense supplier for India and accounts for over 40% of Israel’s defense exports.
Commercial relations between the diamond traders in Gujarat and Israel had existed before 1992, but now annual trade grew from $200 million to nearly $5 billion with gems and jewelry accounting for nearly 40%. Gradually, Science and technology, agriculture, biotech, and space emerged as new areas of cooperation.Tourism provided an impetus to people-to-people relations. India emerged as the preferred destination for young Israelis wanting to unwind after their compulsory military service and Hebrew signage in Varanasi, Manali, and Goa is a common sight. Ambassador Pavan Kapoor was being quite mattered of fact when he described the Modi visit as a ‘coming out visit’ for the relationship.
Mr. Netanyahu Gushingly described the relationship between the two countries as:
(A) A friendship made on earth
(B) A friendship made in heaven
(C) A marriage made in heaven
(D) An enmity made in heaven
-
-
-
-
Correct
Incorrect
Mr. Netanyahu gushingly described the relationship between the two countries as “a marriage made in heaven”,
Unattempted
Mr. Netanyahu gushingly described the relationship between the two countries as “a marriage made in heaven”,
-
Question 4 of 80
4. Question
DIRECTION : READ THE FOLLOWING PASSAGE AND ANSWER THE QUESTION THAT FOLLOW. YOUR ANSWER TO THIS QUESTION SHOULD BE BASED ON THE PASSAGE ONLY.
As high-level visits go, Prime Minister Narendra Modi’s visit to Israel was bound to attract superlatives like ‘historic’ and ‘groundbreaking’. Still, it is clear that the buzz in the relationship is on account of Mr. Modi’s personal diplomatic style and his host’s equally warm response.
Israel’s Prime Minister Benjamin Netanyahu set the tone when he welcomed Mr. Modi at the airport together with the spiritual leaders of all the major faiths in the region, an honor traditionally reserved for the U.S. President and the Pope. Mr. Modi’s trademark bear hugs were reciprocated, three at the airport, and by the time the visit ended, the TV commentators had lost count!
Mr. Netanyahu gushingly described the relationship between the two countries as “a marriage made in heaven”, but behind the success was a receptive political backdrop as well as the careful planning undertaken by both sides.
Marking 25 years of establishing diplomatic relations between the two countries added to the historic character of an Indian Prime Minister’s first visit to a country that had quietly emerged as a strong defense partner. There had been high-level exchanges but the Indian response was cautious. Foreign Minister Shimon Peres first visited India in 1993, and Jaswant Singh reciprocated in 2000; President Ezer Weizman came in 1997, while President Pranab Mukherji’s visit only took place in 2015. The first Israeli Prime Minister to visit India was Ariel Sharon in 2003, and from the time Mr. Modi came to power, a return visit was a certainty. The two leaders had met on the margins of the UN General Assembly and continued their relationship by tweeting their greetings on Hanukkah and Diwali.
Yet, official-level exchanges between the two countries have been intensive, beginning with the visit of Foreign Secretary J.N. Dixit in early 1993. While relations between Mossad and Research and Analysis Wing had existed earlier, the strategic partnership got cemented when National Security Adviser-level dialogue was established in 1999 between Brajesh Mishra and Gen. (retd.) David Ivry. Gen. Ivry was a former Air Force chief who had led the air raid on Osirak, the Iraqi nuclear reactor, in 1981. Incidentally, Israel was one of the few countries that showed a complete understanding of India’s decision to undertake the nuclear tests in 1998. This reinforced both the defense and the counter-terrorism cooperation relationship.
By 2000, India was acquiring surface-to-air missiles (Barak 1) and UAVs (unmanned aerial vehicles) from Israel. Subsequently, the refurbishing of MiG-21 aircraft employed Israeli avionics. During the 1999 Kargil war, Israel assisted with laser-guidance kits mated with gravity bombs, carried by the Mirage 2000 aircraft. With U.S. concurrence, Israel sold India the Phalcon airborne early warning system and mounted on the Russian Il-76, provided AWACS capability. Subsequent acquisitions have included Spike anti-tank guided missiles and the long range surface-to-air missiles in both the naval and land versions. Today, Israel has emerged as the third-largest defense supplier for India and accounts for over 40% of Israel’s defense exports.
Commercial relations between the diamond traders in Gujarat and Israel had existed before 1992, but now annual trade grew from $200 million to nearly $5 billion with gems and jewelry accounting for nearly 40%. Gradually, Science and technology, agriculture, biotech, and space emerged as new areas of cooperation.Tourism provided an impetus to people-to-people relations. India emerged as the preferred destination for young Israelis wanting to unwind after their compulsory military service and Hebrew signage in Varanasi, Manali, and Goa is a common sight. Ambassador Pavan Kapoor was being quite mattered of fact when he described the Modi visit as a ‘coming out visit’ for the relationship.
Which of the following statement is true according to the passage?
I. Foreign Minister Shimon Peres first visited India in 1993, and Jaswant Singh reciprocated in 2000; President Ezer Weizman came in 1997, while President Pranab Mukherji’s visit only took place in 2015
II. Foreign Minister Shimon Peres first visited India in 1993, and Jaswant Singh reciprocated in 2001; President Ezer Weizman came in 1997, while President Pranab Mukherji’s visit only took place in 2015
III. Foreign Minister Shimon Peres first visited India in 1993, and Jaswant Singh reciprocated in 2000; President Ezer Weizman came in 1998, while President Pranab Mukherji’s visit only took place in 2015
(A) Only I
(B) Only II
(C) Only III
(D) Both I and III
-
-
-
-
Correct
Incorrect
Foreign Minister Shimon Peres first visited India in 1993, and Jaswant Singh reciprocated in 2000; President Ezer Weizman came in 1997, while President Pranab Mukherji’s visit only took place in 2015.
Unattempted
Foreign Minister Shimon Peres first visited India in 1993, and Jaswant Singh reciprocated in 2000; President Ezer Weizman came in 1997, while President Pranab Mukherji’s visit only took place in 2015.
-
Question 5 of 80
5. Question
DIRECTION : READ THE FOLLOWING PASSAGE AND ANSWER THE QUESTION THAT FOLLOW. YOUR ANSWER TO THIS QUESTION SHOULD BE BASED ON THE PASSAGE ONLY.
As high-level visits go, Prime Minister Narendra Modi’s visit to Israel was bound to attract superlatives like ‘historic’ and ‘groundbreaking’. Still, it is clear that the buzz in the relationship is on account of Mr. Modi’s personal diplomatic style and his host’s equally warm response.
Israel’s Prime Minister Benjamin Netanyahu set the tone when he welcomed Mr. Modi at the airport together with the spiritual leaders of all the major faiths in the region, an honor traditionally reserved for the U.S. President and the Pope. Mr. Modi’s trademark bear hugs were reciprocated, three at the airport, and by the time the visit ended, the TV commentators had lost count!
Mr. Netanyahu gushingly described the relationship between the two countries as “a marriage made in heaven”, but behind the success was a receptive political backdrop as well as the careful planning undertaken by both sides.
Marking 25 years of establishing diplomatic relations between the two countries added to the historic character of an Indian Prime Minister’s first visit to a country that had quietly emerged as a strong defense partner. There had been high-level exchanges but the Indian response was cautious. Foreign Minister Shimon Peres first visited India in 1993, and Jaswant Singh reciprocated in 2000; President Ezer Weizman came in 1997, while President Pranab Mukherji’s visit only took place in 2015. The first Israeli Prime Minister to visit India was Ariel Sharon in 2003, and from the time Mr. Modi came to power, a return visit was a certainty. The two leaders had met on the margins of the UN General Assembly and continued their relationship by tweeting their greetings on Hanukkah and Diwali.
Yet, official-level exchanges between the two countries have been intensive, beginning with the visit of Foreign Secretary J.N. Dixit in early 1993. While relations between Mossad and Research and Analysis Wing had existed earlier, the strategic partnership got cemented when National Security Adviser-level dialogue was established in 1999 between Brajesh Mishra and Gen. (retd.) David Ivry. Gen. Ivry was a former Air Force chief who had led the air raid on Osirak, the Iraqi nuclear reactor, in 1981. Incidentally, Israel was one of the few countries that showed a complete understanding of India’s decision to undertake the nuclear tests in 1998. This reinforced both the defense and the counter-terrorism cooperation relationship.
By 2000, India was acquiring surface-to-air missiles (Barak 1) and UAVs (unmanned aerial vehicles) from Israel. Subsequently, the refurbishing of MiG-21 aircraft employed Israeli avionics. During the 1999 Kargil war, Israel assisted with laser-guidance kits mated with gravity bombs, carried by the Mirage 2000 aircraft. With U.S. concurrence, Israel sold India the Phalcon airborne early warning system and mounted on the Russian Il-76, provided AWACS capability. Subsequent acquisitions have included Spike anti-tank guided missiles and the long range surface-to-air missiles in both the naval and land versions. Today, Israel has emerged as the third-largest defense supplier for India and accounts for over 40% of Israel’s defense exports.
Commercial relations between the diamond traders in Gujarat and Israel had existed before 1992, but now annual trade grew from $200 million to nearly $5 billion with gems and jewelry accounting for nearly 40%. Gradually, Science and technology, agriculture, biotech, and space emerged as new areas of cooperation.Tourism provided an impetus to people-to-people relations. India emerged as the preferred destination for young Israelis wanting to unwind after their compulsory military service and Hebrew signage in Varanasi, Manali, and Goa is a common sight. Ambassador Pavan Kapoor was being quite mattered of fact when he described the Modi visit as a ‘coming out visit’ for the relationship.
Which of the following statement is not true according to the passage?
I. By 2000, India was acquiring surface-to- surface (Barak 1) and UAVs (unmanned aerial vehicles) from Israel.
II. During the 1999 Kargil war, Israel assisted with laser-guidance kits mated with gravity bombs, carried by the Mirage 2000 aircraft.
III. Israel sold India the Phalcon airborne early warning system and mounted on the Russian Jl-76, provided AWACS capability.
(A) Only I
(B) Only II
(C) Both I and II
(D) Both I and III
-
-
-
-
Correct
Incorrect
Israel sold India the Phalcon airborne early warning system and mounted on the Russian Il-76, provided AWACS capability.
By 2000, India was acquiring surface-to-air missiles (Barak 1) and UAVs (unmanned aerial vehicles) from Israel.
Unattempted
Israel sold India the Phalcon airborne early warning system and mounted on the Russian Il-76, provided AWACS capability.
By 2000, India was acquiring surface-to-air missiles (Barak 1) and UAVs (unmanned aerial vehicles) from Israel.
-
Question 6 of 80
6. Question
DIRECTION : READ THE FOLLOWING PASSAGE AND ANSWER THE QUESTIONS THAT FOLLOW. YOUR ANSWERS TO THESE QUESTIONS SHOULD BE BASED ON THE PASSAGE ONLY.
Many cities haves distributed standardized recycling containers to all households with directions that read: “We would prefer that you use this new container as your primary recycling container as this will expedite pick-up of recyclables. Additional recycling containers may be purchased from the City.”
According to the passage, which of the following is true about the new containers?
(A) The new containers are far better than other containers in every way.
(B) The new containers will help increase the efficiency of the recycling program.
(C) The new containers hold more than the old containers did.
(D) The new containers are less expensive than the old containers.
-
-
-
-
Correct
Incorrect
The passage state use of the new containers will expedite pick-up of recyclables. This indicates that the new containers will make the recycling program more efficient.
Unattempted
The passage state use of the new containers will expedite pick-up of recyclables. This indicates that the new containers will make the recycling program more efficient.
-
Question 7 of 80
7. Question
DIRECTION : READ THE FOLLOWING PASSAGE AND ANSWER THE QUESTION THAT FOLLOW. YOUR ANSWER TO THIS QUESTION SHOULD BE BASED ON THE PASSAGE ONLY.
Cuttlefish are full of personality, as behavioral ecologist Alexandra Schnell found out while researching the cephalopod's potential to display self-control. ” Self-control is thought to be the cornerstone of intelligence, as it is an important prerequisite for complex decision-making and planning for the future,” says Schnell .
[Schnell's] study used a modified version of the ” marshmallow test ” – During the original marshmallow test, psychologist Walter Mischel presented children between age four and six with one marshmallow. He told them that if they waited 15 minutes and didn't eat it, he would give them a second marshmallow. A long-term follow-up study showed that the children who waited for the second marshmallow had more success later in life. The cuttlefish version of the experiment looked a lot different. The researchers worked with six cuttlefish under nine months old and presented them with seafood instead of sweets. (Preliminary experiments showed that cuttlefishes' favorite food is live grass shrimp, while raw prawns are so-so and Asian shore crab is nearly unacceptable.) Since the researchers couldn't explain to the cuttlefish that they would need to wait for their shrimp, they trained them to recognize certain shapes that indicated when a food item would become available. The symbols were pasted on transparent drawers so that the cuttlefish could see the food that was stored inside. One drawer, labeled with a circle to mean “immediate,” held raw king prawn. Another drawer, labeled with a triangle to mean “delayed,” held live grass shrimp. During a control experiment, square labels meant “never.”
“If their self-control is flexible and I hadn't just trained them to wait in any context, you would expect the cuttlefish to take the immediate reward [in the control], even if it's their second preference,” says Schnell . . . and that's what they did. That showed the researchers that cuttlefish wouldn't reject the prawns if it was the only food available. In the experimental trials, the cuttlefish didn't jump on the prawns if the live grass shrimp were labeled with a triangle – many waited for the shrimp drawer to open up. Each time the cuttlefish showed it could wait, the researchers tacked another ten seconds on to the next round of waiting before releasing the shrimp. The longest that a cuttlefish waited was 130 seconds.
Schnell [says] that the cuttlefish usually sat at the bottom of the tank and looked at the two food items while they waited, but sometimes, they would turn away from the king prawn “as if to distract themselves from the temptation of the immediate reward.” In past studies, humans, chimpanzees, parrots and dogs also tried to distract themselves while waiting for a reward.
Not every species can use self-control, but most of the animals that can share another trait in common: long, social lives. Cuttlefish, on the other hand, are solitary creatures that don't form relationships even with mates or young. “We don't know if living in a social group is important for complex cognition unless we also show those abilities are lacking in less social species,” says . . . comparative psychologist Jennifer Vonk.
All of the following constitute a point of difference between the “original” and “modified” versions of the marshmallow test EXCEPT that:
(A) the former was performed over a longer time span than the latter.
(B) the former correlated self-control and future success, while the latter correlated self-control and survival advantages.
(C) the former had human subjects, while the latter had cuttlefish.
(D) the former used verbal communication with its subjects, while the latter had to develop a symbolic means of communication.
-
-
-
-
Correct
Incorrect
In this question, we have to pick a choice that is not a difference between the original and modified versions of the marshmallow test. Option A is the difference. The time difference can be seen in the first paragraph. C is an obvious difference which need not be explained. D is also an obvious difference. B is the best choice as nowhere in the passage is it implied that the latter correlated survival advantages
Unattempted
In this question, we have to pick a choice that is not a difference between the original and modified versions of the marshmallow test. Option A is the difference. The time difference can be seen in the first paragraph. C is an obvious difference which need not be explained. D is also an obvious difference. B is the best choice as nowhere in the passage is it implied that the latter correlated survival advantages
-
Question 8 of 80
8. Question
DIRECTION : READ THE FOLLOWING PASSAGE AND ANSWER THE QUESTIONS THAT FOLLOW. YOUR ANSWERS TO THESE QUESTIONS SHOULD BE BASED ON THE PASSAGE ONLY.
Ratatouille is a dish that has grown in popularity over the last few years. It features eggplant, zucchini, tomatoes, peppers, and garlic; chopped, mixed, sautéed, and finally, cooked slowly over low heat. As the vegetables cook slowly, they make their own broth, which may be extended with a little tomato paste. The name ratatouille comes from the French word touiller, meaning to stir or mix together.
Which of the following is the correct order of steps for making ratatouille?
(A) chop vegetables, add tomato paste, stir or mix together
(B) mix the vegetables together, sauté them, and add tomato paste
(C) cook the vegetables slowly, mix them together, add tomato paste
(D) add tomato paste to extend the broth and cook slowly over low heat
-
-
-
-
Correct
Incorrect
Unattempted
-
Question 9 of 80
9. Question
DIRECTION : READ THE FOLLOWING PASSAGE AND ANSWER THE QUESTION THAT FOLLOW. YOUR ANSWER TO THIS QUESTION SHOULD BE BASED ON THE PASSAGE ONLY.
I have elaborated – a framework for analyzing the contradictory pulls on [Indian] nationalist ideology in its struggle against the dominance of colonialism and the resolution it offered to those contradictions. Briefly, this resolution was built around a separation of the domain of culture into two spheres – the material and the spiritual. It was in the material sphere that the claims of Western civilization were the most powerful. Science, technology, rational forms of economic organization, modern methods of statecraft – these had given the European countries the strength to subjugate the non-European people . To overcome this domination, the colonized people had to learn those superior techniques of organizing material life and incorporate them within their own cultures. But this could not mean the imitation of the West in every aspect of life, for then the very distinction between the West and the East would vanish – the self-identity of national culture would itself be threatened. The discourse of nationalism shows that the material/spiritual distinction was condensed into an analogous, but ideologically far more powerful, dichotomy: that between the outer and the inner. Applying the inner/outer distinction to the matter of concrete day-to-day living separates the social space into ghar and bāhir, the home and the world. The world is the external, the domain of the material; the home represents one's inner spiritual self, one's true identity. The world is a treacherous terrain of the pursuit of material interests, where practical considerations reign supreme. It is also typically the domain of the male. The home in its essence must remain unaffected by the profane activities of the material world – and woman is its representation. And so one gets an identification of social roles by gender to correspond with the separation of the social space into ghar and bāhir.
The colonial situation, and the ideological response of nationalism to the critique of Indian tradition, introduced an entirely new substance to [these dichotomies] and effected their transformation. The material/spiritual dichotomy, to which the terms world and home corresponded, had acquired . a very special significance in the nationalist mind. The world was where the European power had challenged the non-European peoples and, by virtue of its superior material culture, had subjugated them. But, the nationalists asserted, it had failed to colonize the inner, essential, identity of the East which lay in its distinctive, and superior, spiritual culture. [I]n the entire phase of the national struggle, the crucial need was to protect, preserve and strengthen the inner core of the national culture, its spiritual essence.
Once we match this new meaning of the home/world dichotomy with the identification of social roles by gender, we get the ideological framework within which nationalism answered the women's question. It would be a grave error to see in this, as liberals are apt to in their despair at the many marks of social conservatism in nationalist practice, a total rejection of the West. Quite the contrary: the nationalist paradigm in fact supplied an ideological principle of selection.
Which one of the following, if true, would weaken the author's claims in the passage?
(A) The colonial period saw the hybridisation of Indian culture in all realms as it came in contact with British/European culture.
(B) Indian nationalists rejected the cause of English education for women during the colonial period.
(C) The Industrial Revolution played a crucial role in shaping the economic prowess of Britain in the eighteenth century.
(D) Forces of colonial modernity played an important role in shaping anti-colonial Indian nationalism.
-
-
-
-
Correct
Incorrect
The passage says that Indian nationalist borrowed from the material sphere, not the spiritual sphere. A says that “there was hybridization of Indian culture in all spheres”. This weakens the author's claims in the passage.
Unattempted
The passage says that Indian nationalist borrowed from the material sphere, not the spiritual sphere. A says that “there was hybridization of Indian culture in all spheres”. This weakens the author's claims in the passage.
-
Question 10 of 80
10. Question
DIRECTION : READ THE FOLLOWING PASSAGE AND ANSWER THE QUESTION THAT FOLLOW. YOUR ANSWER TO THIS QUESTION SHOULD BE BASED ON THE PASSAGE ONLY.
I have elaborated – a framework for analyzing the contradictory pulls on [Indian] nationalist ideology in its struggle against the dominance of colonialism and the resolution it offered to those contradictions. Briefly, this resolution was built around a separation of the domain of culture into two spheres – the material and the spiritual. It was in the material sphere that the claims of Western civilization were the most powerful. Science, technology, rational forms of economic organization, modern methods of statecraft – these had given the European countries the strength to subjugate the non-European people . To overcome this domination, the colonized people had to learn those superior techniques of organizing material life and incorporate them within their own cultures. But this could not mean the imitation of the West in every aspect of life, for then the very distinction between the West and the East would vanish – the self-identity of national culture would itself be threatened. The discourse of nationalism shows that the material/spiritual distinction was condensed into an analogous, but ideologically far more powerful, dichotomy: that between the outer and the inner. Applying the inner/outer distinction to the matter of concrete day-to-day living separates the social space into ghar and bāhir, the home and the world. The world is the external, the domain of the material; the home represents one's inner spiritual self, one's true identity. The world is a treacherous terrain of the pursuit of material interests, where practical considerations reign supreme. It is also typically the domain of the male. The home in its essence must remain unaffected by the profane activities of the material world – and woman is its representation. And so one gets an identification of social roles by gender to correspond with the separation of the social space into ghar and bāhir.
The colonial situation, and the ideological response of nationalism to the critique of Indian tradition, introduced an entirely new substance to [these dichotomies] and effected their transformation. The material/spiritual dichotomy, to which the terms world and home corresponded, had acquired . a very special significance in the nationalist mind. The world was where the European power had challenged the non-European peoples and, by virtue of its superior material culture, had subjugated them. But, the nationalists asserted, it had failed to colonize the inner, essential, identity of the East which lay in its distinctive, and superior, spiritual culture. [I]n the entire phase of the national struggle, the crucial need was to protect, preserve and strengthen the inner core of the national culture, its spiritual essence.
Once we match this new meaning of the home/world dichotomy with the identification of social roles by gender, we get the ideological framework within which nationalism answered the women's question. It would be a grave error to see in this, as liberals are apt to in their despair at the many marks of social conservatism in nationalist practice, a total rejection of the West. Quite the contrary: the nationalist paradigm in fact supplied an ideological principle of selection.
On the basis of the information in the passage, all of the following are true about the spiritual/material dichotomy of Indian nationalism EXCEPT that it:
(A) constituted the premise of the ghar/bāhir dichotomy.
(B) represented a continuation of age-old oppositions in Indian culture.
(C) helped in safeguarding the identity of Indian nationalism.
(D) was not as ideologically powerful as the inner/outer dichotomy.
-
-
-
-
Correct
Incorrect
Option D is true, as it can be verified from the first sentence of the second paragraph. Option A is easy to eliminate as it is the very theme of the passage. The second last paragraph provides ample evidence for option C. There is no evidence for choice B. It is the right answer.
Unattempted
Option D is true, as it can be verified from the first sentence of the second paragraph. Option A is easy to eliminate as it is the very theme of the passage. The second last paragraph provides ample evidence for option C. There is no evidence for choice B. It is the right answer.
-
Question 11 of 80
11. Question
DIRECTION : READ THE FOLLOWING PASSAGE AND ANSWER THE QUESTIONS THAT FOLLOW. YOUR ANSWERS TO THESE QUESTIONS SHOULD BE BASED ON THE PASSAGE ONLY.
We cannot travel outside our neighbourhood without passports. We must wear the same plainclothes. We must exchange our houses every ten years. We cannot avoid labour. We all go to bed at the same time . We have religious freedom, but we cannot deny that the soul dies with the body, since 'but for the fear of punishment, they would have nothing but contempt for the laws and customs of society'. In More's time, for much of the population, given the plenty and security on offer, such restraints would not have seemed overly unreasonable. For modern readers, however, Utopia appears to rely upon relentless transparency, the repression of variety, and the curtailment of privacy. Utopia provides security: but at what price' In both its external and internal relations, indeed, it seems perilously dystopian.
Such a conclusion might be fortified by examining selectively the tradition which follows more on these points. This often portrays societies where. . .'it would be almost impossible for man to be depraved, or wicked'. This is achieved both through institutions and mores, which underpin the common life. The passions are regulated and inequalities of wealth and distinction are minimized. Needs, vanity, and emulation are restrained, often by prizing equality and holding riches in contempt. The desire for public power is curbed. Marriage and sexual intercourse are often controlled: in Tommaso Campanella's The City of the Sun (1623), the first great literary utopia after More's, relations are forbidden to men before the age of twenty-one and women before nineteen. Communal child-rearing is normal; for Campanella this commences at age two. Greater simplicity of life, 'living according to nature', is often a result: the desire for simplicity and purity are closely related. People become more alike in appearance, opinion, and outlook than they often have been. Unity, order, and homogeneity thus prevail at the cost of individuality and diversity. This model, as J. C. Davis demonstrates, dominated early modern utopianism. And utopian homogeneity remains a familiar theme well into the twentieth century.
Given these considerations, it is not unreasonable to take as our starting point here the hypothesis that utopia and dystopia evidently share more in common than is often supposed. Indeed, they might be twins, the progeny of the same parents. Insofar as this proves to be the case, my linkage of both here will be uncomfortably close for some readers. Yet we should not mistake this argument for the assertion that all utopias are, or tend to produce, dystopias. Those who defend this proposition will find that their association here is not nearly close enough. For we have only to acknowledge the existence of thousands of successful intentional communities in which a cooperative ethos predominates and where harmony without coercion is the rule to set aside such an assertion. Here the individual's submersion in the group is consensual (though this concept is not unproblematic). It results not in enslavement but voluntary submission to group norms. Harmony is achieved without harming others.
All of the following statements can be inferred from the passage EXCEPT that:
(A) utopian and dystopian societies are twins, the progeny of the same parents.
(B) utopian societies exist in a long tradition of literature dealing with imaginary people practicing imaginary customs, in imaginary worlds.
(C) many conceptions of utopian societies emphasise the importance of social uniformity and cultural homogeneity.
(D) it is possible to see utopias as dystopias, with a change in perspective, because one person's utopia could be seen as another's dystopia.
-
-
-
-
Correct
Incorrect
The answer to this question can be found in the last paragraph. The author starts by saying that “utopia and dystopia evidently share more in common…. indeed, they might be twins…”. He further adds “Yet we should not mistake this argument…” From this we can say that option (A) is definitely incorrect, and cannot be inferred. You might wonder as to the evidence for option (B). But the author mentions “More”, who was the first author of a book on Utopia, and further mentions Tommaso who also wrote a book on Utopia. We have enough evidence in the passage that shows that in literature we have enough material that have dealt with the idea of Utopia. option (C) can be inferred from the last sentence of second paragraph, and option (D) can be inferred from last sentence of first paragraph.
Unattempted
The answer to this question can be found in the last paragraph. The author starts by saying that “utopia and dystopia evidently share more in common…. indeed, they might be twins…”. He further adds “Yet we should not mistake this argument…” From this we can say that option (A) is definitely incorrect, and cannot be inferred. You might wonder as to the evidence for option (B). But the author mentions “More”, who was the first author of a book on Utopia, and further mentions Tommaso who also wrote a book on Utopia. We have enough evidence in the passage that shows that in literature we have enough material that have dealt with the idea of Utopia. option (C) can be inferred from the last sentence of second paragraph, and option (D) can be inferred from last sentence of first paragraph.
-
Question 12 of 80
12. Question
DIRECTION : READ THE FOLLOWING PASSAGE AND ANSWER THE QUESTIONS THAT FOLLOW. YOUR ANSWERS TO THESE QUESTIONS SHOULD BE BASED ON THE PASSAGE ONLY.
We cannot travel outside our neighbourhood without passports. We must wear the same plainclothes. We must exchange our houses every ten years. We cannot avoid labour. We all go to bed at the same time . We have religious freedom, but we cannot deny that the soul dies with the body, since 'but for the fear of punishment, they would have nothing but contempt for the laws and customs of society'. In More's time, for much of the population, given the plenty and security on offer, such restraints would not have seemed overly unreasonable. For modern readers, however, Utopia appears to rely upon relentless transparency, the repression of variety, and the curtailment of privacy. Utopia provides security: but at what price' In both its external and internal relations, indeed, it seems perilously dystopian.
Such a conclusion might be fortified by examining selectively the tradition which follows more on these points. This often portrays societies where. . .'it would be almost impossible for man to be depraved, or wicked'. This is achieved both through institutions and mores, which underpin the common life. The passions are regulated and inequalities of wealth and distinction are minimized. Needs, vanity, and emulation are restrained, often by prizing equality and holding riches in contempt. The desire for public power is curbed. Marriage and sexual intercourse are often controlled: in Tommaso Campanella's The City of the Sun (1623), the first great literary utopia after More's, relations are forbidden to men before the age of twenty-one and women before nineteen. Communal child-rearing is normal; for Campanella this commences at age two. Greater simplicity of life, 'living according to nature', is often a result: the desire for simplicity and purity are closely related. People become more alike in appearance, opinion, and outlook than they often have been. Unity, order, and homogeneity thus prevail at the cost of individuality and diversity. This model, as J. C. Davis demonstrates, dominated early modern utopianism. And utopian homogeneity remains a familiar theme well into the twentieth century.
Given these considerations, it is not unreasonable to take as our starting point here the hypothesis that utopia and dystopia evidently share more in common than is often supposed. Indeed, they might be twins, the progeny of the same parents. Insofar as this proves to be the case, my linkage of both here will be uncomfortably close for some readers. Yet we should not mistake this argument for the assertion that all utopias are, or tend to produce, dystopias. Those who defend this proposition will find that their association here is not nearly close enough. For we have only to acknowledge the existence of thousands of successful intentional communities in which a cooperative ethos predominates and where harmony without coercion is the rule to set aside such an assertion. Here the individual's submersion in the group is consensual (though this concept is not unproblematic). It results not in enslavement but voluntary submission to group norms. Harmony is achieved without …harming others.
Following from the passage, which one of the following may be seen as a characteristic of a utopian society?
(A) The regulation of homogeneity through promoting competitive heterogeneity.
(B) A society where public power is earned through merit rather than through privilege.
(C) Institutional surveillance of every individual to ensure his/her security and welfare.
(D) A society without any laws to restrain one's individuality.
-
-
-
-
Correct
Incorrect
This is a very simple question. It can be easily answered. There is no mention of “competitive heterogeneity” in the passage. Thus (A) goes out. There is no mention of (B). (C) is true, as there is enough evidence for it in the first paragraph. (D) is the exact opposite of what utopian society wants. It wants homogeneity and uniformity, which would imply restraints on one's individuality.
Unattempted
This is a very simple question. It can be easily answered. There is no mention of “competitive heterogeneity” in the passage. Thus (A) goes out. There is no mention of (B). (C) is true, as there is enough evidence for it in the first paragraph. (D) is the exact opposite of what utopian society wants. It wants homogeneity and uniformity, which would imply restraints on one's individuality.
-
Question 13 of 80
13. Question
DIRECTION : READ THE FOLLOWING PASSAGE AND ANSWER THE QUESTIONS THAT FOLLOW. YOUR ANSWERS TO THESE QUESTIONS SHOULD BE BASED ON THE PASSAGE ONLY.
Ratatouille is a dish that has grown in popularity over the last few years. It features eggplant, zucchini, tomatoes, peppers, and garlic; chopped, mixed, sautéed, and finally, cooked slowly over low heat. As the vegetables cook slowly, they make their own broth, which may be extended with a little tomato paste. The name ratatouille comes from the French word touiller, meaning to stir or mix together.
Ratatouille can best be described as a
(A) French pastry.
(B) sauce to put over vegetables.
(C) pasta dish extended with tomato paste.
(D) vegetable stew.
-
-
-
-
Correct
Incorrect
The main part of the passage describes how to cook vegetables. Only choice (D) indicates that vegetables are included in the dish. The other choices are not reflected in the passage.
Unattempted
The main part of the passage describes how to cook vegetables. Only choice (D) indicates that vegetables are included in the dish. The other choices are not reflected in the passage.
-
Question 14 of 80
14. Question
DIRECTION : READ THE FOLLOWING PASSAGE AND ANSWER THE QUESTIONS THAT FOLLOW. YOUR ANSWERS TO THESE QUESTIONS SHOULD BE BASED ON THE PASSAGE ONLY.
The competitive civil-service system is designed to give candidates fair and equal treatment and to ensure that federal applicants are hired based on objective criteria. Hiring has to be based solely on a candidate’s knowledge, skills, and abilities (which you’ll sometimes see abbreviated as ksa), and not on external factors such as race, religion, sex, and so on. Whereas employers in the private sector can hire employees for subjective reasons, federal employers must be able to justify their decision with objective evidence that the candi- date is qualified.
The federal government’s practice of hiring on the basis of ksa frequently results in the hiring of employees
(A) based on race, religion, sex, and so forth.
(B) who are unqualified for the job.
(C) who are qualified for the job.
(D) on the basis of subjective judgment.
-
-
-
-
Correct
Incorrect
Unattempted
-
Question 15 of 80
15. Question
DIRECTION : READ THE FOLLOWING PASSAGE AND ANSWER THE QUESTIONS THAT FOLLOW. YOUR ANSWERS TO THESE QUESTIONS SHOULD BE BASED ON THE PASSAGE ONLY.
An ecosystem is a group of animals and plants living in a specific region and interacting with one another and with their physical environment. Ecosystems include physical and chemical com- ponents, such as soils, water, and nutrients that support the organisms living there. These organisms may range from large animals to microscopic bacteria. Ecosystems also can be thought of as the interactions among all organisms in a given habitat; for instance, one species may serve as food for another. People are part of the ecosystems where they live and work. Human activities can harm or destroy local ecosystems unless actions such as land development for housing or businesses are carefully planned to conserve and sustain the ecology of the area. An important part of ecosystem management involves finding ways to protect and enhance economic and social well-being while protecting local ecosystems.
What is the main idea of the passage?
(A) An ecosystem is a community that includes animals, plants, and microscopic bacteria.
(B) Human activities can do great damage to local ecosystems, so human communities should be cautiously planned.
(C) In managing the ecology of an area, it is important to protect both human interests and the interests of other members of local ecosystems.
(D) People should remember that they are a part of the ecosystems where they live and work.
-
-
-
-
Correct
Incorrect
Unattempted
-
Question 16 of 80
16. Question
DIRECTION : READ THE FOLLOWING PASSAGE AND ANSWER THE QUESTIONS THAT FOLLOW. YOUR ANSWERS TO THESE QUESTIONS SHOULD BE BASED ON THE PASSAGE ONLY.
An ecosystem is a group of animals and plants living in a specific region and interacting with one another and with their physical environment. Ecosystems include physical and chemical com- ponents, such as soils, water, and nutrients that support the organisms living there. These organisms may range from large animals to microscopic bacteria. Ecosystems also can be thought of as the interactions among all organisms in a given habitat; for instance, one species may serve as food for another. People are part of the ecosystems where they live and work. Human activities can harm or destroy local ecosystems unless actions such as land development for housing or businesses are carefully planned to conserve and sustain the ecology of the area. An important part of ecosystem management involves finding ways to protect and enhance economic and social well-being while protecting local ecosystems.
Which of the following best sums up activities within an ecosystem?
(A) predator-prey relationships
(B) interactions among all members
(C) human-animal interactions
(D) human relationship with the environment
-
-
-
-
Correct
Incorrect
The passage defines an ecosystem as a community within which all members interrelate. Choice (A) is only one example of an interaction. The other two choices are too limited to sum up ecosystem activities.
Unattempted
The passage defines an ecosystem as a community within which all members interrelate. Choice (A) is only one example of an interaction. The other two choices are too limited to sum up ecosystem activities.
-
Question 17 of 80
17. Question
DIRECTION : READ THE FOLLOWING PASSAGE AND ANSWER THE QUESTIONS THAT FOLLOW. YOUR ANSWERS TO THESE QUESTIONS SHOULD BE BASED ON THE PASSAGE ONLY.
An ecosystem is a group of animals and plants living in a specific region and interacting with one another and with their physical environment. Ecosystems include physical and chemical com- ponents, such as soils, water, and nutrients that support the organisms living there. These organisms may range from large animals to microscopic bacteria. Ecosystems also can be thought of as the interactions among all organisms in a given habitat; for instance, one species may serve as food for another. People are part of the ecosystems where they live and work. Human activities can harm or destroy local ecosystems unless actions such as land development for housing or businesses are carefully planned to conserve and sustain the ecology of the area. An important part of ecosystem management involves finding ways to protect and enhance economic and social well-being while protecting local ecosystems.
An ecosystem can most accurately be defined as a :
(A) geographical area.
(B) community.
(C) habitat.
(D) protected environment.
-
-
-
-
Correct
Incorrect
This is the only choice that reflects the idea of interaction among all members of the group spoken of in the first sentence. The other choices are only physical settings.
Unattempted
This is the only choice that reflects the idea of interaction among all members of the group spoken of in the first sentence. The other choices are only physical settings.
-
Question 18 of 80
18. Question
DIRECTION : READ THE FOLLOWING PASSAGE AND ANSWER THE QUESTIONS THAT FOLLOW. YOUR ANSWERS TO THESE QUESTIONS SHOULD BE BASED ON THE PASSAGE ONLY.
Daffodil bulbs require well-drained soil and a sunny planting location. They should be planted in holes that are 3–6 inches deep and there should be 2–4 inches between bulbs. The bulb should be placed in the hole, pointed side up, root side down. Once the bulb is planted, water the area thoroughly.
According to the above passage, when planting daffodil bulbs, which of the following conditions is not necessary?
(A) a sunny location
(B) well-drained soil
(C) proper placement of bulbs in soil
(D) proper fertilization
-
-
-
-
Correct
Incorrect
The passage mention nothing about fertilization.
Unattempted
The passage mention nothing about fertilization.
-
Question 19 of 80
19. Question
DIRECTION : READ THE FOLLOWING PASSAGE AND ANSWER THE QUESTIONS THAT FOLLOW. YOUR ANSWERS TO THESE QUESTIONS SHOULD BE BASED ON THE PASSAGE ONLY.
Daffodil bulbs require well-drained soil and a sunny planting location. They should be planted in holes that are 3–6 inches deep and there should be 2–4 inches between bulbs. The bulb should be placed in the hole, pointed side up, root side down. Once the bulb is planted, water the area thoroughly.
According to the above passage, which of the following is true?
(A) Daffodils do best in sandy soil.
(B) Daffodil bulbs should be planted in autumn for spring blooming.
(C) It is possible to plant daffodil bulbs upside down.
(D) Daffodil bulbs require daily watering.
-
-
-
-
Correct
Incorrect
The third sentence specifically mentions that the pointed side goes up and the root side faces down. This means that there is an up side and a down side and that it is possible for the bulb to be put into the soil upside down if someone didn’t know better. The other choices may be true but are not mentioned in the passage.
Unattempted
The third sentence specifically mentions that the pointed side goes up and the root side faces down. This means that there is an up side and a down side and that it is possible for the bulb to be put into the soil upside down if someone didn’t know better. The other choices may be true but are not mentioned in the passage.
-
Question 20 of 80
20. Question
DIRECTION : READ THE FOLLOWING PASSAGE AND ANSWER THE QUESTIONS THAT FOLLOW. YOUR ANSWERS TO THESE QUESTIONS SHOULD BE BASED ON THE PASSAGE ONLY.
Many cities haves distributed standardized recycling containers to all households with directions that read: “We would prefer that you use this new container as your primary recycling container as this will expedite pick-up of recyclables. Additional recycling containers may be purchased from the City.”
According to the passage, each household :
(A) may only use one recycling container.
(B) must use the new recycling container.
(C) should use the new recycling container.
(D) must buy a new recycling container.
-
-
-
-
Correct
Incorrect
The passage indicate that the city prefers, but does not require, use of its new container, and that the customers may use more than one container if they purchase an additional one.
Unattempted
The passage indicate that the city prefers, but does not require, use of its new container, and that the customers may use more than one container if they purchase an additional one.
-
Question 21 of 80
21. Question
Directions : Examine information given below and answer the questions that follow:
Five executives A, B, C, D & E of Indian Corporation hold a conference in New Delhi.
Mr. A converses in Hindi and Tamil.
Mr. B converses in Hindi and English.
Mr. C converses in English and Tamil.
Mr. D converses in Bengali and Hindi.
Mr. E, a native Tamil, can also converse in Bengali.
Which of the following can act as an interpreter when Mr. C and Mr. D wish to converse?
(A) Only Mr. A
(B) Only Mr. B
(C) Only Mr. E
(D) Any of the other three executives
-
-
-
-
Correct
Incorrect
When Mr. C and Mr. D converse, they can use English, Tamil, Bengali and Hindi interpreter between them. Mr. A speaks Hindi and Tamil and Mr. B speaks English and Hindi. Mr. E speaks bengali and Tamil.
Unattempted
When Mr. C and Mr. D converse, they can use English, Tamil, Bengali and Hindi interpreter between them. Mr. A speaks Hindi and Tamil and Mr. B speaks English and Hindi. Mr. E speaks bengali and Tamil.
-
Question 22 of 80
22. Question
Directions : Examine information given below and answer the questions that follow:
Five executives A, B, C, D & E of Indian Corporation hold a conference in New Delhi.
Mr. A converses in Hindi and Tamil.
Mr. B converses in Hindi and English.
Mr. C converses in English and Tamil.
Mr. D converses in Bengali and Hindi.
Mr. E, a native Tamil, can also converse in Bengali.
Which of the following pairs cannot converse without an interpreter?
(A) Mr. B and Mr. E
(B) Mr. A and Mr. B
(C) Mr. A and Mr. C
(D) Mr. B and Mr. D
-
-
-
-
Correct
Incorrect
Mr. B understands English and Hindi, while Mr. E speaks two other languages, Bengali and Tamil.
Unattempted
Mr. B understands English and Hindi, while Mr. E speaks two other languages, Bengali and Tamil.
-
Question 23 of 80
23. Question
An Indian who is a scientist as well as a politician is represented in the following diagram by an alphabet. Find the alphabet and choose the correct response.
(A) b
(B) a
(C) c
(D) g
-
-
-
-
Correct
Incorrect
Since, the region ‘a’ is common between Indian scientists and politicians, therefore it represents Indian scientists who are also politicians.
Unattempted
Since, the region ‘a’ is common between Indian scientists and politicians, therefore it represents Indian scientists who are also politicians.
-
Question 24 of 80
24. Question
Directions : Examine the following information carefully and answer the questions that follow:
Nine cities A, B, C, D, E, F, G, H and I are connected with a oneway or a two way route.
The oneway routes are from A to F; D to A; D to F; H to D; D to B; D to E; E to B; B to G; E to F; E to C.
The two way routes are between G and H; H and A; A and I; I and C; F and C.
No other routes exist except the above mentioned routes.
Due to Bandh call given by a political party, no one is allowed to pass through city D. Then, which city can not be reached from any other city?
(A) G
(B) B
(C) E
(D) F
-
-
-
-
Correct
Incorrect
Unattempted
-
Question 25 of 80
25. Question
Directions : Study the following graph and answer the given question. Graph shows the variation in literacy per cent with the increase in the population of four different districts of a newly formulated state.
Which district shows two different behaviour of literacy per cent with the increase in population?
(A) A
(B) B
(C) C
(D) D
-
-
-
-
Correct
Incorrect
District A & B are showing a fixed behaviour, while C is showing 3 distinct behaviours but District D shows two different types of behaviour because firstly, the graph is increasing proportionally and then constant throughout the increasing population.
Unattempted
District A & B are showing a fixed behaviour, while C is showing 3 distinct behaviours but District D shows two different types of behaviour because firstly, the graph is increasing proportionally and then constant throughout the increasing population.
-
Question 26 of 80
26. Question
In a row of boys facing West, K is twelfth from the left end and fourth to the right of L. What is the position of L from the left ends of that row?
(A) 8th
(B) 9th
(C) 7th
(D) 4th
-
-
-
-
Correct
Incorrect
From the left ends the position of L is = 12th – 4th = 8th
Unattempted
From the left ends the position of L is = 12th – 4th = 8th
-
Question 27 of 80
27. Question
In a row of boys there are 50 boys and all of them are facing North, Sahaj is 19th from the left end and fourth to the right of Mahak, what is the position of Mahak from the right ends of that row?
(A) 28th
(B) 36th
(C) 37th
(D) 25th
-
-
-
-
Correct
Incorrect
From the left ends the position of Mahak is = 19th – 4th = 15th
Hence, the position of Mahak from the right end is
= Number of Boys – position of Mahak from the left ends + 1
= 50 – 15 + 1 = 36th
Unattempted
From the left ends the position of Mahak is = 19th – 4th = 15th
Hence, the position of Mahak from the right end is
= Number of Boys – position of Mahak from the left ends + 1
= 50 – 15 + 1 = 36th
-
Question 28 of 80
28. Question
In a row of Children facing north, K is 15th to the left of H, who is 22nd from the right end. If P is 14th from the left end and 6th to the right of K, how many children are there in that row?
(A) 41
(B) 38
(C) 42
(D) 44
-
-
-
-
Correct
Incorrect
Here, P is 14th from the left end and 6th to the right of K.
Hence, K is 8th from the left side.
Again, K is 15th to the left of H, who is 22nd from the right end.
Hence, K is 37th from the right side.
Therefore, the total number of Children is
= K’s position from the right ends + K’s position from the left ends – 1
= 37 + 8 – 1 = 45 – 1 = 44
Unattempted
Here, P is 14th from the left end and 6th to the right of K.
Hence, K is 8th from the left side.
Again, K is 15th to the left of H, who is 22nd from the right end.
Hence, K is 37th from the right side.
Therefore, the total number of Children is
= K’s position from the right ends + K’s position from the left ends – 1
= 37 + 8 – 1 = 45 – 1 = 44
-
Question 29 of 80
29. Question
Direction: Read the following information carefully to answer the questions given below.
Devesh, Gautam, Aditya, Byom, Chandan, Harish, Fahad, and Emi are eight persons, having a different height. Devesh is shorter than Aditya, but taller than Gautam. Emi is taller than Harish but shorter than Chandan. Byom is shorter than Devesh but taller than Fahad. Chandan is shorter than Gautam. Gautam is not as tall as Fahad.
Which of the following is 5th shortest?
(A) Byom
(B) Chandan
(C) Fahad
(D) Gautam
-
-
-
-
Correct
Incorrect
Aditya > Devesh > Byom > Fahad > Gautam > Chandan > Emi > Harish
Unattempted
Aditya > Devesh > Byom > Fahad > Gautam > Chandan > Emi > Harish
-
Question 30 of 80
30. Question
Direction: Read the following information carefully to answer the questions given below.
Devesh, Gautam, Aditya, Byom, Chandan, Harish, Fahad, and Emi are eight persons, having a different height. Devesh is shorter than Aditya, but taller than Gautam. Emi is taller than Harish but shorter than Chandan. Byom is shorter than Devesh but taller than Fahad. Chandan is shorter than Gautam. Gautam is not as tall as Fahad.
Who among the following is shortest among them?
(A) Byom
(B) Chandan
(C) Fahad
(D) Harish
-
-
-
-
Correct
Incorrect
Aditya > Devesh > Byom > Fahad > Gautam > Chandan > Emi > Harish
Unattempted
Aditya > Devesh > Byom > Fahad > Gautam > Chandan > Emi > Harish
-
Question 31 of 80
31. Question
Direction : Read the following information carefully to answer the questions given below.
There are six Investors named Kavita, Laxman, Manish, Niraj, Om and Parvati, purchased different number of shares in HUL Company. No two person purchased the same number of shares. According to the number of purchased share, three people among them invested between Om and Kavita where Om invested more than Kavita. Parvati who purchased 37 shares, purchased less than Niraj. Manish purchased 34 shares which is less than Laxman. None of them purchased between the number of shares Manish and Laxman purchased. Om did not purchase maximum share among them.
Who purchased the maximum number of shares?
(A) Manish
(B) Kavita
(C) Parvati
(D) Niraj
-
-
-
-
Correct
Incorrect
Niraj > Om > Parvati (37) > Laxman > Manish (34) > Kavita
Hence, Niraj invested highest among them.
Unattempted
Niraj > Om > Parvati (37) > Laxman > Manish (34) > Kavita
Hence, Niraj invested highest among them.
-
Question 32 of 80
32. Question
Direction : Read the following information carefully to answer the questions given below.
There are six Investors named Kavita, Laxman, Manish, Niraj, Om and Parvati, purchased different number of shares in HUL Company. No two person purchased the same number of shares. According to the number of purchased share, three people among them invested between Om and Kavita where Om invested more than Kavita. Parvati who purchased 37 shares, purchased less than Niraj. Manish purchased 34 shares which is less than Laxman. None of them purchased between the number of shares Manish and Laxman purchased. Om did not purchase maximum share among them.
Who among the following person has purchased third highest number of shares?
(A) Niraj
(B) Parvati
(C) Manish
(D) Om
-
-
-
-
Correct
Incorrect
Niraj > Om > Parvati (37) > Laxman > Manish (34) > Kavita
Hence, Parvati purchased third highest no of shares among them.
Unattempted
Niraj > Om > Parvati (37) > Laxman > Manish (34) > Kavita
Hence, Parvati purchased third highest no of shares among them.
-
Question 33 of 80
33. Question
Find the missing number in series marked by “?”
2 , 3 , 8 , 63, ?
(A) 3968
(B) 256
(C) 3863
(D) 175
-
-
-
-
Correct
Incorrect
The pattern is,
22 = 4 – 1 = 3
32 = 9 – 1 = 8
82 = 64 – 1 = 63
632 = 3969 – 1 = 3968
3968 is the missing number.
Unattempted
The pattern is,
22 = 4 – 1 = 3
32 = 9 – 1 = 8
82 = 64 – 1 = 63
632 = 3969 – 1 = 3968
3968 is the missing number.
-
Question 34 of 80
34. Question
Find the missing number in series marked by “?”
3, 7, 16, 35, “?”, 153
(A) 72
(B) 64
(C) 134
(D) 74
-
-
-
-
Correct
Incorrect
The pattern is,
3*2 + 1 = 7
7*2 + 2 = 16
16*2 + 3 = 35
35*2 + 4 = 74
74*2 + 5 = 153
74 is the missing number
Unattempted
The pattern is,
3*2 + 1 = 7
7*2 + 2 = 16
16*2 + 3 = 35
35*2 + 4 = 74
74*2 + 5 = 153
74 is the missing number
-
Question 35 of 80
35. Question
Consider the following students in an examination :
A scored more than B
C scored as much as D
E scored less than F
B scored more than C
F secored less than D
Who got lowest score ?
(A) E
(B) C
(C) D
(D) F
-
-
-
-
Correct
Incorrect
A > B, C = D, E < F, B > C, F < D
(1) (3) (5) (2) (4)
A > B > C = D > F > E.
So, ‘E’ got lowest score.
Unattempted
A > B, C = D, E < F, B > C, F < D
(1) (3) (5) (2) (4)
A > B > C = D > F > E.
So, ‘E’ got lowest score.
-
Question 36 of 80
36. Question
Directions : Examine the information given below and answer the questions that follow:
Kush, Ganesh and Hari are intelligent.
Kush, Ram and Jivan are hardworking.
Ram, Hari and Jivan are honest.
Kush, Ganesh and Jivan are ambitious.
Which of the following persons is neither hardworking nor ambitious?
(A) Kush
(B) Ganesh
(C) Hari
(D) Ram
-
-
-
-
Correct
Incorrect
Hari is neither hardworking nor ambitious.
Unattempted
Hari is neither hardworking nor ambitious.
-
Question 37 of 80
37. Question
A cube of 3 cm side is divided into smaller cubes of side 1 cm. How many times will it be cut to obtain smaller cubes?
(A) 8
(B) 4
(C) 6
(D) 9
-
-
-
-
Correct
Incorrect
Unattempted
-
Question 38 of 80
38. Question
Babu said to Sanjay, “That boy playing with football is younger of the two brothers of the daughter of my father's wife.” How is the boy playing football related to Babu?
(A) Uncle
(B) Elder brother
(C) Younger brother
(D) Cousin
-
-
-
-
Correct
Incorrect
Younger of the two brothers of the daughter of my father’s wife (as my father’s wife = my mother)
So, younger of the two brothers of the daughter of my mother (as daughter of my mother = my sister). So, younger of the two brothers of my sister means ‘my younger brother’. Hence, that boy is the younger brother of Babu.
Unattempted
Younger of the two brothers of the daughter of my father’s wife (as my father’s wife = my mother)
So, younger of the two brothers of the daughter of my mother (as daughter of my mother = my sister). So, younger of the two brothers of my sister means ‘my younger brother’. Hence, that boy is the younger brother of Babu.
-
Question 39 of 80
39. Question
The question below consists of a question and two statements numbered I and II given below it. You have to decide whether the data provided in the statements are sufficient to answer the question.
How far did Mohan walk from the starting point?
1. Mohan walked 20 metres towards West, took a right turn and walked 30 metres, again took a right turn and walked 20 metres.
2. Mohan walked 20 metres towards South, took a left turn and walked 30 metres, again took a left turn and walked 20 metres.
Read both the statements and give answer :
(A) The data in Statement I alone are sufficient to answer the question, while the data in Statement II alone are not sufficient to answer the question.
(B) The data in Statement II alone are sufficient to answer the question, while the data in Statement I alone are not sufficient to answer the question.
(C) The data either in Statement I alone or in Statement II alone are sufficient to answer the question.
(D) The data given in both the Statements I and II together are not sufficient to answer the question.
-
-
-
-
Correct
Incorrect
Unattempted
-
Question 40 of 80
40. Question
Directions : Study the following information carefully and answer the questions given below:
Seven persons A, B, C, D, E, F and G are sitting in a circle and are facing the circle.
G is third to the left of A.
F is third to the left of B.
D is not the neighbour of G.
C is the neighbour of A.
B is to the immediate left of A.
What is the correct sitting position of E?
(A) To the immediate right of B
(B) Between G and F
(C) Third to the right of C
(D) To the immediate right of G
-
-
-
-
Correct
Incorrect
Unattempted
-
Question 41 of 80
41. Question
Find the number of permutations of the letters of the word ALLAHABAD.
(A) 7460
(B) 7560
(C) 7566
(D) 7569
-
-
-
-
Correct
Incorrect
Given word – ALLAHABAD
Here, there are 9 objects (letters) of which there are 4As, 2 Ls and rest are all different.
Therefore, the required number of arrangements = 9!/(4! 2!)
= (1 × 2 × 3 × 4 × 5 × 6 × 7 × 8 × 9)/ (1 × 2 × 3 × 4 × 1 × 2)
= (5 × 6 × 7 × 8 × 9)/2 = 7560
Unattempted
Given word – ALLAHABAD
Here, there are 9 objects (letters) of which there are 4As, 2 Ls and rest are all different.
Therefore, the required number of arrangements = 9!/(4! 2!)
= (1 × 2 × 3 × 4 × 5 × 6 × 7 × 8 × 9)/ (1 × 2 × 3 × 4 × 1 × 2)
= (5 × 6 × 7 × 8 × 9)/2 = 7560
-
Question 42 of 80
42. Question
Find the next term in the series.
294, 448, 648, 900, 1210, ?
(A) 1520
(B) 1480
(C) 1495
(D) 1584
-
-
-
-
Correct
Incorrect
The series is as follows:
72 x 6 = 294
82 x 7 = 448
92 x 8 = 648
102 x 9 = 900
112 x 10 = 1210
122 x 11 = 1584
Unattempted
The series is as follows:
72 x 6 = 294
82 x 7 = 448
92 x 8 = 648
102 x 9 = 900
112 x 10 = 1210
122 x 11 = 1584
-
Question 43 of 80
43. Question
If today is Friday, then what will be the day after 363 days?
(A) Sunday
(B) Saturday
(C) Thursday
(D) None of these
-
-
-
-
Correct
Incorrect
Today is Friday.
Each day gets repeated after 7 days.
So, on dividing 363 by 7, we get remainder = 6
1 – Saturday
2 – Sunday
3 – Monday
4 – Tuesday
5 – Wednesday
6 – Thursday
Hence, option C is the correct answer.
Unattempted
Today is Friday.
Each day gets repeated after 7 days.
So, on dividing 363 by 7, we get remainder = 6
1 – Saturday
2 – Sunday
3 – Monday
4 – Tuesday
5 – Wednesday
6 – Thursday
Hence, option C is the correct answer.
-
Question 44 of 80
44. Question
If the day after the 11th of March is Thursday, then what will be the last day of the month?
(A) Thursday
(B) Wednesday
(C) Friday
(D) Tuesday
-
-
-
-
Correct
Incorrect
The day after the 11th of March is Thursday means on 12th of March is Thursday.
March has 31 days.
19th march is Thursday then 26th March is Thursday.
26 + 5 = 31st March.
Thursday + 5 = Tuesday.
Hence, option D is the correct answer.
Unattempted
The day after the 11th of March is Thursday means on 12th of March is Thursday.
March has 31 days.
19th march is Thursday then 26th March is Thursday.
26 + 5 = 31st March.
Thursday + 5 = Tuesday.
Hence, option D is the correct answer.
-
Question 45 of 80
45. Question
If the day before yesterday was Monday, then what day will it be the day after tomorrow?
(A) Friday
(B) Thursday
(C) Wednesday
(D) Tuesday
-
-
-
-
Correct
Incorrect
The day before yesterday is Monday,
Yesterday is Tuesday,
Today is Wednesday
Tomorrow is Thursday
The day after tomorrow is Friday.
Hence, option A is the correct answer.
Unattempted
The day before yesterday is Monday,
Yesterday is Tuesday,
Today is Wednesday
Tomorrow is Thursday
The day after tomorrow is Friday.
Hence, option A is the correct answer.
-
Question 46 of 80
46. Question
Republic Day of India was celebrated on Thursday in 2017. On which day it was celebrated in 2021?
(A) Tuesday
(B) Wednesday
(C) Thursday
(D) Friday
-
-
-
-
Correct
Incorrect
We know that in 365 days we have 1 odd day.
26 January 2017 = Thursday
26 January 2018 = Friday
26 January 2019 = Saturday
26 January 2020 = Sunday.
2020 is a leap year so February has 29 days.
26 January 2021 = Sunday + 2 = Tuesday (+2 because of leap year, In 366 days we have 2 odd days)
Unattempted
We know that in 365 days we have 1 odd day.
26 January 2017 = Thursday
26 January 2018 = Friday
26 January 2019 = Saturday
26 January 2020 = Sunday.
2020 is a leap year so February has 29 days.
26 January 2021 = Sunday + 2 = Tuesday (+2 because of leap year, In 366 days we have 2 odd days)
-
Question 47 of 80
47. Question
Answer the question based on the figure given below in which Rectangle represents Males, Circle represents the urbans, Square represents the educated and Triangle represents the civil servants.
The number indicating the educated civil servants who are males but not urban is
(A) 7
(B) 10
(C) 8
(D) 4
-
-
-
-
Correct
Incorrect
Educated civil servant Males: 10 and 8
Non urban: 10
Unattempted
Educated civil servant Males: 10 and 8
Non urban: 10
-
Question 48 of 80
48. Question
Excluding stoppages, the speed of a bus is 54 kmph and including stoppages, it is 45 kmph. For how many minutes does the bus stop per hour?
(A) 5 min
(B) 10 min
(C) 4 min
(D) 9 min
-
-
-
-
Correct
Incorrect
Speed without stoppages = 54km/hr
Speed with stoppages = 45km/hr
Stoppages per hour =
Unattempted
Speed without stoppages = 54km/hr
Speed with stoppages = 45km/hr
Stoppages per hour =
-
Question 49 of 80
49. Question
DIRECTION : READ THE FOLLOWING PASSAGE AND ANSWER THE QUESTION THAT FOLLOW. YOUR ANSWER TO THIS QUESTION SHOULD BE BASED ON THE PASSAGE ONLY.
The most important reason for this state of affairs, perhaps, is that India was the only country in the world to truly recognise the achievements of the Soviet Union-rather than merely focus on the debilitating faults that Communism brought to its people. The people of India realised that the achievement of one hundred percent literacy in a country much, much larger than its own and with similarly complicated ethnic and religious groupings, the rapid industrialization of a nation that was a primarily agrarian society when the Bolshevik revolution took place in 1917, the attendant revolutionary steps in science and technology, the accessibility of health care (primeval according to Western standards, perhaps, but not according to Indian ones) to the general population, and despite prohibition of the government of the time the vast outpourings in literature, music, art, etc. are momentous and remarkable feats in any country. In contrast, all that the West focused on were the massive human rights violations by the Soviet State on its people, the deliberate uprooting and mass migrations of ethnic peoples from one part of the country to another in the name of industrialization, the end of religion. In short, all the tools of information were employed to condemn the ideology of Communism, so much at variance with capitalist thinking. The difference with the Indian perception, I think here is, that while the Indians reacted as negatively to what the Soviet governments did to its people in the name of good governance (witness the imprisonment of Boris Pasternak and the formation of an international committee to put pressure for his release with Jawaharlal Nehru at its head), they took the pain not to condemn the people of that broad country in black and white terms; they understood that mingled in the shades of grey were grains of uniqueness (The Russians have never failed that characteristic in themselves; they have twice experimented with completely different ideologies, Communism and Capitalism both in the space of a century).
Which of the following statements according to the passage is correct?
(A) India took heed of the weak faults of Russian policies and systems.
(B) The process of industrialisation had already started when the Russian Revolution took place in 1917.
(C) India seriously commended the achievements of Russia, i.e., hundred percent literacy and rapid industrialisation.
(D) Literature, art and music received a setback during the communist regime in Russia.
-
-
-
-
Correct
Incorrect
Refer to these lines, “The people of India realized that the achievement of one hundred percent literacy in a country much, much larger than its own and with similarly complicated ethnic and religious groupings, the rapid industrialization of a nation that was a primarily agrarian society when the Bolshevik revolution took place in 1917, the attendant revolutionary steps in science and technology, the accessibility of health care (primeval according to Western standards, perhaps, but not according to Indian ones) to the general population, and despite the prohibition of the government of the time the vast outpourings in literature, music, art, etc. are momentous and remarkable feats in any country“
India appreciated the achievement of Russia in the field of literacy and rapid industrialization
Unattempted
Refer to these lines, “The people of India realized that the achievement of one hundred percent literacy in a country much, much larger than its own and with similarly complicated ethnic and religious groupings, the rapid industrialization of a nation that was a primarily agrarian society when the Bolshevik revolution took place in 1917, the attendant revolutionary steps in science and technology, the accessibility of health care (primeval according to Western standards, perhaps, but not according to Indian ones) to the general population, and despite the prohibition of the government of the time the vast outpourings in literature, music, art, etc. are momentous and remarkable feats in any country“
India appreciated the achievement of Russia in the field of literacy and rapid industrialization
-
Question 50 of 80
50. Question
Answer the question based on the figure given below in which Rectangle represents Males, Circle represents the urbans, Square represents the educated and Triangle represents the civil servants.
The number indicating the educated males who are urban civil servants is
a.5
(B) 4
(C) 11
(D) 8
-
-
-
-
Correct
Incorrect
Educated male urban civil servants: 8
Unattempted
Educated male urban civil servants: 8
-
Question 51 of 80
51. Question
DIRECTION : READ THE FOLLOWING PASSAGE AND ANSWER THE QUESTION THAT FOLLOW. YOUR ANSWER TO THIS QUESTION SHOULD BE BASED ON THE PASSAGE ONLY.
The most important reason for this state of affairs, perhaps, is that India was the only country in the world to truly recognise the achievements of the Soviet Union-rather than merely focus on the debilitating faults that Communism brought to its people. The people of India realised that the achievement of one hundred percent literacy in a country much, much larger than its own and with similarly complicated ethnic and religious groupings, the rapid industrialization of a nation that was a primarily agrarian society when the Bolshevik revolution took place in 1917, the attendant revolutionary steps in science and technology, the accessibility of health care (primeval according to Western standards, perhaps, but not according to Indian ones) to the general population, and despite prohibition of the government of the time the vast outpourings in literature, music, art, etc. are momentous and remarkable feats in any country. In contrast, all that the West focused on were the massive human rights violations by the Soviet State on its people, the deliberate uprooting and mass migrations of ethnic peoples from one part of the country to another in the name of industrialization, the end of religion. In short, all the tools of information were employed to condemn the ideology of Communism, so much at variance with capitalist thinking. The difference with the Indian perception, I think here is, that while the Indians reacted as negatively to what the Soviet governments did to its people in the name of good governance (witness the imprisonment of Boris Pasternak and the formation of an international committee to put pressure for his release with Jawaharlal Nehru at its head), they took the pain not to condemn the people of that broad country in black and white terms; they understood that mingled in the shades of grey were grains of uniqueness (The Russians have never failed that characteristic in themselves; they have twice experimented with completely different ideologies, Communism and Capitalism both in the space of a century).
The Indian perception of the Soviet Union was always
(A) Neutral
(B) Applauding
(C) Counter-reactionary
(D) Critical
-
-
-
-
Correct
Incorrect
As indicated in the passage India applauded the achievements of Russia.
Unattempted
As indicated in the passage India applauded the achievements of Russia.
-
Question 52 of 80
52. Question
DIRECTION : READ THE FOLLOWING PASSAGE AND ANSWER THE QUESTION THAT FOLLOW. YOUR ANSWER TO THIS QUESTION SHOULD BE BASED ON THE PASSAGE ONLY.
The most important reason for this state of affairs, perhaps, is that India was the only country in the world to truly recognise the achievements of the Soviet Union-rather than merely focus on the debilitating faults that Communism brought to its people. The people of India realised that the achievement of one hundred percent literacy in a country much, much larger than its own and with similarly complicated ethnic and religious groupings, the rapid industrialization of a nation that was a primarily agrarian society when the Bolshevik revolution took place in 1917, the attendant revolutionary steps in science and technology, the accessibility of health care (primeval according to Western standards, perhaps, but not according to Indian ones) to the general population, and despite prohibition of the government of the time the vast outpourings in literature, music, art, etc. are momentous and remarkable feats in any country. In contrast, all that the West focused on were the massive human rights violations by the Soviet State on its people, the deliberate uprooting and mass migrations of ethnic peoples from one part of the country to another in the name of industrialization, the end of religion. In short, all the tools of information were employed to condemn the ideology of Communism, so much at variance with capitalist thinking. The difference with the Indian perception, I think here is, that while the Indians reacted as negatively to what the Soviet governments did to its people in the name of good governance (witness the imprisonment of Boris Pasternak and the formation of an international committee to put pressure for his release with Jawaharlal Nehru at its head), they took the pain not to condemn the people of that broad country in black and white terms; they understood that mingled in the shades of grey were grains of uniqueness (The Russians have never failed that characteristic in themselves; they have twice experimented with completely different ideologies, Communism and Capitalism both in the space of a century).
The West did not focus on
(A) Massive human rights violation of the Soviet state on its people.
(B) Rapid growth of nuclear weapons in Russia.
(C) Deliberate uprooting and mass migration of ethnic people in the name of industrialisation.
(D) Both (A) and (C).
-
-
-
-
Correct
Incorrect
Refer to these lines, “In contrast, all that the West focused on were the massive human rights violations by the Soviet State on its people, the deliberate uprooting and mass migrations of ethnic peoples from one part of the country to another in the name of industrialization, the end of religion.”
All that west focused on is mentioned in options A and C but not B.
Unattempted
Refer to these lines, “In contrast, all that the West focused on were the massive human rights violations by the Soviet State on its people, the deliberate uprooting and mass migrations of ethnic peoples from one part of the country to another in the name of industrialization, the end of religion.”
All that west focused on is mentioned in options A and C but not B.
-
Question 53 of 80
53. Question
A man wants to reach a window which is 40 feet above the ground. The distance from the foot of the ladder to the wall is 9 feet. How long should the ladder be?
(A) 81 feet
(B) 40 feet
(C) 41 feet
(D) 54 feet
-
-
-
-
Correct
Incorrect
By Pythagoras theorem,
Length of ladder =
Unattempted
By Pythagoras theorem,
Length of ladder =
-
Question 54 of 80
54. Question
If A is to the south of B and C is to the east of B, what is the direction of A with respect to C?
(A) North–east
(B) South–east
(C) North–west
(D) South–west
-
-
-
-
Correct
Incorrect
From the figure given below, A is south–west of C.
Unattempted
From the figure given below, A is south–west of C.
-
Question 55 of 80
55. Question
DIRECTION : READ THE FOLLOWING PASSAGE AND ANSWER THE QUESTION THAT FOLLOW. YOUR ANSWER TO THIS QUESTION SHOULD BE BASED ON THE PASSAGE ONLY.
For the Maya of the Classic period, who lived in Southern Mexico and Central America between 250 and 900 CE, the category of “persons” was not coincident with human beings, as it is for us. That is, human beings were persons – but other, nonhuman entities could be persons, too. In order to explore the slippage of categories between “humans” and “persons”, I examined a very specific category of ancient Maya images, found painted in scenes on ceramic vessels. I sought out instances in which faces (some combination of eyes, nose, and mouth) are shown on inanimate objects. Consider my iPhone, which needs to be fed with electricity every night, swaddled in a protective bumper, and enjoys communicating with other fellow-phone-beings. Does it have personhood (if at all) because itis connected to me, drawing this resource from me as an owner or source? For the Maya (who did have plenty of other communicating objects, if not smartphones), the answer was no. Nonhuman persons were not tethered to specific humans, and they did not derive their personhood from a connection with a human. It's a profoundly democratising way of understanding the world. Humans are not more important persons – we are just one of many kinds of persons who inhabit this world.
The Maya saw personhood as 'activated' by experiencing certain bodily needs and through participation in certain social activities. For example, among the faced objects that I examined, persons are marked by personal requirements (such as hunger, tiredness, physical closeness), and by community obligations (communication, interaction, ritual observance). In the images I examined, we see, for instance, faced objects being cradled in humans' arms; we also see them speaking to humans. These core elements of personhood are both turned inward, what the body or self of a person requires, and outward, what a community expects of the persons who are a part of it, underlining the reciprocal nature of community membership.
Personhood was a nonbinary proposition for the Maya. Entities were able to be persons while also being something else. The faced objects I looked at indicate that they continue to be functional, doing what objects do (a stone implement continues to chop, an incense burner continues to do its smoky work). Furthermore, the Maya visually depicted many objects in ways that indicated the material category to which they belonged – drawings of the stone implement show that a person-tool is still made of stone. One additional complexity: the incense burner (which would have been made of clay, and decorated with spiky appliques representing the sacred ceiba tree found in this region) is categorised as a person – but also as a tree. With these Maya examples, we are challenged to discard the person/nonperson binary that constitutes our basic ontological outlook. The porousness of boundaries that we have seen in the Maya world points towards the possibility of living with a certain uncategorisability of the world.
Which one of the following, if true about the Classic Maya, would invalidate the purpose of the iPhone example in the passage?
(A) The clay incense burner with spiky appliques was categorised only as a person and not as a tree by the Classic Maya.
(B) Classic Maya songs represent both humans and non-living objects as characters, talking and interacting with each other.
(C) The personhood of the incense burner and the stone chopper was a function of their usefulness to humans.
(D) Unlike modern societies equipped with mobile phones, the Classic Maya did not have any communicating objects.
-
-
-
-
Correct
Incorrect
To answer this question we have to understand the iPhone example. The author says that to us iPhone has a personhood because it is connected to or useful to me, but this was not the case with Maya people. To them nonhuman persons were not tethered to specific humans. To invalidate this example, we have to pick a choice that goes against this. Option C precisely does that. It makes the personhood of the incense burner and the stone chopper a function of their usefulness to humans, something that the author wants to deny through the example of iPhone. Thus if C is true than the purpose of the iPhone example is invalidated. All the other three choices don't invalidate the iPhone example in any way.
Unattempted
To answer this question we have to understand the iPhone example. The author says that to us iPhone has a personhood because it is connected to or useful to me, but this was not the case with Maya people. To them nonhuman persons were not tethered to specific humans. To invalidate this example, we have to pick a choice that goes against this. Option C precisely does that. It makes the personhood of the incense burner and the stone chopper a function of their usefulness to humans, something that the author wants to deny through the example of iPhone. Thus if C is true than the purpose of the iPhone example is invalidated. All the other three choices don't invalidate the iPhone example in any way.
-
Question 56 of 80
56. Question
Find the next term in the series.
28, 44, -81, -45, -388, ?
(A) -342
(B) -314
(C) -373
(D) -324
-
-
-
-
Correct
Incorrect
The series is as follows:
28 + 42 = 44
44 – 53 = -81
-81 + 62 = -45
-45 – 73 = -388
-388 + 82 = -324
Unattempted
The series is as follows:
28 + 42 = 44
44 – 53 = -81
-81 + 62 = -45
-45 – 73 = -388
-388 + 82 = -324
-
Question 57 of 80
57. Question
P is facing the north direction. He then turns right and walks 20 m. He then turns to his left and walks 20 m. Next, he moves 20 m to his right. He then turns to his right again and walks 40 m. Finally, he turns to his right and moves 30 m. In which direction is he now with respect to his starting point?
(A) South–west
(B) South
(C) North–west
(D) South–east
-
-
-
-
Correct
Incorrect
Let us assume that P starts from point A.
He turns right and walks 20 m towards east up to point B, turns left and moves 20 m up to point C, turns right and moves 20 m up to point D.
At D where he is facing east, he takes a right turn and turns towards south and walks 40 m up to E.
Next, he turns right again and walks 30 m up to F, his final position. F is south–east of A.
Therefore, P is south–east of his starting point.
Unattempted
Let us assume that P starts from point A.
He turns right and walks 20 m towards east up to point B, turns left and moves 20 m up to point C, turns right and moves 20 m up to point D.
At D where he is facing east, he takes a right turn and turns towards south and walks 40 m up to E.
Next, he turns right again and walks 30 m up to F, his final position. F is south–east of A.
Therefore, P is south–east of his starting point.
-
Question 58 of 80
58. Question
How many numbers are there between 99 and 1000, having at least one of their digits 7?
(A) 251
(B) 252
(C) 253
(D) 254
-
-
-
-
Correct
Incorrect
Numbers between 99 and 1000 are all three-digit numbers.
Total number of 3 digit numbers having at least one of their digits as 7
= (Total numbers of three-digit numbers) – (Total number of 3 digit numbers in which 7 does not appear at all)
= (9 × 10 × 10) – (8 × 9 × 9)
= 900 – 648 = 252
Unattempted
Numbers between 99 and 1000 are all three-digit numbers.
Total number of 3 digit numbers having at least one of their digits as 7
= (Total numbers of three-digit numbers) – (Total number of 3 digit numbers in which 7 does not appear at all)
= (9 × 10 × 10) – (8 × 9 × 9)
= 900 – 648 = 252
-
Question 59 of 80
59. Question
How many 5-digit telephone numbers can be constructed using the digits 0 to 9, if each number starts with 67 and no digit appears more than once?
(A) 332
(B) 335
(C) 336
(D) 339
-
-
-
-
Correct
Incorrect
Let ABCDE be a five-digit number.
Given that the first two digits of each number are 6 and 7.
Therefore, the number is 67CDE.
As repetition is not allowed and 6 and 7 are already taken, the digits available for place C are 0, 1, 2, 3, 4, 5, 8, 9, i.e. eight possible digits.
Suppose one of them is taken at C, now the digits possible at place D is 7.
Similarly, at E, the possible digit is 6.
Therefore, the total five-digit numbers with given conditions = 8 × 7 × 6 = 336.
Unattempted
Let ABCDE be a five-digit number.
Given that the first two digits of each number are 6 and 7.
Therefore, the number is 67CDE.
As repetition is not allowed and 6 and 7 are already taken, the digits available for place C are 0, 1, 2, 3, 4, 5, 8, 9, i.e. eight possible digits.
Suppose one of them is taken at C, now the digits possible at place D is 7.
Similarly, at E, the possible digit is 6.
Therefore, the total five-digit numbers with given conditions = 8 × 7 × 6 = 336.
-
Question 60 of 80
60. Question
Find the next term in the series.
632, 582, 534, 488, 444, ?
(A) 417
(B) 402
(C) 410
(D) 408
-
-
-
-
Correct
Incorrect
The pattern is as follows:
252 + 7 = 632
242 + 6 = 582
232 + 5 = 534
222 + 4 = 488
212 + 3 = 444
202 + 2 = 402
Unattempted
The pattern is as follows:
252 + 7 = 632
242 + 6 = 582
232 + 5 = 534
222 + 4 = 488
212 + 3 = 444
202 + 2 = 402
-
Question 61 of 80
61. Question
If a * b = 2 (a + b), then 5 * 2 is equal to :
(A) 3
(B) 10
(C) 14
(D) 20
-
-
-
-
Correct
Incorrect
a * b = 2 (a + b)
So, 5 * 2 = 2 (5 + 2)
= 2 × 7 = 14
Unattempted
a * b = 2 (a + b)
So, 5 * 2 = 2 (5 + 2)
= 2 × 7 = 14
-
Question 62 of 80
62. Question
In how many of the distinct permutations of the letters in MISSISSIPPI do the four Is not come together?
(A) 33810
(B) 33812
(C) 33815
(D) 33862
-
-
-
-
Correct
Incorrect
Given word – MISSISSIPPI
M – 1
I – 4
S – 4
P – 2
Number of permutations = 11!/(4! 4! 2!)
= (11 × 10 × 9 × 8 × 7 × 6 × 5 × 4!)/ (4! × 24 × 2) = 34650
We take that 4 I’s come together, and they are treated as 1 letter,
∴ Total number of letters=11 – 4 + 1 = 8
⇒ Number of permutations = 8!/(4! 2!)
= (8 × 7 × 6 × 5 × 4!)/ (4! × 2) = 840
Therefore, the total number of permutations where four Is don’t come together
= 34650 – 840 = 33810
Unattempted
Given word – MISSISSIPPI
M – 1
I – 4
S – 4
P – 2
Number of permutations = 11!/(4! 4! 2!)
= (11 × 10 × 9 × 8 × 7 × 6 × 5 × 4!)/ (4! × 24 × 2) = 34650
We take that 4 I’s come together, and they are treated as 1 letter,
∴ Total number of letters=11 – 4 + 1 = 8
⇒ Number of permutations = 8!/(4! 2!)
= (8 × 7 × 6 × 5 × 4!)/ (4! × 2) = 840
Therefore, the total number of permutations where four Is don’t come together
= 34650 – 840 = 33810
-
Question 63 of 80
63. Question
DIRECTION : READ THE FOLLOWING PASSAGE AND ANSWER THE QUESTIONS THAT FOLLOW. YOUR ANSWERS TO THESE QUESTIONS SHOULD BE BASED ON THE PASSAGE ONLY.
If you’re a fitness walker, there is no need for a commute to a health club. Your neighborhood can be your health club. You don’t need a lot of fancy equipment to get a good workout either. All you need is a well-designed pair of athletic shoes.
This paragraph best supports the statement that :
(A) fitness walking is a better form of exercise than weight lifting.
(B) a membership in a health club is a poor investment.
(C) walking outdoors provides a better workout than walking indoors.
(D) fitness walking is a convenient and valuable form of exercise.
-
-
-
-
Correct
Incorrect
The author stresses the convenience of fitness walking, by stating that it does not require a commute to a health club. The paragraph also implies that fitness walking will result in a good workout. Choice (A) is incorrect because no comparison to weight lifting is made. Choice (B) may seem like a logical answer, but the paragraph only refers to people who are fitness walkers, so for others, a health club might be a good investment. Choice (C) is not in the passage.
Unattempted
The author stresses the convenience of fitness walking, by stating that it does not require a commute to a health club. The paragraph also implies that fitness walking will result in a good workout. Choice (A) is incorrect because no comparison to weight lifting is made. Choice (B) may seem like a logical answer, but the paragraph only refers to people who are fitness walkers, so for others, a health club might be a good investment. Choice (C) is not in the passage.
-
Question 64 of 80
64. Question
In a small village, there are 87 families, of which 52 families have at most 2 children. In a rural development programme, 20 families are to be chosen for assistance, of which at least 18 families must have at most 2 children. In how many ways can the choice be made?
(A) 52C18 × 35C2 + 52C19 × 35C1 + 52C20
(B) 52C18 × 35C2 + 52C29 × 35C1 + 52C20
(C) 52C18 × 35C2 + 52C19 × 35C1 + 52C26
(D) 52C15 × 35C2 + 52C19 × 35C1 + 52C20
-
-
-
-
Correct
Incorrect
Given,
Total number of families = 87
Number of families with at most 2 children = 52
Remaining families = 87 – 52 = 35
Also, for the rural development programme, 20 families are to be chosen for assistance, of which at least 18 families must have at most 2 children.
Thus, the following are the number of possible choices:
52C18 × 35C2 (18 families having at most 2 children and 2 selected from other types of families)
52C19 × 35C1 (19 families having at most 2 children and 1 selected from other types of families)
52C20 (All selected 20 families having at most 2 children)
Hence, the total number of possible choices = 52C18 × 35C2 + 52C19 × 35C1 + 52C20
Unattempted
Given,
Total number of families = 87
Number of families with at most 2 children = 52
Remaining families = 87 – 52 = 35
Also, for the rural development programme, 20 families are to be chosen for assistance, of which at least 18 families must have at most 2 children.
Thus, the following are the number of possible choices:
52C18 × 35C2 (18 families having at most 2 children and 2 selected from other types of families)
52C19 × 35C1 (19 families having at most 2 children and 1 selected from other types of families)
52C20 (All selected 20 families having at most 2 children)
Hence, the total number of possible choices = 52C18 × 35C2 + 52C19 × 35C1 + 52C20
-
Question 65 of 80
65. Question
Find the next term in the series.
10, 18, 28, 40, 54, ?
(A) 56
(B) 68
(C) 70
(D) 71
-
-
-
-
Correct
Incorrect
The pattern is as follows:
1 x 5 + 5 = 10
2 x 6 + 6 = 18
3 x 7 + 7 = 28
4 x 8 + 8 = 40
5 x 9 + 9 = 54
6 x 10 + 10 = 70
Unattempted
The pattern is as follows:
1 x 5 + 5 = 10
2 x 6 + 6 = 18
3 x 7 + 7 = 28
4 x 8 + 8 = 40
5 x 9 + 9 = 54
6 x 10 + 10 = 70
-
Question 66 of 80
66. Question
DIRECTION : READ THE FOLLOWING PASSAGE AND ANSWER THE QUESTION THAT FOLLOW. YOUR ANSWER TO THIS QUESTION SHOULD BE BASED ON THE PASSAGE ONLY.
Today we can hardly conceive of ourselves without an unconscious. Yet between 1700 and1900, this notion developed as a genuinely original thought. The “unconscious” burst the shell of conventional language, coined as it had been to embody the fleeting ideas and the shifting conceptions of several generations until, finally, it became fixed and defined in specialized terms within the realm of medical psychology and Freudian psychoanalysis.
The vocabulary concerning the soul and the mind increased enormously in the course of the nineteenth century. The enrichments of literary and intellectual language led to an altered understanding of the meanings that underlie time-honored expressions and traditional catchwords. At the same time, once coined, powerful new ideas attracted to themselves a whole host of seemingly unrelated issues, practices, and experiences, creating a peculiar network of preoccupations that as a group had not existed before. The drawn-out attempt to approach and define the unconscious brought together the spiritualist and the psychical researcher of borderline phenomena (such as apparitions, spectral illusions, haunted houses, mediums, trance, automatic writing); the psychiatrist or alienist probing the nature of mental disease, of abnormal ideation, hallucination, delirium, melancholia, mania; the surgeon performing operations with the aid of hypnotism; the magnetizer claiming to correct the disequilibrium in the universal flow of magnetic fluids but who soon came to be regarded as a clever manipulator of the imagination; the physiologist and the physician who puzzled oversleep, dreams, sleepwalking, anesthesia, the influence of the mind on the body in health and disease; the neurologist concerned with the functions of the brain and the physiological basis of mental life; the philosopher interested in the will, the emotions, consciousness, knowledge, imagination and the creative genius; and, last but not least, the psychologist.
Significantly, most if not all of these practices (for example, hypnotism in surgery or psychological magnetism) originated in the waning years of the eighteenth century and during the early decades of the nineteenth century, as did some of the disciplines (such as psychology and psychical research). The majority of topics too were either new or assumed hitherto unknown colors. Thus, before 1790, few if any spoke, in medical terms, of the affinity between creative genius and the hallucinations of the insane .
Striving vaguely and independently to give expression to a latent conception, various lines of thought can be brought together by some novel term. The new concept then serves as a kind of resting place or stocktaking in the development of ideas, giving satisfaction and a stimulus for further discussion or speculation. Thus, the massive introduction of the term unconscious by Hartmann in 1869 appeared to focalize many stray thoughts, affording a temporary feeling that a crucial step had been taken forward, a comprehensive knowledge gained, a knowledge that required only further elaboration, explication, and unfolding in order to bring in a bounty of higher understanding. Ultimately, Hartmann's attempt at defining the unconscious proved fruitless because he extended its reach into every realm of organic and inorganic, spiritual, intellectual, and instinctive existence, severely diluting the precision and compromising the impact of the concept.
Which one of the following sets of words is closest to mapping the main arguments of the passage?
(A) Language; Unconscious; Psychoanalysis.
(B) Unconscious; Latent conception; Dreams.
(C) Literary language; Unconscious; Insanity.
(D) Imagination; Magnetism; Psychiatry.
-
-
-
-
Correct
Incorrect
The question asks us to pick the choice that is closest to mapping the main arguments of the passage. We have to ensure that the three ideas have significant presence in the passage, precisely in the order in which they have come in the passage. Here we can mark the answer by looking at the first word of each choice. The first paragraph and the first few lines of second paragraph talk about how the word unconscious burst the shell of conventional language. This makes choice A our best bet. Literary language, in choice C, is different from language, while the latter means language in general, the former means “highly stylized language”. Thus literary language is not the author's idea; it is how the word unconscious burst the shell of conventional language, not literary language.
Latent conception in option B, and magnetism in option D make those choices irrelevant.
Unattempted
The question asks us to pick the choice that is closest to mapping the main arguments of the passage. We have to ensure that the three ideas have significant presence in the passage, precisely in the order in which they have come in the passage. Here we can mark the answer by looking at the first word of each choice. The first paragraph and the first few lines of second paragraph talk about how the word unconscious burst the shell of conventional language. This makes choice A our best bet. Literary language, in choice C, is different from language, while the latter means language in general, the former means “highly stylized language”. Thus literary language is not the author's idea; it is how the word unconscious burst the shell of conventional language, not literary language.
Latent conception in option B, and magnetism in option D make those choices irrelevant.
-
Question 67 of 80
67. Question
Answer the question based on the figure given below in which Rectangle represents Males, Circle represents the urbans, Square represents the educated and Triangle represents the civil servants.
The number indicating the uneducated females who are urban civil servants is
(A) 11
(B) 5
(C) 9
(D) 7
-
-
-
-
Correct
Incorrect
Uneducated female urban civil servants: 11
Unattempted
Uneducated female urban civil servants: 11
-
Question 68 of 80
68. Question
DIRECTION : READ THE FOLLOWING PASSAGE AND ANSWER THE QUESTIONS THAT FOLLOW. YOUR ANSWERS TO THESE QUESTIONS SHOULD BE BASED ON THE PASSAGE ONLY.
Once people wore garlic around their necks to ward off disease. Today, most Americans would scoff at the idea of wearing a necklace of garlic cloves to enhance their well-being. However, you might find a number of Americans willing to ingest capsules of pulverized garlic or other herbal supplements in the name of health. Complementary and alternative medicine, which includes a range of practices outside of conventional medicine such as herbs, homeopathy, massage therapy, yoga, and acupuncture, hold increasing appeal for Americans. In fact, according to one estimate, 42% of Americans have used alternative therapies. In all age groups, the use of unconventional healthcare practices has steadily increased in the last 30 years, and the trend is likely to continue, although people born before 1945 are the least likely to turn to these therapies.
Why have so many patients turned to alternative therapies? Many are frustrated by the time constraints of managed care and alienated by conventional medicine’s focus on technology. Others feel that a holistic approach to healthcare better reflects their beliefs and values. Others seek therapies that relieve symptoms associated with chronic disease; symptoms that mainstream medicine cannot treat. Some alternative therapies have even crossed the line into mainstream medicine, as scientific investigation has confirmed their safety and efficacy. For example, physicians may currently prescribe acupuncture for pain management or to control the nausea associated with chemotherapy. Additionally, many U.S. medical schools teach courses in alternative therapies, and many health insurance companies offer some alternative medicine benefits.
What is the main idea of this passage?
(A) Alternative medicine is now a big business in the United States with more Americans seeking it out than ever before.
(B) Today, it is not unusual for mainstream doctors to incorporate alternative therapies into their practice.
(C) Over the last few decades, alternative medicine has become more popular, accepted, and practiced in the United States.
(D) People are tired of conventional medicine’s focus on technology.
-
-
-
-
Correct
Incorrect
Unattempted
-
Question 69 of 80
69. Question
Answer the question based on the figure given below in which Rectangle represents Males, Circle represents the urbans, Square represents the educated and Triangle represents the civil servants.
The number indicating the educated urban males who are not civil servants is
(A) 7
(B) 8
(C) 9
(D) 10
-
-
-
-
Correct
Incorrect
Educated Urban Males: 8 and 9
As 8 represents the civil servants, 9 is the number that we are looking for.
Unattempted
Educated Urban Males: 8 and 9
As 8 represents the civil servants, 9 is the number that we are looking for.
-
Question 70 of 80
70. Question
Find the next term in the series.
809, 881, 945,1125, 1135, ?
(A) 1105
(B) 1150
(C) 1173
(D) 1169
-
-
-
-
Correct
Incorrect
The series adds the product of the digits to the previous number:
809 + 72 = 881
881 + 64 = 945
945 + 180 = 1125
1125 + 10 = 1135
1135 + 15 = 1150
Unattempted
The series adds the product of the digits to the previous number:
809 + 72 = 881
881 + 64 = 945
945 + 180 = 1125
1125 + 10 = 1135
1135 + 15 = 1150
-
Question 71 of 80
71. Question
Out of 130 students appearing in an exam, 62 failed in Science, 52 failed in English, whereas 24 failed in both Science and English. The number of students who passed is
(A) 50
(B) 20
(C) 40
(D) 30
-
-
-
-
Correct
Incorrect
Total students =130
Number of failed students = (62 + 52) – 24 = 114 – 24 = 90
(24 is subtracted to avoid repetition as it is included in both 62 and 52)
Number of students who passed = 130 – 90 = 40
Unattempted
Total students =130
Number of failed students = (62 + 52) – 24 = 114 – 24 = 90
(24 is subtracted to avoid repetition as it is included in both 62 and 52)
Number of students who passed = 130 – 90 = 40
-
Question 72 of 80
72. Question
A person travels in the north direction, then turns right, then again turns right and thereafter turns left. In which direction is the person travelling now?
(A) North
(B) South
(C) East
(D) West
-
-
-
-
Correct
Incorrect
The movement of the person is indicated in the figure given below (from A to B, B to C, C to D and D to E).
The final movement is in the direction indicated by the line segment DE, which is towards the East direction.
Unattempted
The movement of the person is indicated in the figure given below (from A to B, B to C, C to D and D to E).
The final movement is in the direction indicated by the line segment DE, which is towards the East direction.
-
Question 73 of 80
73. Question
A publisher printed 3000 copies of a book for sale, the cost of each book being Rs. 7.00. He distributed 500 copies to different institutions free of cost. He allowed a book free of cost for each 24 books purchased. If the price of each book is fixed at Rs. 14.50 determine the rate of profit or loss of the publisher.
(A) 66% loss
(B) 66% profit
(C) 60% profit
(D) 60% loss
-
-
-
-
Correct
Incorrect
Total cost of the books = Rs. (3000 × 7) = Rs. 21000
500 books are given free of cost.
Selling price for 25 books = 24 × 14.50 = Rs. 348
So, Total selling price = Rs. 34800
Gain = Rs. (34800 – 21000) = Rs. 13800
So, Gain% = 13800/21000 x 100 ≈ 66%
Unattempted
Total cost of the books = Rs. (3000 × 7) = Rs. 21000
500 books are given free of cost.
Selling price for 25 books = 24 × 14.50 = Rs. 348
So, Total selling price = Rs. 34800
Gain = Rs. (34800 – 21000) = Rs. 13800
So, Gain% = 13800/21000 x 100 ≈ 66%
-
Question 74 of 80
74. Question
DIRECTION : READ THE FOLLOWING PASSAGE AND ANSWER THE QUESTION THAT FOLLOW. YOUR ANSWER TO THIS QUESTION SHOULD BE BASED ON THE PASSAGE ONLY.
It's easy to forget that most of the world's languages are still transmitted orally with no widely established written form. While speech communities are increasingly involved in projects to protect their languages – in print, on air and online – orality is fragile and contributes to linguistic vulnerability. But indigenous languages are about much more than unusual words and intriguing grammar: They function as vehicles for the transmission of cultural traditions, environmental understandings and knowledge about medicinal plants, all at risk when elders die and livelihoods are disrupted.
Both push and pull factors lead to the decline of languages. Through war, famine and natural disasters, whole communities can be destroyed, taking their language with them to the grave, such as the indigenous populations of Tasmania who were wiped out by colonists. More commonly, speakers live on but abandon their language in favor of another vernacular, a widespread process that linguists refer to as “language shift” from which few languages are immune. Such trading up and out of a speech form occurs for complex political, cultural and economic reasons – sometimes voluntary for economic and educational reasons, although often amplified by state coercion or neglect. Welsh, long stigmatized and disparaged by the British state, has rebounded with vigor.
Many speakers of endangered, poorly documented languages have embraced new digital media with excitement. Speakers of previously exclusively oral tongues are turning to the web as a virtual space for languages to live on. Internet technology offers powerful ways for oral traditions and cultural practices to survive, even thrive, among increasingly mobile communities. I have watched as videos of traditional wedding ceremonies and songs are recorded on smartphones in London by Nepali migrants, then uploaded to YouTube and watched an hour later by relatives in remote Himalayan villages . . .Globalization is regularly, and often uncritically, pilloried as a major threat to linguistic diversity. But in fact, globalization is as much process as it is ideology, certainly when it comes to language. The real forces behind cultural homogenization are unbending beliefs, exchanged through a globalized delivery system, reinforced by the historical monolingualism prevalent in much of the West.
Monolingualism – the condition of being able to speak only one language – is regularly accompanied by a deep-seated conviction in the value of that language over all others. Across the largest economies that make up the G8, being monolingual is still often the norm, with multilingualism appearing unusual and even somewhat exotic. The monolingual mindset stands in sharp contrast to the lived reality of most the world, which throughout its history has been more multilingual than unilingual. Monolingualism, then, not globalization, should be our primary concern.
Multilingualism can help us live in a more connected and more interdependent world. By widening access to technology, globalization can support indigenous and scholarly communities engaged in documenting and protecting our shared linguistic heritage. For the last 5,000 years, the rise and fall of languages was intimately tied to the plow, sword and book. In our digital age, the keyboard, screen and web will play a decisive role in shaping the future linguistic diversity of our species.
The author lists all of the following as reasons for the decline or disappearance of a language EXCEPT:
(A) governments promoting certain languages over others.
(B) a catastrophic event that entirely eliminates a people and their culture.
(C) people shifting away from their own language to study or work in another language.
(D) the focus on only a few languages as a result of widespread internet use.
-
-
-
-
Correct
Incorrect
This is a very simple question. Options A, B and C have been discussed as reasons behind decline or disappearance of a language. The author talks positively about internet in preserving languages. Thus D is the best choice.
Unattempted
This is a very simple question. Options A, B and C have been discussed as reasons behind decline or disappearance of a language. The author talks positively about internet in preserving languages. Thus D is the best choice.
-
Question 75 of 80
75. Question
Bimalbabu sells two cars each of Rs. 99,000. He makes a profit of 10% on the first car, but incurs a loss of 10% on the second. What will be his percentage of profit or loss on the whole transaction ?
(A) 1% profit
(B) 1% loss
(C) 4% profit
(D) 4% loss
-
-
-
-
Correct
Incorrect
C.P. of car sold at 10% profit = (100/100 + gain%) x S.P.
= 100/110 x 99000 = Rs. 90000
C.P. of car sold at 10% loss = 100/90 x 99000 = Rs. 110000
Total C.P. = Rs. (90000 + 110000)
= Rs. 200000
Total S.P. = Rs. 2 × 99000 = Rs. 198000
So, Loss = Rs. 2000
So, Loss% = 2000/200000 x 100 = 1%
Unattempted
C.P. of car sold at 10% profit = (100/100 + gain%) x S.P.
= 100/110 x 99000 = Rs. 90000
C.P. of car sold at 10% loss = 100/90 x 99000 = Rs. 110000
Total C.P. = Rs. (90000 + 110000)
= Rs. 200000
Total S.P. = Rs. 2 × 99000 = Rs. 198000
So, Loss = Rs. 2000
So, Loss% = 2000/200000 x 100 = 1%
-
Question 76 of 80
76. Question
If a * b = 2a – 3b + ab, then 3 * 5 + 5 * 3 is equal to :
(A) 22
() 24
(C) 26
(D) 28
-
-
-
-
Correct
Incorrect
a * b = 2a – 3b + ab
3 * 5 = 2 × 3 – 3 × 5 + 3 × 5 = 6
5 * 3 = 2 × 5 – 3 × 3 + 3 × 5
= 10 – 9 + 15 = 16
Therefore, 3 * 5 + 5 *3
= 6 + 16 = 22
Unattempted
a * b = 2a – 3b + ab
3 * 5 = 2 × 3 – 3 × 5 + 3 × 5 = 6
5 * 3 = 2 × 5 – 3 × 3 + 3 × 5
= 10 – 9 + 15 = 16
Therefore, 3 * 5 + 5 *3
= 6 + 16 = 22
-
Question 77 of 80
77. Question
Find the next term in the series.
2750, 2624, 2474, 2298, 2094, ?
(A) 1848
(B) 1856
(C) 1836
(D) 1815
-
-
-
-
Correct
Incorrect
The series is as follows:
2750 – 9 x 14 = 2624
2624 – 10 x 15 = 2474
2474 – 11 x 16 = 2298
2298 – 12 x 17 = 2094
2094 – 13 x 18 = 1856
Unattempted
The series is as follows:
2750 – 9 x 14 = 2624
2624 – 10 x 15 = 2474
2474 – 11 x 16 = 2298
2298 – 12 x 17 = 2094
2094 – 13 x 18 = 1856
-
Question 78 of 80
78. Question
DIRECTION : READ THE FOLLOWING PASSAGE AND ANSWER THE QUESTIONS THAT FOLLOW. YOUR ANSWERS TO THESE QUESTIONS SHOULD BE BASED ON THE PASSAGE ONLY.
Evolutionary psychology takes as its starting point the uncontroversial assertion that the anatomical and physiological features of the human brain have arisen as a result of adaptations to the demands of the environment over the millennia. However, from this reasonable point of departure, these psychologists make unreasonable extrapolations. They claim that the behaviour of contemporary man, in almost all its aspects is a reflection of features of the brain that acquired their present characteristics during those earliest days of our species when early man struggled to survive and multiply. These suggestions have a ready audience and the idea that Stone age man is alive in our genome and dictating aspects of our genome and dictating aspects of our behaviour has gained ground in the popular imagination.
The tabloids repeatedly run articles about ‘discoveries’ relating to ‘genes’ for aggression, depression, repression, and anything for which we need a readymade excuse. Such insistence on a genetic basis for behaviour negates the cultural influences and the social realities that separate us from our ancestors. The difficulty with pseudo science of this nature is just this popular appeal. People are eager to accept what is printed as incontrovertible, assuming quite without foundation, that anything printed has bona find antecedents. We would do well to remember that the phrenologists of the 19th century held sway for a considerable time in the absence of any evidence that behavioural tendencies could be deuced from the shape of the skull. The phrenologists are no more, but their genes would seem to be thriving.
The author mentions phrenologists as
(1) pseudo scientists who are the logical antecedents of evolutionary psychologists.
(2) a group with inherent appeal to the followers of evolutionary psychologists.
(3) a warning against blind acceptance of ideas.
Which of the statements (s) given above is/ are correct?
(A) 1 only
(B) 2 and 3 only
(C) 3 only
(D) None of these
-
-
-
-
Correct
Incorrect
The words ‘we would do well to remember’ confirm that the author is warning us about something. He is reminding us that the phrenologists gained prominence without any evidence for their beliefs and so warns us against blind acceptance of ideas.
Unattempted
The words ‘we would do well to remember’ confirm that the author is warning us about something. He is reminding us that the phrenologists gained prominence without any evidence for their beliefs and so warns us against blind acceptance of ideas.
-
Question 79 of 80
79. Question
If p × q = p + q + p/q , the value of 8 × 2 is :
(A) 6
(B) 10
(C) 14
(D) 16
-
-
-
-
Correct
Incorrect
pxq = p+q+p/q
So, 8×2 = 8+2 + 8/2 ́
= 10 + 4 = 14
Unattempted
pxq = p+q+p/q
So, 8×2 = 8+2 + 8/2 ́
= 10 + 4 = 14
-
Question 80 of 80
80. Question
If x * y = 3x +2y,
Then 2 * 3 + 3 * 4 is equal to
(A) 18
(B) 29
(C) 32
(D) 38
-
-
-
-
Correct
Incorrect
x * y = 3x + 2y
2 * 3 + 3 * 4
= 3 × 2 + 2 × 3 + 3 × 3 + 2 × 4
= 6 + 6 + 9 + 8 = 29
Unattempted
x * y = 3x + 2y
2 * 3 + 3 * 4
= 3 × 2 + 2 × 3 + 3 × 3 + 2 × 4
= 6 + 6 + 9 + 8 = 29
CSAT FLT – 4 – PRELIMS 2024
0 of 80 questions completed
Questions:
- 1
- 2
- 3
- 4
- 5
- 6
- 7
- 8
- 9
- 10
- 11
- 12
- 13
- 14
- 15
- 16
- 17
- 18
- 19
- 20
- 21
- 22
- 23
- 24
- 25
- 26
- 27
- 28
- 29
- 30
- 31
- 32
- 33
- 34
- 35
- 36
- 37
- 38
- 39
- 40
- 41
- 42
- 43
- 44
- 45
- 46
- 47
- 48
- 49
- 50
- 51
- 52
- 53
- 54
- 55
- 56
- 57
- 58
- 59
- 60
- 61
- 62
- 63
- 64
- 65
- 66
- 67
- 68
- 69
- 70
- 71
- 72
- 73
- 74
- 75
- 76
- 77
- 78
- 79
- 80
Information
.
You have already completed the Test before. Hence you can not start it again.
Test is loading…
You must sign in or sign up to start the Test.
You have to finish following quiz, to start this Test:
Your results are here!! for” CSAT FLT – 4 – PRELIMS 2024 “
0 of 80 questions answered correctly
Your time:
Time has elapsed
Your Final Score is : 0
You have attempted : 0
Number of Correct Questions : 0 and scored 0
Number of Incorrect Questions : 0 and Negative marks 0
-
Not categorized
You have attempted: 0
Number of Correct Questions: 0 and scored 0
Number of Incorrect Questions: 0 and Negative marks 0
Pos. | Name | Entered on | Points | Result |
---|---|---|---|---|
Table is loading | ||||
No data available | ||||
- 1
- 2
- 3
- 4
- 5
- 6
- 7
- 8
- 9
- 10
- 11
- 12
- 13
- 14
- 15
- 16
- 17
- 18
- 19
- 20
- 21
- 22
- 23
- 24
- 25
- 26
- 27
- 28
- 29
- 30
- 31
- 32
- 33
- 34
- 35
- 36
- 37
- 38
- 39
- 40
- 41
- 42
- 43
- 44
- 45
- 46
- 47
- 48
- 49
- 50
- 51
- 52
- 53
- 54
- 55
- 56
- 57
- 58
- 59
- 60
- 61
- 62
- 63
- 64
- 65
- 66
- 67
- 68
- 69
- 70
- 71
- 72
- 73
- 74
- 75
- 76
- 77
- 78
- 79
- 80
- Answered
- Review
- Question 1 of 80
1. Question
DIRECTION : READ THE FOLLOWING PASSAGE AND ANSWER THE QUESTION THAT FOLLOW. YOUR ANSWER TO THIS QUESTION SHOULD BE BASED ON THE PASSAGE ONLY.
It's easy to forget that most of the world's languages are still transmitted orally with no widely established written form. While speech communities are increasingly involved in projects to protect their languages – in print, on air and online – orality is fragile and contributes to linguistic vulnerability. But indigenous languages are about much more than unusual words and intriguing grammar: They function as vehicles for the transmission of cultural traditions, environmental understandings and knowledge about medicinal plants, all at risk when elders die and livelihoods are disrupted.
Both push and pull factors lead to the decline of languages. Through war, famine and natural disasters, whole communities can be destroyed, taking their language with them to the grave, such as the indigenous populations of Tasmania who were wiped out by colonists. More commonly, speakers live on but abandon their language in favor of another vernacular, a widespread process that linguists refer to as “language shift” from which few languages are immune. Such trading up and out of a speech form occurs for complex political, cultural and economic reasons – sometimes voluntary for economic and educational reasons, although often amplified by state coercion or neglect. Welsh, long stigmatized and disparaged by the British state, has rebounded with vigor.
Many speakers of endangered, poorly documented languages have embraced new digital media with excitement. Speakers of previously exclusively oral tongues are turning to the web as a virtual space for languages to live on. Internet technology offers powerful ways for oral traditions and cultural practices to survive, even thrive, among increasingly mobile communities. I have watched as videos of traditional wedding ceremonies and songs are recorded on smartphones in London by Nepali migrants, then uploaded to YouTube and watched an hour later by relatives in remote Himalayan villages . . .Globalization is regularly, and often uncritically, pilloried as a major threat to linguistic diversity. But in fact, globalization is as much process as it is ideology, certainly when it comes to language. The real forces behind cultural homogenization are unbending beliefs, exchanged through a globalized delivery system, reinforced by the historical monolingualism prevalent in much of the West.
Monolingualism – the condition of being able to speak only one language – is regularly accompanied by a deep-seated conviction in the value of that language over all others. Across the largest economies that make up the G8, being monolingual is still often the norm, with multilingualism appearing unusual and even somewhat exotic. The monolingual mindset stands in sharp contrast to the lived reality of most the world, which throughout its history has been more multilingual than unilingual. Monolingualism, then, not globalization, should be our primary concern.
Multilingualism can help us live in a more connected and more interdependent world. By widening access to technology, globalization can support indigenous and scholarly communities engaged in documenting and protecting our shared linguistic heritage. For the last 5,000 years, the rise and fall of languages was intimately tied to the plow, sword and book. In our digital age, the keyboard, screen and web will play a decisive role in shaping the future linguistic diversity of our species.
We can infer all of the following about indigenous languages from the passage EXCEPT that:
(A) they are repositories of traditional knowledge about the environment and culture.
(B) people are increasingly working on documenting these languages.
(C) they are in danger of being wiped out as most can only be transmitted orally.
(D) their vocabulary and grammatical constructs have been challenging to document.CorrectIncorrectThis question is pertaining to indigenous languages. From third paragraph, we can infer options A,B and C. Option D cannot be inferred because if many speakers of poorly documented languages have embraced new digital media with excitement, there would not be many challenges in it. Moreover, there is no evidence of any challenges faced.
UnattemptedThis question is pertaining to indigenous languages. From third paragraph, we can infer options A,B and C. Option D cannot be inferred because if many speakers of poorly documented languages have embraced new digital media with excitement, there would not be many challenges in it. Moreover, there is no evidence of any challenges faced.
- Question 2 of 80
2. Question
DIRECTION : READ THE FOLLOWING PASSAGE AND ANSWER THE QUESTION THAT FOLLOW. YOUR ANSWER TO THIS QUESTION SHOULD BE BASED ON THE PASSAGE ONLY.
Today we can hardly conceive of ourselves without an unconscious. Yet between 1700 and1900, this notion developed as a genuinely original thought. The “unconscious” burst the shell of conventional language, coined as it had been to embody the fleeting ideas and the shifting conceptions of several generations until, finally, it became fixed and defined in specialized terms within the realm of medical psychology and Freudian psychoanalysis.
The vocabulary concerning the soul and the mind increased enormously in the course of the nineteenth century. The enrichments of literary and intellectual language led to an altered understanding of the meanings that underlie time-honored expressions and traditional catchwords. At the same time, once coined, powerful new ideas attracted to themselves a whole host of seemingly unrelated issues, practices, and experiences, creating a peculiar network of preoccupations that as a group had not existed before. The drawn-out attempt to approach and define the unconscious brought together the spiritualist and the psychical researcher of borderline phenomena (such as apparitions, spectral illusions, haunted houses, mediums, trance, automatic writing); the psychiatrist or alienist probing the nature of mental disease, of abnormal ideation, hallucination, delirium, melancholia, mania; the surgeon performing operations with the aid of hypnotism; the magnetizer claiming to correct the disequilibrium in the universal flow of magnetic fluids but who soon came to be regarded as a clever manipulator of the imagination; the physiologist and the physician who puzzled oversleep, dreams, sleepwalking, anesthesia, the influence of the mind on the body in health and disease; the neurologist concerned with the functions of the brain and the physiological basis of mental life; the philosopher interested in the will, the emotions, consciousness, knowledge, imagination and the creative genius; and, last but not least, the psychologist.
Significantly, most if not all of these practices (for example, hypnotism in surgery or psychological magnetism) originated in the waning years of the eighteenth century and during the early decades of the nineteenth century, as did some of the disciplines (such as psychology and psychical research). The majority of topics too were either new or assumed hitherto unknown colors. Thus, before 1790, few if any spoke, in medical terms, of the affinity between creative genius and the hallucinations of the insane .
Striving vaguely and independently to give expression to a latent conception, various lines of thought can be brought together by some novel term. The new concept then serves as a kind of resting place or stocktaking in the development of ideas, giving satisfaction and a stimulus for further discussion or speculation. Thus, the massive introduction of the term unconscious by Hartmann in 1869 appeared to focalize many stray thoughts, affording a temporary feeling that a crucial step had been taken forward, a comprehensive knowledge gained, a knowledge that required only further elaboration, explication, and unfolding in order to bring in a bounty of higher understanding. Ultimately, Hartmann's attempt at defining the unconscious proved fruitless because he extended its reach into every realm of organic and inorganic, spiritual, intellectual, and instinctive existence, severely diluting the precision and compromising the impact of the concept.
All of the following statements may be considered valid inferences from the passage, EXCEPT:
(A) New conceptions in the nineteenth century could provide new knowledge because of the establishment of fields such as anaesthesiology.
(B) Unrelated practices began to be treated as related to each other, as knowledge of the mind grew in the nineteenth century.
(C) Without the linguistic developments of the nineteenth century, the growth of understanding of the soul and the mind may not have happened.
(D) Eighteenth century thinkers were the first to perceive a connection between creative genius and insanity.CorrectIncorrectThis question apparently looks tough because it has the phrase “valid inferences… except”. Option B can be inferred from the entire second paragraph. From the first paragraph we can infer C. From the last sentence of second last paragraph we can infer D. Thus A is the best choice, as we don't have any evidence for A.
UnattemptedThis question apparently looks tough because it has the phrase “valid inferences… except”. Option B can be inferred from the entire second paragraph. From the first paragraph we can infer C. From the last sentence of second last paragraph we can infer D. Thus A is the best choice, as we don't have any evidence for A.
- Question 3 of 80
3. Question
DIRECTION : READ THE FOLLOWING PASSAGE AND ANSWER THE QUESTION THAT FOLLOW. YOUR ANSWER TO THIS QUESTION SHOULD BE BASED ON THE PASSAGE ONLY.
As high-level visits go, Prime Minister Narendra Modi’s visit to Israel was bound to attract superlatives like ‘historic’ and ‘groundbreaking’. Still, it is clear that the buzz in the relationship is on account of Mr. Modi’s personal diplomatic style and his host’s equally warm response.
Israel’s Prime Minister Benjamin Netanyahu set the tone when he welcomed Mr. Modi at the airport together with the spiritual leaders of all the major faiths in the region, an honor traditionally reserved for the U.S. President and the Pope. Mr. Modi’s trademark bear hugs were reciprocated, three at the airport, and by the time the visit ended, the TV commentators had lost count!
Mr. Netanyahu gushingly described the relationship between the two countries as “a marriage made in heaven”, but behind the success was a receptive political backdrop as well as the careful planning undertaken by both sides.
Marking 25 years of establishing diplomatic relations between the two countries added to the historic character of an Indian Prime Minister’s first visit to a country that had quietly emerged as a strong defense partner. There had been high-level exchanges but the Indian response was cautious. Foreign Minister Shimon Peres first visited India in 1993, and Jaswant Singh reciprocated in 2000; President Ezer Weizman came in 1997, while President Pranab Mukherji’s visit only took place in 2015. The first Israeli Prime Minister to visit India was Ariel Sharon in 2003, and from the time Mr. Modi came to power, a return visit was a certainty. The two leaders had met on the margins of the UN General Assembly and continued their relationship by tweeting their greetings on Hanukkah and Diwali.
Yet, official-level exchanges between the two countries have been intensive, beginning with the visit of Foreign Secretary J.N. Dixit in early 1993. While relations between Mossad and Research and Analysis Wing had existed earlier, the strategic partnership got cemented when National Security Adviser-level dialogue was established in 1999 between Brajesh Mishra and Gen. (retd.) David Ivry. Gen. Ivry was a former Air Force chief who had led the air raid on Osirak, the Iraqi nuclear reactor, in 1981. Incidentally, Israel was one of the few countries that showed a complete understanding of India’s decision to undertake the nuclear tests in 1998. This reinforced both the defense and the counter-terrorism cooperation relationship.
By 2000, India was acquiring surface-to-air missiles (Barak 1) and UAVs (unmanned aerial vehicles) from Israel. Subsequently, the refurbishing of MiG-21 aircraft employed Israeli avionics. During the 1999 Kargil war, Israel assisted with laser-guidance kits mated with gravity bombs, carried by the Mirage 2000 aircraft. With U.S. concurrence, Israel sold India the Phalcon airborne early warning system and mounted on the Russian Il-76, provided AWACS capability. Subsequent acquisitions have included Spike anti-tank guided missiles and the long range surface-to-air missiles in both the naval and land versions. Today, Israel has emerged as the third-largest defense supplier for India and accounts for over 40% of Israel’s defense exports.
Commercial relations between the diamond traders in Gujarat and Israel had existed before 1992, but now annual trade grew from $200 million to nearly $5 billion with gems and jewelry accounting for nearly 40%. Gradually, Science and technology, agriculture, biotech, and space emerged as new areas of cooperation.Tourism provided an impetus to people-to-people relations. India emerged as the preferred destination for young Israelis wanting to unwind after their compulsory military service and Hebrew signage in Varanasi, Manali, and Goa is a common sight. Ambassador Pavan Kapoor was being quite mattered of fact when he described the Modi visit as a ‘coming out visit’ for the relationship.
Mr. Netanyahu Gushingly described the relationship between the two countries as:
(A) A friendship made on earth
(B) A friendship made in heaven
(C) A marriage made in heaven
(D) An enmity made in heavenCorrectIncorrectMr. Netanyahu gushingly described the relationship between the two countries as “a marriage made in heaven”,
UnattemptedMr. Netanyahu gushingly described the relationship between the two countries as “a marriage made in heaven”,
- Question 4 of 80
4. Question
DIRECTION : READ THE FOLLOWING PASSAGE AND ANSWER THE QUESTION THAT FOLLOW. YOUR ANSWER TO THIS QUESTION SHOULD BE BASED ON THE PASSAGE ONLY.
As high-level visits go, Prime Minister Narendra Modi’s visit to Israel was bound to attract superlatives like ‘historic’ and ‘groundbreaking’. Still, it is clear that the buzz in the relationship is on account of Mr. Modi’s personal diplomatic style and his host’s equally warm response.
Israel’s Prime Minister Benjamin Netanyahu set the tone when he welcomed Mr. Modi at the airport together with the spiritual leaders of all the major faiths in the region, an honor traditionally reserved for the U.S. President and the Pope. Mr. Modi’s trademark bear hugs were reciprocated, three at the airport, and by the time the visit ended, the TV commentators had lost count!
Mr. Netanyahu gushingly described the relationship between the two countries as “a marriage made in heaven”, but behind the success was a receptive political backdrop as well as the careful planning undertaken by both sides.
Marking 25 years of establishing diplomatic relations between the two countries added to the historic character of an Indian Prime Minister’s first visit to a country that had quietly emerged as a strong defense partner. There had been high-level exchanges but the Indian response was cautious. Foreign Minister Shimon Peres first visited India in 1993, and Jaswant Singh reciprocated in 2000; President Ezer Weizman came in 1997, while President Pranab Mukherji’s visit only took place in 2015. The first Israeli Prime Minister to visit India was Ariel Sharon in 2003, and from the time Mr. Modi came to power, a return visit was a certainty. The two leaders had met on the margins of the UN General Assembly and continued their relationship by tweeting their greetings on Hanukkah and Diwali.
Yet, official-level exchanges between the two countries have been intensive, beginning with the visit of Foreign Secretary J.N. Dixit in early 1993. While relations between Mossad and Research and Analysis Wing had existed earlier, the strategic partnership got cemented when National Security Adviser-level dialogue was established in 1999 between Brajesh Mishra and Gen. (retd.) David Ivry. Gen. Ivry was a former Air Force chief who had led the air raid on Osirak, the Iraqi nuclear reactor, in 1981. Incidentally, Israel was one of the few countries that showed a complete understanding of India’s decision to undertake the nuclear tests in 1998. This reinforced both the defense and the counter-terrorism cooperation relationship.
By 2000, India was acquiring surface-to-air missiles (Barak 1) and UAVs (unmanned aerial vehicles) from Israel. Subsequently, the refurbishing of MiG-21 aircraft employed Israeli avionics. During the 1999 Kargil war, Israel assisted with laser-guidance kits mated with gravity bombs, carried by the Mirage 2000 aircraft. With U.S. concurrence, Israel sold India the Phalcon airborne early warning system and mounted on the Russian Il-76, provided AWACS capability. Subsequent acquisitions have included Spike anti-tank guided missiles and the long range surface-to-air missiles in both the naval and land versions. Today, Israel has emerged as the third-largest defense supplier for India and accounts for over 40% of Israel’s defense exports.
Commercial relations between the diamond traders in Gujarat and Israel had existed before 1992, but now annual trade grew from $200 million to nearly $5 billion with gems and jewelry accounting for nearly 40%. Gradually, Science and technology, agriculture, biotech, and space emerged as new areas of cooperation.Tourism provided an impetus to people-to-people relations. India emerged as the preferred destination for young Israelis wanting to unwind after their compulsory military service and Hebrew signage in Varanasi, Manali, and Goa is a common sight. Ambassador Pavan Kapoor was being quite mattered of fact when he described the Modi visit as a ‘coming out visit’ for the relationship.
Which of the following statement is true according to the passage?
I. Foreign Minister Shimon Peres first visited India in 1993, and Jaswant Singh reciprocated in 2000; President Ezer Weizman came in 1997, while President Pranab Mukherji’s visit only took place in 2015
II. Foreign Minister Shimon Peres first visited India in 1993, and Jaswant Singh reciprocated in 2001; President Ezer Weizman came in 1997, while President Pranab Mukherji’s visit only took place in 2015
III. Foreign Minister Shimon Peres first visited India in 1993, and Jaswant Singh reciprocated in 2000; President Ezer Weizman came in 1998, while President Pranab Mukherji’s visit only took place in 2015
(A) Only I
(B) Only II
(C) Only III
(D) Both I and IIICorrectIncorrectForeign Minister Shimon Peres first visited India in 1993, and Jaswant Singh reciprocated in 2000; President Ezer Weizman came in 1997, while President Pranab Mukherji’s visit only took place in 2015.
UnattemptedForeign Minister Shimon Peres first visited India in 1993, and Jaswant Singh reciprocated in 2000; President Ezer Weizman came in 1997, while President Pranab Mukherji’s visit only took place in 2015.
- Question 5 of 80
5. Question
DIRECTION : READ THE FOLLOWING PASSAGE AND ANSWER THE QUESTION THAT FOLLOW. YOUR ANSWER TO THIS QUESTION SHOULD BE BASED ON THE PASSAGE ONLY.
As high-level visits go, Prime Minister Narendra Modi’s visit to Israel was bound to attract superlatives like ‘historic’ and ‘groundbreaking’. Still, it is clear that the buzz in the relationship is on account of Mr. Modi’s personal diplomatic style and his host’s equally warm response.
Israel’s Prime Minister Benjamin Netanyahu set the tone when he welcomed Mr. Modi at the airport together with the spiritual leaders of all the major faiths in the region, an honor traditionally reserved for the U.S. President and the Pope. Mr. Modi’s trademark bear hugs were reciprocated, three at the airport, and by the time the visit ended, the TV commentators had lost count!
Mr. Netanyahu gushingly described the relationship between the two countries as “a marriage made in heaven”, but behind the success was a receptive political backdrop as well as the careful planning undertaken by both sides.
Marking 25 years of establishing diplomatic relations between the two countries added to the historic character of an Indian Prime Minister’s first visit to a country that had quietly emerged as a strong defense partner. There had been high-level exchanges but the Indian response was cautious. Foreign Minister Shimon Peres first visited India in 1993, and Jaswant Singh reciprocated in 2000; President Ezer Weizman came in 1997, while President Pranab Mukherji’s visit only took place in 2015. The first Israeli Prime Minister to visit India was Ariel Sharon in 2003, and from the time Mr. Modi came to power, a return visit was a certainty. The two leaders had met on the margins of the UN General Assembly and continued their relationship by tweeting their greetings on Hanukkah and Diwali.
Yet, official-level exchanges between the two countries have been intensive, beginning with the visit of Foreign Secretary J.N. Dixit in early 1993. While relations between Mossad and Research and Analysis Wing had existed earlier, the strategic partnership got cemented when National Security Adviser-level dialogue was established in 1999 between Brajesh Mishra and Gen. (retd.) David Ivry. Gen. Ivry was a former Air Force chief who had led the air raid on Osirak, the Iraqi nuclear reactor, in 1981. Incidentally, Israel was one of the few countries that showed a complete understanding of India’s decision to undertake the nuclear tests in 1998. This reinforced both the defense and the counter-terrorism cooperation relationship.
By 2000, India was acquiring surface-to-air missiles (Barak 1) and UAVs (unmanned aerial vehicles) from Israel. Subsequently, the refurbishing of MiG-21 aircraft employed Israeli avionics. During the 1999 Kargil war, Israel assisted with laser-guidance kits mated with gravity bombs, carried by the Mirage 2000 aircraft. With U.S. concurrence, Israel sold India the Phalcon airborne early warning system and mounted on the Russian Il-76, provided AWACS capability. Subsequent acquisitions have included Spike anti-tank guided missiles and the long range surface-to-air missiles in both the naval and land versions. Today, Israel has emerged as the third-largest defense supplier for India and accounts for over 40% of Israel’s defense exports.
Commercial relations between the diamond traders in Gujarat and Israel had existed before 1992, but now annual trade grew from $200 million to nearly $5 billion with gems and jewelry accounting for nearly 40%. Gradually, Science and technology, agriculture, biotech, and space emerged as new areas of cooperation.Tourism provided an impetus to people-to-people relations. India emerged as the preferred destination for young Israelis wanting to unwind after their compulsory military service and Hebrew signage in Varanasi, Manali, and Goa is a common sight. Ambassador Pavan Kapoor was being quite mattered of fact when he described the Modi visit as a ‘coming out visit’ for the relationship.
Which of the following statement is not true according to the passage?
I. By 2000, India was acquiring surface-to- surface (Barak 1) and UAVs (unmanned aerial vehicles) from Israel.
II. During the 1999 Kargil war, Israel assisted with laser-guidance kits mated with gravity bombs, carried by the Mirage 2000 aircraft.
III. Israel sold India the Phalcon airborne early warning system and mounted on the Russian Jl-76, provided AWACS capability.
(A) Only I
(B) Only II
(C) Both I and II
(D) Both I and IIICorrectIncorrectIsrael sold India the Phalcon airborne early warning system and mounted on the Russian Il-76, provided AWACS capability.
By 2000, India was acquiring surface-to-air missiles (Barak 1) and UAVs (unmanned aerial vehicles) from Israel.UnattemptedIsrael sold India the Phalcon airborne early warning system and mounted on the Russian Il-76, provided AWACS capability.
By 2000, India was acquiring surface-to-air missiles (Barak 1) and UAVs (unmanned aerial vehicles) from Israel. - Question 6 of 80
6. Question
DIRECTION : READ THE FOLLOWING PASSAGE AND ANSWER THE QUESTIONS THAT FOLLOW. YOUR ANSWERS TO THESE QUESTIONS SHOULD BE BASED ON THE PASSAGE ONLY.
Many cities haves distributed standardized recycling containers to all households with directions that read: “We would prefer that you use this new container as your primary recycling container as this will expedite pick-up of recyclables. Additional recycling containers may be purchased from the City.”
According to the passage, which of the following is true about the new containers?
(A) The new containers are far better than other containers in every way.
(B) The new containers will help increase the efficiency of the recycling program.
(C) The new containers hold more than the old containers did.
(D) The new containers are less expensive than the old containers.CorrectIncorrectThe passage state use of the new containers will expedite pick-up of recyclables. This indicates that the new containers will make the recycling program more efficient.
UnattemptedThe passage state use of the new containers will expedite pick-up of recyclables. This indicates that the new containers will make the recycling program more efficient.
- Question 7 of 80
7. Question
DIRECTION : READ THE FOLLOWING PASSAGE AND ANSWER THE QUESTION THAT FOLLOW. YOUR ANSWER TO THIS QUESTION SHOULD BE BASED ON THE PASSAGE ONLY.
Cuttlefish are full of personality, as behavioral ecologist Alexandra Schnell found out while researching the cephalopod's potential to display self-control. ” Self-control is thought to be the cornerstone of intelligence, as it is an important prerequisite for complex decision-making and planning for the future,” says Schnell .
[Schnell's] study used a modified version of the ” marshmallow test ” – During the original marshmallow test, psychologist Walter Mischel presented children between age four and six with one marshmallow. He told them that if they waited 15 minutes and didn't eat it, he would give them a second marshmallow. A long-term follow-up study showed that the children who waited for the second marshmallow had more success later in life. The cuttlefish version of the experiment looked a lot different. The researchers worked with six cuttlefish under nine months old and presented them with seafood instead of sweets. (Preliminary experiments showed that cuttlefishes' favorite food is live grass shrimp, while raw prawns are so-so and Asian shore crab is nearly unacceptable.) Since the researchers couldn't explain to the cuttlefish that they would need to wait for their shrimp, they trained them to recognize certain shapes that indicated when a food item would become available. The symbols were pasted on transparent drawers so that the cuttlefish could see the food that was stored inside. One drawer, labeled with a circle to mean “immediate,” held raw king prawn. Another drawer, labeled with a triangle to mean “delayed,” held live grass shrimp. During a control experiment, square labels meant “never.”
“If their self-control is flexible and I hadn't just trained them to wait in any context, you would expect the cuttlefish to take the immediate reward [in the control], even if it's their second preference,” says Schnell . . . and that's what they did. That showed the researchers that cuttlefish wouldn't reject the prawns if it was the only food available. In the experimental trials, the cuttlefish didn't jump on the prawns if the live grass shrimp were labeled with a triangle – many waited for the shrimp drawer to open up. Each time the cuttlefish showed it could wait, the researchers tacked another ten seconds on to the next round of waiting before releasing the shrimp. The longest that a cuttlefish waited was 130 seconds.
Schnell [says] that the cuttlefish usually sat at the bottom of the tank and looked at the two food items while they waited, but sometimes, they would turn away from the king prawn “as if to distract themselves from the temptation of the immediate reward.” In past studies, humans, chimpanzees, parrots and dogs also tried to distract themselves while waiting for a reward.
Not every species can use self-control, but most of the animals that can share another trait in common: long, social lives. Cuttlefish, on the other hand, are solitary creatures that don't form relationships even with mates or young. “We don't know if living in a social group is important for complex cognition unless we also show those abilities are lacking in less social species,” says . . . comparative psychologist Jennifer Vonk.
All of the following constitute a point of difference between the “original” and “modified” versions of the marshmallow test EXCEPT that:
(A) the former was performed over a longer time span than the latter.
(B) the former correlated self-control and future success, while the latter correlated self-control and survival advantages.
(C) the former had human subjects, while the latter had cuttlefish.
(D) the former used verbal communication with its subjects, while the latter had to develop a symbolic means of communication.CorrectIncorrectIn this question, we have to pick a choice that is not a difference between the original and modified versions of the marshmallow test. Option A is the difference. The time difference can be seen in the first paragraph. C is an obvious difference which need not be explained. D is also an obvious difference. B is the best choice as nowhere in the passage is it implied that the latter correlated survival advantages
UnattemptedIn this question, we have to pick a choice that is not a difference between the original and modified versions of the marshmallow test. Option A is the difference. The time difference can be seen in the first paragraph. C is an obvious difference which need not be explained. D is also an obvious difference. B is the best choice as nowhere in the passage is it implied that the latter correlated survival advantages
- Question 8 of 80
8. Question
DIRECTION : READ THE FOLLOWING PASSAGE AND ANSWER THE QUESTIONS THAT FOLLOW. YOUR ANSWERS TO THESE QUESTIONS SHOULD BE BASED ON THE PASSAGE ONLY.
Ratatouille is a dish that has grown in popularity over the last few years. It features eggplant, zucchini, tomatoes, peppers, and garlic; chopped, mixed, sautéed, and finally, cooked slowly over low heat. As the vegetables cook slowly, they make their own broth, which may be extended with a little tomato paste. The name ratatouille comes from the French word touiller, meaning to stir or mix together.
Which of the following is the correct order of steps for making ratatouille?
(A) chop vegetables, add tomato paste, stir or mix together
(B) mix the vegetables together, sauté them, and add tomato paste
(C) cook the vegetables slowly, mix them together, add tomato paste
(D) add tomato paste to extend the broth and cook slowly over low heatCorrectIncorrectUnattempted - Question 9 of 80
9. Question
DIRECTION : READ THE FOLLOWING PASSAGE AND ANSWER THE QUESTION THAT FOLLOW. YOUR ANSWER TO THIS QUESTION SHOULD BE BASED ON THE PASSAGE ONLY.
I have elaborated – a framework for analyzing the contradictory pulls on [Indian] nationalist ideology in its struggle against the dominance of colonialism and the resolution it offered to those contradictions. Briefly, this resolution was built around a separation of the domain of culture into two spheres – the material and the spiritual. It was in the material sphere that the claims of Western civilization were the most powerful. Science, technology, rational forms of economic organization, modern methods of statecraft – these had given the European countries the strength to subjugate the non-European people . To overcome this domination, the colonized people had to learn those superior techniques of organizing material life and incorporate them within their own cultures. But this could not mean the imitation of the West in every aspect of life, for then the very distinction between the West and the East would vanish – the self-identity of national culture would itself be threatened. The discourse of nationalism shows that the material/spiritual distinction was condensed into an analogous, but ideologically far more powerful, dichotomy: that between the outer and the inner. Applying the inner/outer distinction to the matter of concrete day-to-day living separates the social space into ghar and bāhir, the home and the world. The world is the external, the domain of the material; the home represents one's inner spiritual self, one's true identity. The world is a treacherous terrain of the pursuit of material interests, where practical considerations reign supreme. It is also typically the domain of the male. The home in its essence must remain unaffected by the profane activities of the material world – and woman is its representation. And so one gets an identification of social roles by gender to correspond with the separation of the social space into ghar and bāhir.
The colonial situation, and the ideological response of nationalism to the critique of Indian tradition, introduced an entirely new substance to [these dichotomies] and effected their transformation. The material/spiritual dichotomy, to which the terms world and home corresponded, had acquired . a very special significance in the nationalist mind. The world was where the European power had challenged the non-European peoples and, by virtue of its superior material culture, had subjugated them. But, the nationalists asserted, it had failed to colonize the inner, essential, identity of the East which lay in its distinctive, and superior, spiritual culture. [I]n the entire phase of the national struggle, the crucial need was to protect, preserve and strengthen the inner core of the national culture, its spiritual essence.
Once we match this new meaning of the home/world dichotomy with the identification of social roles by gender, we get the ideological framework within which nationalism answered the women's question. It would be a grave error to see in this, as liberals are apt to in their despair at the many marks of social conservatism in nationalist practice, a total rejection of the West. Quite the contrary: the nationalist paradigm in fact supplied an ideological principle of selection.
Which one of the following, if true, would weaken the author's claims in the passage?
(A) The colonial period saw the hybridisation of Indian culture in all realms as it came in contact with British/European culture.
(B) Indian nationalists rejected the cause of English education for women during the colonial period.
(C) The Industrial Revolution played a crucial role in shaping the economic prowess of Britain in the eighteenth century.
(D) Forces of colonial modernity played an important role in shaping anti-colonial Indian nationalism.CorrectIncorrectThe passage says that Indian nationalist borrowed from the material sphere, not the spiritual sphere. A says that “there was hybridization of Indian culture in all spheres”. This weakens the author's claims in the passage.
UnattemptedThe passage says that Indian nationalist borrowed from the material sphere, not the spiritual sphere. A says that “there was hybridization of Indian culture in all spheres”. This weakens the author's claims in the passage.
- Question 10 of 80
10. Question
DIRECTION : READ THE FOLLOWING PASSAGE AND ANSWER THE QUESTION THAT FOLLOW. YOUR ANSWER TO THIS QUESTION SHOULD BE BASED ON THE PASSAGE ONLY.
I have elaborated – a framework for analyzing the contradictory pulls on [Indian] nationalist ideology in its struggle against the dominance of colonialism and the resolution it offered to those contradictions. Briefly, this resolution was built around a separation of the domain of culture into two spheres – the material and the spiritual. It was in the material sphere that the claims of Western civilization were the most powerful. Science, technology, rational forms of economic organization, modern methods of statecraft – these had given the European countries the strength to subjugate the non-European people . To overcome this domination, the colonized people had to learn those superior techniques of organizing material life and incorporate them within their own cultures. But this could not mean the imitation of the West in every aspect of life, for then the very distinction between the West and the East would vanish – the self-identity of national culture would itself be threatened. The discourse of nationalism shows that the material/spiritual distinction was condensed into an analogous, but ideologically far more powerful, dichotomy: that between the outer and the inner. Applying the inner/outer distinction to the matter of concrete day-to-day living separates the social space into ghar and bāhir, the home and the world. The world is the external, the domain of the material; the home represents one's inner spiritual self, one's true identity. The world is a treacherous terrain of the pursuit of material interests, where practical considerations reign supreme. It is also typically the domain of the male. The home in its essence must remain unaffected by the profane activities of the material world – and woman is its representation. And so one gets an identification of social roles by gender to correspond with the separation of the social space into ghar and bāhir.
The colonial situation, and the ideological response of nationalism to the critique of Indian tradition, introduced an entirely new substance to [these dichotomies] and effected their transformation. The material/spiritual dichotomy, to which the terms world and home corresponded, had acquired . a very special significance in the nationalist mind. The world was where the European power had challenged the non-European peoples and, by virtue of its superior material culture, had subjugated them. But, the nationalists asserted, it had failed to colonize the inner, essential, identity of the East which lay in its distinctive, and superior, spiritual culture. [I]n the entire phase of the national struggle, the crucial need was to protect, preserve and strengthen the inner core of the national culture, its spiritual essence.
Once we match this new meaning of the home/world dichotomy with the identification of social roles by gender, we get the ideological framework within which nationalism answered the women's question. It would be a grave error to see in this, as liberals are apt to in their despair at the many marks of social conservatism in nationalist practice, a total rejection of the West. Quite the contrary: the nationalist paradigm in fact supplied an ideological principle of selection.
On the basis of the information in the passage, all of the following are true about the spiritual/material dichotomy of Indian nationalism EXCEPT that it:
(A) constituted the premise of the ghar/bāhir dichotomy.
(B) represented a continuation of age-old oppositions in Indian culture.
(C) helped in safeguarding the identity of Indian nationalism.
(D) was not as ideologically powerful as the inner/outer dichotomy.CorrectIncorrectOption D is true, as it can be verified from the first sentence of the second paragraph. Option A is easy to eliminate as it is the very theme of the passage. The second last paragraph provides ample evidence for option C. There is no evidence for choice B. It is the right answer.
UnattemptedOption D is true, as it can be verified from the first sentence of the second paragraph. Option A is easy to eliminate as it is the very theme of the passage. The second last paragraph provides ample evidence for option C. There is no evidence for choice B. It is the right answer.
- Question 11 of 80
11. Question
DIRECTION : READ THE FOLLOWING PASSAGE AND ANSWER THE QUESTIONS THAT FOLLOW. YOUR ANSWERS TO THESE QUESTIONS SHOULD BE BASED ON THE PASSAGE ONLY.
We cannot travel outside our neighbourhood without passports. We must wear the same plainclothes. We must exchange our houses every ten years. We cannot avoid labour. We all go to bed at the same time . We have religious freedom, but we cannot deny that the soul dies with the body, since 'but for the fear of punishment, they would have nothing but contempt for the laws and customs of society'. In More's time, for much of the population, given the plenty and security on offer, such restraints would not have seemed overly unreasonable. For modern readers, however, Utopia appears to rely upon relentless transparency, the repression of variety, and the curtailment of privacy. Utopia provides security: but at what price' In both its external and internal relations, indeed, it seems perilously dystopian.
Such a conclusion might be fortified by examining selectively the tradition which follows more on these points. This often portrays societies where. . .'it would be almost impossible for man to be depraved, or wicked'. This is achieved both through institutions and mores, which underpin the common life. The passions are regulated and inequalities of wealth and distinction are minimized. Needs, vanity, and emulation are restrained, often by prizing equality and holding riches in contempt. The desire for public power is curbed. Marriage and sexual intercourse are often controlled: in Tommaso Campanella's The City of the Sun (1623), the first great literary utopia after More's, relations are forbidden to men before the age of twenty-one and women before nineteen. Communal child-rearing is normal; for Campanella this commences at age two. Greater simplicity of life, 'living according to nature', is often a result: the desire for simplicity and purity are closely related. People become more alike in appearance, opinion, and outlook than they often have been. Unity, order, and homogeneity thus prevail at the cost of individuality and diversity. This model, as J. C. Davis demonstrates, dominated early modern utopianism. And utopian homogeneity remains a familiar theme well into the twentieth century.
Given these considerations, it is not unreasonable to take as our starting point here the hypothesis that utopia and dystopia evidently share more in common than is often supposed. Indeed, they might be twins, the progeny of the same parents. Insofar as this proves to be the case, my linkage of both here will be uncomfortably close for some readers. Yet we should not mistake this argument for the assertion that all utopias are, or tend to produce, dystopias. Those who defend this proposition will find that their association here is not nearly close enough. For we have only to acknowledge the existence of thousands of successful intentional communities in which a cooperative ethos predominates and where harmony without coercion is the rule to set aside such an assertion. Here the individual's submersion in the group is consensual (though this concept is not unproblematic). It results not in enslavement but voluntary submission to group norms. Harmony is achieved without harming others.
All of the following statements can be inferred from the passage EXCEPT that:
(A) utopian and dystopian societies are twins, the progeny of the same parents.
(B) utopian societies exist in a long tradition of literature dealing with imaginary people practicing imaginary customs, in imaginary worlds.
(C) many conceptions of utopian societies emphasise the importance of social uniformity and cultural homogeneity.
(D) it is possible to see utopias as dystopias, with a change in perspective, because one person's utopia could be seen as another's dystopia.CorrectIncorrectThe answer to this question can be found in the last paragraph. The author starts by saying that “utopia and dystopia evidently share more in common…. indeed, they might be twins…”. He further adds “Yet we should not mistake this argument…” From this we can say that option (A) is definitely incorrect, and cannot be inferred. You might wonder as to the evidence for option (B). But the author mentions “More”, who was the first author of a book on Utopia, and further mentions Tommaso who also wrote a book on Utopia. We have enough evidence in the passage that shows that in literature we have enough material that have dealt with the idea of Utopia. option (C) can be inferred from the last sentence of second paragraph, and option (D) can be inferred from last sentence of first paragraph.
UnattemptedThe answer to this question can be found in the last paragraph. The author starts by saying that “utopia and dystopia evidently share more in common…. indeed, they might be twins…”. He further adds “Yet we should not mistake this argument…” From this we can say that option (A) is definitely incorrect, and cannot be inferred. You might wonder as to the evidence for option (B). But the author mentions “More”, who was the first author of a book on Utopia, and further mentions Tommaso who also wrote a book on Utopia. We have enough evidence in the passage that shows that in literature we have enough material that have dealt with the idea of Utopia. option (C) can be inferred from the last sentence of second paragraph, and option (D) can be inferred from last sentence of first paragraph.
- Question 12 of 80
12. Question
DIRECTION : READ THE FOLLOWING PASSAGE AND ANSWER THE QUESTIONS THAT FOLLOW. YOUR ANSWERS TO THESE QUESTIONS SHOULD BE BASED ON THE PASSAGE ONLY.
We cannot travel outside our neighbourhood without passports. We must wear the same plainclothes. We must exchange our houses every ten years. We cannot avoid labour. We all go to bed at the same time . We have religious freedom, but we cannot deny that the soul dies with the body, since 'but for the fear of punishment, they would have nothing but contempt for the laws and customs of society'. In More's time, for much of the population, given the plenty and security on offer, such restraints would not have seemed overly unreasonable. For modern readers, however, Utopia appears to rely upon relentless transparency, the repression of variety, and the curtailment of privacy. Utopia provides security: but at what price' In both its external and internal relations, indeed, it seems perilously dystopian.
Such a conclusion might be fortified by examining selectively the tradition which follows more on these points. This often portrays societies where. . .'it would be almost impossible for man to be depraved, or wicked'. This is achieved both through institutions and mores, which underpin the common life. The passions are regulated and inequalities of wealth and distinction are minimized. Needs, vanity, and emulation are restrained, often by prizing equality and holding riches in contempt. The desire for public power is curbed. Marriage and sexual intercourse are often controlled: in Tommaso Campanella's The City of the Sun (1623), the first great literary utopia after More's, relations are forbidden to men before the age of twenty-one and women before nineteen. Communal child-rearing is normal; for Campanella this commences at age two. Greater simplicity of life, 'living according to nature', is often a result: the desire for simplicity and purity are closely related. People become more alike in appearance, opinion, and outlook than they often have been. Unity, order, and homogeneity thus prevail at the cost of individuality and diversity. This model, as J. C. Davis demonstrates, dominated early modern utopianism. And utopian homogeneity remains a familiar theme well into the twentieth century.
Given these considerations, it is not unreasonable to take as our starting point here the hypothesis that utopia and dystopia evidently share more in common than is often supposed. Indeed, they might be twins, the progeny of the same parents. Insofar as this proves to be the case, my linkage of both here will be uncomfortably close for some readers. Yet we should not mistake this argument for the assertion that all utopias are, or tend to produce, dystopias. Those who defend this proposition will find that their association here is not nearly close enough. For we have only to acknowledge the existence of thousands of successful intentional communities in which a cooperative ethos predominates and where harmony without coercion is the rule to set aside such an assertion. Here the individual's submersion in the group is consensual (though this concept is not unproblematic). It results not in enslavement but voluntary submission to group norms. Harmony is achieved without …harming others.
Following from the passage, which one of the following may be seen as a characteristic of a utopian society?
(A) The regulation of homogeneity through promoting competitive heterogeneity.
(B) A society where public power is earned through merit rather than through privilege.
(C) Institutional surveillance of every individual to ensure his/her security and welfare.
(D) A society without any laws to restrain one's individuality.CorrectIncorrectThis is a very simple question. It can be easily answered. There is no mention of “competitive heterogeneity” in the passage. Thus (A) goes out. There is no mention of (B). (C) is true, as there is enough evidence for it in the first paragraph. (D) is the exact opposite of what utopian society wants. It wants homogeneity and uniformity, which would imply restraints on one's individuality.
UnattemptedThis is a very simple question. It can be easily answered. There is no mention of “competitive heterogeneity” in the passage. Thus (A) goes out. There is no mention of (B). (C) is true, as there is enough evidence for it in the first paragraph. (D) is the exact opposite of what utopian society wants. It wants homogeneity and uniformity, which would imply restraints on one's individuality.
- Question 13 of 80
13. Question
DIRECTION : READ THE FOLLOWING PASSAGE AND ANSWER THE QUESTIONS THAT FOLLOW. YOUR ANSWERS TO THESE QUESTIONS SHOULD BE BASED ON THE PASSAGE ONLY.
Ratatouille is a dish that has grown in popularity over the last few years. It features eggplant, zucchini, tomatoes, peppers, and garlic; chopped, mixed, sautéed, and finally, cooked slowly over low heat. As the vegetables cook slowly, they make their own broth, which may be extended with a little tomato paste. The name ratatouille comes from the French word touiller, meaning to stir or mix together.
Ratatouille can best be described as a
(A) French pastry.
(B) sauce to put over vegetables.
(C) pasta dish extended with tomato paste.
(D) vegetable stew.CorrectIncorrectThe main part of the passage describes how to cook vegetables. Only choice (D) indicates that vegetables are included in the dish. The other choices are not reflected in the passage.
UnattemptedThe main part of the passage describes how to cook vegetables. Only choice (D) indicates that vegetables are included in the dish. The other choices are not reflected in the passage.
- Question 14 of 80
14. Question
DIRECTION : READ THE FOLLOWING PASSAGE AND ANSWER THE QUESTIONS THAT FOLLOW. YOUR ANSWERS TO THESE QUESTIONS SHOULD BE BASED ON THE PASSAGE ONLY.
The competitive civil-service system is designed to give candidates fair and equal treatment and to ensure that federal applicants are hired based on objective criteria. Hiring has to be based solely on a candidate’s knowledge, skills, and abilities (which you’ll sometimes see abbreviated as ksa), and not on external factors such as race, religion, sex, and so on. Whereas employers in the private sector can hire employees for subjective reasons, federal employers must be able to justify their decision with objective evidence that the candi- date is qualified.
The federal government’s practice of hiring on the basis of ksa frequently results in the hiring of employees
(A) based on race, religion, sex, and so forth.
(B) who are unqualified for the job.
(C) who are qualified for the job.
(D) on the basis of subjective judgment.CorrectIncorrectUnattempted - Question 15 of 80
15. Question
DIRECTION : READ THE FOLLOWING PASSAGE AND ANSWER THE QUESTIONS THAT FOLLOW. YOUR ANSWERS TO THESE QUESTIONS SHOULD BE BASED ON THE PASSAGE ONLY.
An ecosystem is a group of animals and plants living in a specific region and interacting with one another and with their physical environment. Ecosystems include physical and chemical com- ponents, such as soils, water, and nutrients that support the organisms living there. These organisms may range from large animals to microscopic bacteria. Ecosystems also can be thought of as the interactions among all organisms in a given habitat; for instance, one species may serve as food for another. People are part of the ecosystems where they live and work. Human activities can harm or destroy local ecosystems unless actions such as land development for housing or businesses are carefully planned to conserve and sustain the ecology of the area. An important part of ecosystem management involves finding ways to protect and enhance economic and social well-being while protecting local ecosystems.
What is the main idea of the passage?
(A) An ecosystem is a community that includes animals, plants, and microscopic bacteria.
(B) Human activities can do great damage to local ecosystems, so human communities should be cautiously planned.
(C) In managing the ecology of an area, it is important to protect both human interests and the interests of other members of local ecosystems.
(D) People should remember that they are a part of the ecosystems where they live and work.CorrectIncorrectUnattempted - Question 16 of 80
16. Question
DIRECTION : READ THE FOLLOWING PASSAGE AND ANSWER THE QUESTIONS THAT FOLLOW. YOUR ANSWERS TO THESE QUESTIONS SHOULD BE BASED ON THE PASSAGE ONLY.
An ecosystem is a group of animals and plants living in a specific region and interacting with one another and with their physical environment. Ecosystems include physical and chemical com- ponents, such as soils, water, and nutrients that support the organisms living there. These organisms may range from large animals to microscopic bacteria. Ecosystems also can be thought of as the interactions among all organisms in a given habitat; for instance, one species may serve as food for another. People are part of the ecosystems where they live and work. Human activities can harm or destroy local ecosystems unless actions such as land development for housing or businesses are carefully planned to conserve and sustain the ecology of the area. An important part of ecosystem management involves finding ways to protect and enhance economic and social well-being while protecting local ecosystems.
Which of the following best sums up activities within an ecosystem?
(A) predator-prey relationships
(B) interactions among all members
(C) human-animal interactions
(D) human relationship with the environmentCorrectIncorrectThe passage defines an ecosystem as a community within which all members interrelate. Choice (A) is only one example of an interaction. The other two choices are too limited to sum up ecosystem activities.
UnattemptedThe passage defines an ecosystem as a community within which all members interrelate. Choice (A) is only one example of an interaction. The other two choices are too limited to sum up ecosystem activities.
- Question 17 of 80
17. Question
DIRECTION : READ THE FOLLOWING PASSAGE AND ANSWER THE QUESTIONS THAT FOLLOW. YOUR ANSWERS TO THESE QUESTIONS SHOULD BE BASED ON THE PASSAGE ONLY.
An ecosystem is a group of animals and plants living in a specific region and interacting with one another and with their physical environment. Ecosystems include physical and chemical com- ponents, such as soils, water, and nutrients that support the organisms living there. These organisms may range from large animals to microscopic bacteria. Ecosystems also can be thought of as the interactions among all organisms in a given habitat; for instance, one species may serve as food for another. People are part of the ecosystems where they live and work. Human activities can harm or destroy local ecosystems unless actions such as land development for housing or businesses are carefully planned to conserve and sustain the ecology of the area. An important part of ecosystem management involves finding ways to protect and enhance economic and social well-being while protecting local ecosystems.
An ecosystem can most accurately be defined as a :
(A) geographical area.
(B) community.
(C) habitat.
(D) protected environment.CorrectIncorrectThis is the only choice that reflects the idea of interaction among all members of the group spoken of in the first sentence. The other choices are only physical settings.
UnattemptedThis is the only choice that reflects the idea of interaction among all members of the group spoken of in the first sentence. The other choices are only physical settings.
- Question 18 of 80
18. Question
DIRECTION : READ THE FOLLOWING PASSAGE AND ANSWER THE QUESTIONS THAT FOLLOW. YOUR ANSWERS TO THESE QUESTIONS SHOULD BE BASED ON THE PASSAGE ONLY.
Daffodil bulbs require well-drained soil and a sunny planting location. They should be planted in holes that are 3–6 inches deep and there should be 2–4 inches between bulbs. The bulb should be placed in the hole, pointed side up, root side down. Once the bulb is planted, water the area thoroughly.
According to the above passage, when planting daffodil bulbs, which of the following conditions is not necessary?
(A) a sunny location
(B) well-drained soil
(C) proper placement of bulbs in soil
(D) proper fertilizationCorrectIncorrectThe passage mention nothing about fertilization.
UnattemptedThe passage mention nothing about fertilization.
- Question 19 of 80
19. Question
DIRECTION : READ THE FOLLOWING PASSAGE AND ANSWER THE QUESTIONS THAT FOLLOW. YOUR ANSWERS TO THESE QUESTIONS SHOULD BE BASED ON THE PASSAGE ONLY.
Daffodil bulbs require well-drained soil and a sunny planting location. They should be planted in holes that are 3–6 inches deep and there should be 2–4 inches between bulbs. The bulb should be placed in the hole, pointed side up, root side down. Once the bulb is planted, water the area thoroughly.
According to the above passage, which of the following is true?
(A) Daffodils do best in sandy soil.
(B) Daffodil bulbs should be planted in autumn for spring blooming.
(C) It is possible to plant daffodil bulbs upside down.
(D) Daffodil bulbs require daily watering.CorrectIncorrectThe third sentence specifically mentions that the pointed side goes up and the root side faces down. This means that there is an up side and a down side and that it is possible for the bulb to be put into the soil upside down if someone didn’t know better. The other choices may be true but are not mentioned in the passage.
UnattemptedThe third sentence specifically mentions that the pointed side goes up and the root side faces down. This means that there is an up side and a down side and that it is possible for the bulb to be put into the soil upside down if someone didn’t know better. The other choices may be true but are not mentioned in the passage.
- Question 20 of 80
20. Question
DIRECTION : READ THE FOLLOWING PASSAGE AND ANSWER THE QUESTIONS THAT FOLLOW. YOUR ANSWERS TO THESE QUESTIONS SHOULD BE BASED ON THE PASSAGE ONLY.
Many cities haves distributed standardized recycling containers to all households with directions that read: “We would prefer that you use this new container as your primary recycling container as this will expedite pick-up of recyclables. Additional recycling containers may be purchased from the City.”
According to the passage, each household :
(A) may only use one recycling container.
(B) must use the new recycling container.
(C) should use the new recycling container.
(D) must buy a new recycling container.CorrectIncorrectThe passage indicate that the city prefers, but does not require, use of its new container, and that the customers may use more than one container if they purchase an additional one.
UnattemptedThe passage indicate that the city prefers, but does not require, use of its new container, and that the customers may use more than one container if they purchase an additional one.
- Question 21 of 80
21. Question
Directions : Examine information given below and answer the questions that follow:
Five executives A, B, C, D & E of Indian Corporation hold a conference in New Delhi.
Mr. A converses in Hindi and Tamil.
Mr. B converses in Hindi and English.
Mr. C converses in English and Tamil.
Mr. D converses in Bengali and Hindi.
Mr. E, a native Tamil, can also converse in Bengali.
Which of the following can act as an interpreter when Mr. C and Mr. D wish to converse?
(A) Only Mr. A
(B) Only Mr. B
(C) Only Mr. E
(D) Any of the other three executivesCorrectIncorrectWhen Mr. C and Mr. D converse, they can use English, Tamil, Bengali and Hindi interpreter between them. Mr. A speaks Hindi and Tamil and Mr. B speaks English and Hindi. Mr. E speaks bengali and Tamil.
UnattemptedWhen Mr. C and Mr. D converse, they can use English, Tamil, Bengali and Hindi interpreter between them. Mr. A speaks Hindi and Tamil and Mr. B speaks English and Hindi. Mr. E speaks bengali and Tamil.
- Question 22 of 80
22. Question
Directions : Examine information given below and answer the questions that follow:
Five executives A, B, C, D & E of Indian Corporation hold a conference in New Delhi.
Mr. A converses in Hindi and Tamil.
Mr. B converses in Hindi and English.
Mr. C converses in English and Tamil.
Mr. D converses in Bengali and Hindi.
Mr. E, a native Tamil, can also converse in Bengali.
Which of the following pairs cannot converse without an interpreter?
(A) Mr. B and Mr. E
(B) Mr. A and Mr. B
(C) Mr. A and Mr. C
(D) Mr. B and Mr. DCorrectIncorrectMr. B understands English and Hindi, while Mr. E speaks two other languages, Bengali and Tamil.
UnattemptedMr. B understands English and Hindi, while Mr. E speaks two other languages, Bengali and Tamil.
- Question 23 of 80
23. Question
An Indian who is a scientist as well as a politician is represented in the following diagram by an alphabet. Find the alphabet and choose the correct response.
(A) b
(B) a
(C) c
(D) gCorrectIncorrectSince, the region ‘a’ is common between Indian scientists and politicians, therefore it represents Indian scientists who are also politicians.
UnattemptedSince, the region ‘a’ is common between Indian scientists and politicians, therefore it represents Indian scientists who are also politicians.
- Question 24 of 80
24. Question
Directions : Examine the following information carefully and answer the questions that follow:
Nine cities A, B, C, D, E, F, G, H and I are connected with a oneway or a two way route.
The oneway routes are from A to F; D to A; D to F; H to D; D to B; D to E; E to B; B to G; E to F; E to C.
The two way routes are between G and H; H and A; A and I; I and C; F and C.
No other routes exist except the above mentioned routes.
Due to Bandh call given by a political party, no one is allowed to pass through city D. Then, which city can not be reached from any other city?
(A) G
(B) B
(C) E
(D) FCorrectIncorrectUnattempted - Question 25 of 80
25. Question
Directions : Study the following graph and answer the given question. Graph shows the variation in literacy per cent with the increase in the population of four different districts of a newly formulated state.
Which district shows two different behaviour of literacy per cent with the increase in population?
(A) A
(B) B
(C) C
(D) DCorrectIncorrectDistrict A & B are showing a fixed behaviour, while C is showing 3 distinct behaviours but District D shows two different types of behaviour because firstly, the graph is increasing proportionally and then constant throughout the increasing population.
UnattemptedDistrict A & B are showing a fixed behaviour, while C is showing 3 distinct behaviours but District D shows two different types of behaviour because firstly, the graph is increasing proportionally and then constant throughout the increasing population.
- Question 26 of 80
26. Question
In a row of boys facing West, K is twelfth from the left end and fourth to the right of L. What is the position of L from the left ends of that row?
(A) 8th
(B) 9th
(C) 7th
(D) 4thCorrectIncorrectFrom the left ends the position of L is = 12th – 4th = 8th
UnattemptedFrom the left ends the position of L is = 12th – 4th = 8th
- Question 27 of 80
27. Question
In a row of boys there are 50 boys and all of them are facing North, Sahaj is 19th from the left end and fourth to the right of Mahak, what is the position of Mahak from the right ends of that row?
(A) 28th
(B) 36th
(C) 37th
(D) 25thCorrectIncorrectFrom the left ends the position of Mahak is = 19th – 4th = 15th
Hence, the position of Mahak from the right end is
= Number of Boys – position of Mahak from the left ends + 1
= 50 – 15 + 1 = 36thUnattemptedFrom the left ends the position of Mahak is = 19th – 4th = 15th
Hence, the position of Mahak from the right end is
= Number of Boys – position of Mahak from the left ends + 1
= 50 – 15 + 1 = 36th - Question 28 of 80
28. Question
In a row of Children facing north, K is 15th to the left of H, who is 22nd from the right end. If P is 14th from the left end and 6th to the right of K, how many children are there in that row?
(A) 41
(B) 38
(C) 42
(D) 44CorrectIncorrectHere, P is 14th from the left end and 6th to the right of K.
Hence, K is 8th from the left side.
Again, K is 15th to the left of H, who is 22nd from the right end.
Hence, K is 37th from the right side.
Therefore, the total number of Children is
= K’s position from the right ends + K’s position from the left ends – 1
= 37 + 8 – 1 = 45 – 1 = 44UnattemptedHere, P is 14th from the left end and 6th to the right of K.
Hence, K is 8th from the left side.
Again, K is 15th to the left of H, who is 22nd from the right end.
Hence, K is 37th from the right side.
Therefore, the total number of Children is
= K’s position from the right ends + K’s position from the left ends – 1
= 37 + 8 – 1 = 45 – 1 = 44 - Question 29 of 80
29. Question
Direction: Read the following information carefully to answer the questions given below.
Devesh, Gautam, Aditya, Byom, Chandan, Harish, Fahad, and Emi are eight persons, having a different height. Devesh is shorter than Aditya, but taller than Gautam. Emi is taller than Harish but shorter than Chandan. Byom is shorter than Devesh but taller than Fahad. Chandan is shorter than Gautam. Gautam is not as tall as Fahad.
Which of the following is 5th shortest?
(A) Byom
(B) Chandan
(C) Fahad
(D) GautamCorrectIncorrectAditya > Devesh > Byom > Fahad > Gautam > Chandan > Emi > Harish
UnattemptedAditya > Devesh > Byom > Fahad > Gautam > Chandan > Emi > Harish
- Question 30 of 80
30. Question
Direction: Read the following information carefully to answer the questions given below.
Devesh, Gautam, Aditya, Byom, Chandan, Harish, Fahad, and Emi are eight persons, having a different height. Devesh is shorter than Aditya, but taller than Gautam. Emi is taller than Harish but shorter than Chandan. Byom is shorter than Devesh but taller than Fahad. Chandan is shorter than Gautam. Gautam is not as tall as Fahad.
Who among the following is shortest among them?
(A) Byom
(B) Chandan
(C) Fahad
(D) HarishCorrectIncorrectAditya > Devesh > Byom > Fahad > Gautam > Chandan > Emi > Harish
UnattemptedAditya > Devesh > Byom > Fahad > Gautam > Chandan > Emi > Harish
- Question 31 of 80
31. Question
Direction : Read the following information carefully to answer the questions given below.
There are six Investors named Kavita, Laxman, Manish, Niraj, Om and Parvati, purchased different number of shares in HUL Company. No two person purchased the same number of shares. According to the number of purchased share, three people among them invested between Om and Kavita where Om invested more than Kavita. Parvati who purchased 37 shares, purchased less than Niraj. Manish purchased 34 shares which is less than Laxman. None of them purchased between the number of shares Manish and Laxman purchased. Om did not purchase maximum share among them.
Who purchased the maximum number of shares?
(A) Manish
(B) Kavita
(C) Parvati
(D) NirajCorrectIncorrectNiraj > Om > Parvati (37) > Laxman > Manish (34) > Kavita
Hence, Niraj invested highest among them.UnattemptedNiraj > Om > Parvati (37) > Laxman > Manish (34) > Kavita
Hence, Niraj invested highest among them. - Question 32 of 80
32. Question
Direction : Read the following information carefully to answer the questions given below.
There are six Investors named Kavita, Laxman, Manish, Niraj, Om and Parvati, purchased different number of shares in HUL Company. No two person purchased the same number of shares. According to the number of purchased share, three people among them invested between Om and Kavita where Om invested more than Kavita. Parvati who purchased 37 shares, purchased less than Niraj. Manish purchased 34 shares which is less than Laxman. None of them purchased between the number of shares Manish and Laxman purchased. Om did not purchase maximum share among them.
Who among the following person has purchased third highest number of shares?
(A) Niraj
(B) Parvati
(C) Manish
(D) OmCorrectIncorrectNiraj > Om > Parvati (37) > Laxman > Manish (34) > Kavita
Hence, Parvati purchased third highest no of shares among them.UnattemptedNiraj > Om > Parvati (37) > Laxman > Manish (34) > Kavita
Hence, Parvati purchased third highest no of shares among them. - Question 33 of 80
33. Question
Find the missing number in series marked by “?”
2 , 3 , 8 , 63, ?
(A) 3968
(B) 256
(C) 3863
(D) 175CorrectIncorrectThe pattern is,
22 = 4 – 1 = 3
32 = 9 – 1 = 8
82 = 64 – 1 = 63
632 = 3969 – 1 = 3968
3968 is the missing number.UnattemptedThe pattern is,
22 = 4 – 1 = 3
32 = 9 – 1 = 8
82 = 64 – 1 = 63
632 = 3969 – 1 = 3968
3968 is the missing number. - Question 34 of 80
34. Question
Find the missing number in series marked by “?”
3, 7, 16, 35, “?”, 153
(A) 72
(B) 64
(C) 134
(D) 74CorrectIncorrectThe pattern is,
3*2 + 1 = 7
7*2 + 2 = 16
16*2 + 3 = 35
35*2 + 4 = 74
74*2 + 5 = 153
74 is the missing numberUnattemptedThe pattern is,
3*2 + 1 = 7
7*2 + 2 = 16
16*2 + 3 = 35
35*2 + 4 = 74
74*2 + 5 = 153
74 is the missing number - Question 35 of 80
35. Question
Consider the following students in an examination :
A scored more than B
C scored as much as D
E scored less than F
B scored more than C
F secored less than D
Who got lowest score ?
(A) E
(B) C
(C) D
(D) FCorrectIncorrectA > B, C = D, E < F, B > C, F < D
(1) (3) (5) (2) (4)
A > B > C = D > F > E.
So, ‘E’ got lowest score.UnattemptedA > B, C = D, E < F, B > C, F < D
(1) (3) (5) (2) (4)
A > B > C = D > F > E.
So, ‘E’ got lowest score. - Question 36 of 80
36. Question
Directions : Examine the information given below and answer the questions that follow:
Kush, Ganesh and Hari are intelligent.
Kush, Ram and Jivan are hardworking.
Ram, Hari and Jivan are honest.
Kush, Ganesh and Jivan are ambitious.
Which of the following persons is neither hardworking nor ambitious?
(A) Kush
(B) Ganesh
(C) Hari
(D) RamCorrectIncorrectHari is neither hardworking nor ambitious.
UnattemptedHari is neither hardworking nor ambitious.
- Question 37 of 80
37. Question
A cube of 3 cm side is divided into smaller cubes of side 1 cm. How many times will it be cut to obtain smaller cubes?
(A) 8
(B) 4
(C) 6
(D) 9CorrectIncorrectUnattempted - Question 38 of 80
38. Question
Babu said to Sanjay, “That boy playing with football is younger of the two brothers of the daughter of my father's wife.” How is the boy playing football related to Babu?
(A) Uncle
(B) Elder brother
(C) Younger brother
(D) CousinCorrectIncorrectYounger of the two brothers of the daughter of my father’s wife (as my father’s wife = my mother)
So, younger of the two brothers of the daughter of my mother (as daughter of my mother = my sister). So, younger of the two brothers of my sister means ‘my younger brother’. Hence, that boy is the younger brother of Babu.UnattemptedYounger of the two brothers of the daughter of my father’s wife (as my father’s wife = my mother)
So, younger of the two brothers of the daughter of my mother (as daughter of my mother = my sister). So, younger of the two brothers of my sister means ‘my younger brother’. Hence, that boy is the younger brother of Babu. - Question 39 of 80
39. Question
The question below consists of a question and two statements numbered I and II given below it. You have to decide whether the data provided in the statements are sufficient to answer the question.
How far did Mohan walk from the starting point?
1. Mohan walked 20 metres towards West, took a right turn and walked 30 metres, again took a right turn and walked 20 metres.
2. Mohan walked 20 metres towards South, took a left turn and walked 30 metres, again took a left turn and walked 20 metres.
Read both the statements and give answer :
(A) The data in Statement I alone are sufficient to answer the question, while the data in Statement II alone are not sufficient to answer the question.
(B) The data in Statement II alone are sufficient to answer the question, while the data in Statement I alone are not sufficient to answer the question.
(C) The data either in Statement I alone or in Statement II alone are sufficient to answer the question.
(D) The data given in both the Statements I and II together are not sufficient to answer the question.CorrectIncorrectUnattempted - Question 40 of 80
40. Question
Directions : Study the following information carefully and answer the questions given below:
Seven persons A, B, C, D, E, F and G are sitting in a circle and are facing the circle.
G is third to the left of A.
F is third to the left of B.
D is not the neighbour of G.
C is the neighbour of A.
B is to the immediate left of A.
What is the correct sitting position of E?
(A) To the immediate right of B
(B) Between G and F
(C) Third to the right of C
(D) To the immediate right of GCorrectIncorrectUnattempted - Question 41 of 80
41. Question
Find the number of permutations of the letters of the word ALLAHABAD.
(A) 7460
(B) 7560
(C) 7566
(D) 7569CorrectIncorrectGiven word – ALLAHABAD
Here, there are 9 objects (letters) of which there are 4As, 2 Ls and rest are all different.
Therefore, the required number of arrangements = 9!/(4! 2!)
= (1 × 2 × 3 × 4 × 5 × 6 × 7 × 8 × 9)/ (1 × 2 × 3 × 4 × 1 × 2)
= (5 × 6 × 7 × 8 × 9)/2 = 7560UnattemptedGiven word – ALLAHABAD
Here, there are 9 objects (letters) of which there are 4As, 2 Ls and rest are all different.
Therefore, the required number of arrangements = 9!/(4! 2!)
= (1 × 2 × 3 × 4 × 5 × 6 × 7 × 8 × 9)/ (1 × 2 × 3 × 4 × 1 × 2)
= (5 × 6 × 7 × 8 × 9)/2 = 7560 - Question 42 of 80
42. Question
Find the next term in the series.
294, 448, 648, 900, 1210, ?
(A) 1520
(B) 1480
(C) 1495
(D) 1584CorrectIncorrectThe series is as follows:
72 x 6 = 294
82 x 7 = 448
92 x 8 = 648
102 x 9 = 900
112 x 10 = 1210
122 x 11 = 1584UnattemptedThe series is as follows:
72 x 6 = 294
82 x 7 = 448
92 x 8 = 648
102 x 9 = 900
112 x 10 = 1210
122 x 11 = 1584 - Question 43 of 80
43. Question
If today is Friday, then what will be the day after 363 days?
(A) Sunday
(B) Saturday
(C) Thursday
(D) None of theseCorrectIncorrectToday is Friday.
Each day gets repeated after 7 days.
So, on dividing 363 by 7, we get remainder = 6
1 – Saturday
2 – Sunday
3 – Monday
4 – Tuesday
5 – Wednesday
6 – Thursday
Hence, option C is the correct answer.UnattemptedToday is Friday.
Each day gets repeated after 7 days.
So, on dividing 363 by 7, we get remainder = 6
1 – Saturday
2 – Sunday
3 – Monday
4 – Tuesday
5 – Wednesday
6 – Thursday
Hence, option C is the correct answer. - Question 44 of 80
44. Question
If the day after the 11th of March is Thursday, then what will be the last day of the month?
(A) Thursday
(B) Wednesday
(C) Friday
(D) TuesdayCorrectIncorrectThe day after the 11th of March is Thursday means on 12th of March is Thursday.
March has 31 days.
19th march is Thursday then 26th March is Thursday.
26 + 5 = 31st March.
Thursday + 5 = Tuesday.
Hence, option D is the correct answer.UnattemptedThe day after the 11th of March is Thursday means on 12th of March is Thursday.
March has 31 days.
19th march is Thursday then 26th March is Thursday.
26 + 5 = 31st March.
Thursday + 5 = Tuesday.
Hence, option D is the correct answer. - Question 45 of 80
45. Question
If the day before yesterday was Monday, then what day will it be the day after tomorrow?
(A) Friday
(B) Thursday
(C) Wednesday
(D) TuesdayCorrectIncorrectThe day before yesterday is Monday,
Yesterday is Tuesday,
Today is Wednesday
Tomorrow is Thursday
The day after tomorrow is Friday.
Hence, option A is the correct answer.UnattemptedThe day before yesterday is Monday,
Yesterday is Tuesday,
Today is Wednesday
Tomorrow is Thursday
The day after tomorrow is Friday.
Hence, option A is the correct answer. - Question 46 of 80
46. Question
Republic Day of India was celebrated on Thursday in 2017. On which day it was celebrated in 2021?
(A) Tuesday
(B) Wednesday
(C) Thursday
(D) FridayCorrectIncorrectWe know that in 365 days we have 1 odd day.
26 January 2017 = Thursday
26 January 2018 = Friday
26 January 2019 = Saturday
26 January 2020 = Sunday.
2020 is a leap year so February has 29 days.
26 January 2021 = Sunday + 2 = Tuesday (+2 because of leap year, In 366 days we have 2 odd days)UnattemptedWe know that in 365 days we have 1 odd day.
26 January 2017 = Thursday
26 January 2018 = Friday
26 January 2019 = Saturday
26 January 2020 = Sunday.
2020 is a leap year so February has 29 days.
26 January 2021 = Sunday + 2 = Tuesday (+2 because of leap year, In 366 days we have 2 odd days) - Question 47 of 80
47. Question
Answer the question based on the figure given below in which Rectangle represents Males, Circle represents the urbans, Square represents the educated and Triangle represents the civil servants.
The number indicating the educated civil servants who are males but not urban is
(A) 7
(B) 10
(C) 8
(D) 4CorrectIncorrectEducated civil servant Males: 10 and 8
Non urban: 10UnattemptedEducated civil servant Males: 10 and 8
Non urban: 10 - Question 48 of 80
48. Question
Excluding stoppages, the speed of a bus is 54 kmph and including stoppages, it is 45 kmph. For how many minutes does the bus stop per hour?
(A) 5 min
(B) 10 min
(C) 4 min
(D) 9 minCorrectIncorrectSpeed without stoppages = 54km/hr
Speed with stoppages = 45km/hr
Stoppages per hour =
UnattemptedSpeed without stoppages = 54km/hr
Speed with stoppages = 45km/hr
Stoppages per hour =
- Question 49 of 80
49. Question
DIRECTION : READ THE FOLLOWING PASSAGE AND ANSWER THE QUESTION THAT FOLLOW. YOUR ANSWER TO THIS QUESTION SHOULD BE BASED ON THE PASSAGE ONLY.
The most important reason for this state of affairs, perhaps, is that India was the only country in the world to truly recognise the achievements of the Soviet Union-rather than merely focus on the debilitating faults that Communism brought to its people. The people of India realised that the achievement of one hundred percent literacy in a country much, much larger than its own and with similarly complicated ethnic and religious groupings, the rapid industrialization of a nation that was a primarily agrarian society when the Bolshevik revolution took place in 1917, the attendant revolutionary steps in science and technology, the accessibility of health care (primeval according to Western standards, perhaps, but not according to Indian ones) to the general population, and despite prohibition of the government of the time the vast outpourings in literature, music, art, etc. are momentous and remarkable feats in any country. In contrast, all that the West focused on were the massive human rights violations by the Soviet State on its people, the deliberate uprooting and mass migrations of ethnic peoples from one part of the country to another in the name of industrialization, the end of religion. In short, all the tools of information were employed to condemn the ideology of Communism, so much at variance with capitalist thinking. The difference with the Indian perception, I think here is, that while the Indians reacted as negatively to what the Soviet governments did to its people in the name of good governance (witness the imprisonment of Boris Pasternak and the formation of an international committee to put pressure for his release with Jawaharlal Nehru at its head), they took the pain not to condemn the people of that broad country in black and white terms; they understood that mingled in the shades of grey were grains of uniqueness (The Russians have never failed that characteristic in themselves; they have twice experimented with completely different ideologies, Communism and Capitalism both in the space of a century).
Which of the following statements according to the passage is correct?
(A) India took heed of the weak faults of Russian policies and systems.
(B) The process of industrialisation had already started when the Russian Revolution took place in 1917.
(C) India seriously commended the achievements of Russia, i.e., hundred percent literacy and rapid industrialisation.
(D) Literature, art and music received a setback during the communist regime in Russia.CorrectIncorrectRefer to these lines, “The people of India realized that the achievement of one hundred percent literacy in a country much, much larger than its own and with similarly complicated ethnic and religious groupings, the rapid industrialization of a nation that was a primarily agrarian society when the Bolshevik revolution took place in 1917, the attendant revolutionary steps in science and technology, the accessibility of health care (primeval according to Western standards, perhaps, but not according to Indian ones) to the general population, and despite the prohibition of the government of the time the vast outpourings in literature, music, art, etc. are momentous and remarkable feats in any country“
India appreciated the achievement of Russia in the field of literacy and rapid industrializationUnattemptedRefer to these lines, “The people of India realized that the achievement of one hundred percent literacy in a country much, much larger than its own and with similarly complicated ethnic and religious groupings, the rapid industrialization of a nation that was a primarily agrarian society when the Bolshevik revolution took place in 1917, the attendant revolutionary steps in science and technology, the accessibility of health care (primeval according to Western standards, perhaps, but not according to Indian ones) to the general population, and despite the prohibition of the government of the time the vast outpourings in literature, music, art, etc. are momentous and remarkable feats in any country“
India appreciated the achievement of Russia in the field of literacy and rapid industrialization - Question 50 of 80
50. Question
Answer the question based on the figure given below in which Rectangle represents Males, Circle represents the urbans, Square represents the educated and Triangle represents the civil servants.
The number indicating the educated males who are urban civil servants is
a.5
(B) 4
(C) 11
(D) 8CorrectIncorrectEducated male urban civil servants: 8
UnattemptedEducated male urban civil servants: 8
- Question 51 of 80
51. Question
DIRECTION : READ THE FOLLOWING PASSAGE AND ANSWER THE QUESTION THAT FOLLOW. YOUR ANSWER TO THIS QUESTION SHOULD BE BASED ON THE PASSAGE ONLY.
The most important reason for this state of affairs, perhaps, is that India was the only country in the world to truly recognise the achievements of the Soviet Union-rather than merely focus on the debilitating faults that Communism brought to its people. The people of India realised that the achievement of one hundred percent literacy in a country much, much larger than its own and with similarly complicated ethnic and religious groupings, the rapid industrialization of a nation that was a primarily agrarian society when the Bolshevik revolution took place in 1917, the attendant revolutionary steps in science and technology, the accessibility of health care (primeval according to Western standards, perhaps, but not according to Indian ones) to the general population, and despite prohibition of the government of the time the vast outpourings in literature, music, art, etc. are momentous and remarkable feats in any country. In contrast, all that the West focused on were the massive human rights violations by the Soviet State on its people, the deliberate uprooting and mass migrations of ethnic peoples from one part of the country to another in the name of industrialization, the end of religion. In short, all the tools of information were employed to condemn the ideology of Communism, so much at variance with capitalist thinking. The difference with the Indian perception, I think here is, that while the Indians reacted as negatively to what the Soviet governments did to its people in the name of good governance (witness the imprisonment of Boris Pasternak and the formation of an international committee to put pressure for his release with Jawaharlal Nehru at its head), they took the pain not to condemn the people of that broad country in black and white terms; they understood that mingled in the shades of grey were grains of uniqueness (The Russians have never failed that characteristic in themselves; they have twice experimented with completely different ideologies, Communism and Capitalism both in the space of a century).
The Indian perception of the Soviet Union was always
(A) Neutral
(B) Applauding
(C) Counter-reactionary
(D) CriticalCorrectIncorrectAs indicated in the passage India applauded the achievements of Russia.
UnattemptedAs indicated in the passage India applauded the achievements of Russia.
- Question 52 of 80
52. Question
DIRECTION : READ THE FOLLOWING PASSAGE AND ANSWER THE QUESTION THAT FOLLOW. YOUR ANSWER TO THIS QUESTION SHOULD BE BASED ON THE PASSAGE ONLY.
The most important reason for this state of affairs, perhaps, is that India was the only country in the world to truly recognise the achievements of the Soviet Union-rather than merely focus on the debilitating faults that Communism brought to its people. The people of India realised that the achievement of one hundred percent literacy in a country much, much larger than its own and with similarly complicated ethnic and religious groupings, the rapid industrialization of a nation that was a primarily agrarian society when the Bolshevik revolution took place in 1917, the attendant revolutionary steps in science and technology, the accessibility of health care (primeval according to Western standards, perhaps, but not according to Indian ones) to the general population, and despite prohibition of the government of the time the vast outpourings in literature, music, art, etc. are momentous and remarkable feats in any country. In contrast, all that the West focused on were the massive human rights violations by the Soviet State on its people, the deliberate uprooting and mass migrations of ethnic peoples from one part of the country to another in the name of industrialization, the end of religion. In short, all the tools of information were employed to condemn the ideology of Communism, so much at variance with capitalist thinking. The difference with the Indian perception, I think here is, that while the Indians reacted as negatively to what the Soviet governments did to its people in the name of good governance (witness the imprisonment of Boris Pasternak and the formation of an international committee to put pressure for his release with Jawaharlal Nehru at its head), they took the pain not to condemn the people of that broad country in black and white terms; they understood that mingled in the shades of grey were grains of uniqueness (The Russians have never failed that characteristic in themselves; they have twice experimented with completely different ideologies, Communism and Capitalism both in the space of a century).
The West did not focus on
(A) Massive human rights violation of the Soviet state on its people.
(B) Rapid growth of nuclear weapons in Russia.
(C) Deliberate uprooting and mass migration of ethnic people in the name of industrialisation.
(D) Both (A) and (C).CorrectIncorrectRefer to these lines, “In contrast, all that the West focused on were the massive human rights violations by the Soviet State on its people, the deliberate uprooting and mass migrations of ethnic peoples from one part of the country to another in the name of industrialization, the end of religion.”
All that west focused on is mentioned in options A and C but not B.UnattemptedRefer to these lines, “In contrast, all that the West focused on were the massive human rights violations by the Soviet State on its people, the deliberate uprooting and mass migrations of ethnic peoples from one part of the country to another in the name of industrialization, the end of religion.”
All that west focused on is mentioned in options A and C but not B. - Question 53 of 80
53. Question
A man wants to reach a window which is 40 feet above the ground. The distance from the foot of the ladder to the wall is 9 feet. How long should the ladder be?
(A) 81 feet
(B) 40 feet
(C) 41 feet
(D) 54 feetCorrectIncorrect
By Pythagoras theorem,
Length of ladder =Unattempted
By Pythagoras theorem,
Length of ladder = - Question 54 of 80
54. Question
If A is to the south of B and C is to the east of B, what is the direction of A with respect to C?
(A) North–east
(B) South–east
(C) North–west
(D) South–westCorrectIncorrectFrom the figure given below, A is south–west of C.
UnattemptedFrom the figure given below, A is south–west of C.
- Question 55 of 80
55. Question
DIRECTION : READ THE FOLLOWING PASSAGE AND ANSWER THE QUESTION THAT FOLLOW. YOUR ANSWER TO THIS QUESTION SHOULD BE BASED ON THE PASSAGE ONLY.
For the Maya of the Classic period, who lived in Southern Mexico and Central America between 250 and 900 CE, the category of “persons” was not coincident with human beings, as it is for us. That is, human beings were persons – but other, nonhuman entities could be persons, too. In order to explore the slippage of categories between “humans” and “persons”, I examined a very specific category of ancient Maya images, found painted in scenes on ceramic vessels. I sought out instances in which faces (some combination of eyes, nose, and mouth) are shown on inanimate objects. Consider my iPhone, which needs to be fed with electricity every night, swaddled in a protective bumper, and enjoys communicating with other fellow-phone-beings. Does it have personhood (if at all) because itis connected to me, drawing this resource from me as an owner or source? For the Maya (who did have plenty of other communicating objects, if not smartphones), the answer was no. Nonhuman persons were not tethered to specific humans, and they did not derive their personhood from a connection with a human. It's a profoundly democratising way of understanding the world. Humans are not more important persons – we are just one of many kinds of persons who inhabit this world.
The Maya saw personhood as 'activated' by experiencing certain bodily needs and through participation in certain social activities. For example, among the faced objects that I examined, persons are marked by personal requirements (such as hunger, tiredness, physical closeness), and by community obligations (communication, interaction, ritual observance). In the images I examined, we see, for instance, faced objects being cradled in humans' arms; we also see them speaking to humans. These core elements of personhood are both turned inward, what the body or self of a person requires, and outward, what a community expects of the persons who are a part of it, underlining the reciprocal nature of community membership.
Personhood was a nonbinary proposition for the Maya. Entities were able to be persons while also being something else. The faced objects I looked at indicate that they continue to be functional, doing what objects do (a stone implement continues to chop, an incense burner continues to do its smoky work). Furthermore, the Maya visually depicted many objects in ways that indicated the material category to which they belonged – drawings of the stone implement show that a person-tool is still made of stone. One additional complexity: the incense burner (which would have been made of clay, and decorated with spiky appliques representing the sacred ceiba tree found in this region) is categorised as a person – but also as a tree. With these Maya examples, we are challenged to discard the person/nonperson binary that constitutes our basic ontological outlook. The porousness of boundaries that we have seen in the Maya world points towards the possibility of living with a certain uncategorisability of the world.
Which one of the following, if true about the Classic Maya, would invalidate the purpose of the iPhone example in the passage?
(A) The clay incense burner with spiky appliques was categorised only as a person and not as a tree by the Classic Maya.
(B) Classic Maya songs represent both humans and non-living objects as characters, talking and interacting with each other.
(C) The personhood of the incense burner and the stone chopper was a function of their usefulness to humans.
(D) Unlike modern societies equipped with mobile phones, the Classic Maya did not have any communicating objects.CorrectIncorrectTo answer this question we have to understand the iPhone example. The author says that to us iPhone has a personhood because it is connected to or useful to me, but this was not the case with Maya people. To them nonhuman persons were not tethered to specific humans. To invalidate this example, we have to pick a choice that goes against this. Option C precisely does that. It makes the personhood of the incense burner and the stone chopper a function of their usefulness to humans, something that the author wants to deny through the example of iPhone. Thus if C is true than the purpose of the iPhone example is invalidated. All the other three choices don't invalidate the iPhone example in any way.
UnattemptedTo answer this question we have to understand the iPhone example. The author says that to us iPhone has a personhood because it is connected to or useful to me, but this was not the case with Maya people. To them nonhuman persons were not tethered to specific humans. To invalidate this example, we have to pick a choice that goes against this. Option C precisely does that. It makes the personhood of the incense burner and the stone chopper a function of their usefulness to humans, something that the author wants to deny through the example of iPhone. Thus if C is true than the purpose of the iPhone example is invalidated. All the other three choices don't invalidate the iPhone example in any way.
- Question 56 of 80
56. Question
Find the next term in the series.
28, 44, -81, -45, -388, ?
(A) -342
(B) -314
(C) -373
(D) -324CorrectIncorrectThe series is as follows:
28 + 42 = 44
44 – 53 = -81
-81 + 62 = -45
-45 – 73 = -388
-388 + 82 = -324UnattemptedThe series is as follows:
28 + 42 = 44
44 – 53 = -81
-81 + 62 = -45
-45 – 73 = -388
-388 + 82 = -324 - Question 57 of 80
57. Question
P is facing the north direction. He then turns right and walks 20 m. He then turns to his left and walks 20 m. Next, he moves 20 m to his right. He then turns to his right again and walks 40 m. Finally, he turns to his right and moves 30 m. In which direction is he now with respect to his starting point?
(A) South–west
(B) South
(C) North–west
(D) South–eastCorrectIncorrectLet us assume that P starts from point A.
He turns right and walks 20 m towards east up to point B, turns left and moves 20 m up to point C, turns right and moves 20 m up to point D.
At D where he is facing east, he takes a right turn and turns towards south and walks 40 m up to E.
Next, he turns right again and walks 30 m up to F, his final position. F is south–east of A.
Therefore, P is south–east of his starting point.
UnattemptedLet us assume that P starts from point A.
He turns right and walks 20 m towards east up to point B, turns left and moves 20 m up to point C, turns right and moves 20 m up to point D.
At D where he is facing east, he takes a right turn and turns towards south and walks 40 m up to E.
Next, he turns right again and walks 30 m up to F, his final position. F is south–east of A.
Therefore, P is south–east of his starting point.
- Question 58 of 80
58. Question
How many numbers are there between 99 and 1000, having at least one of their digits 7?
(A) 251
(B) 252
(C) 253
(D) 254CorrectIncorrectNumbers between 99 and 1000 are all three-digit numbers.
Total number of 3 digit numbers having at least one of their digits as 7
= (Total numbers of three-digit numbers) – (Total number of 3 digit numbers in which 7 does not appear at all)
= (9 × 10 × 10) – (8 × 9 × 9)
= 900 – 648 = 252UnattemptedNumbers between 99 and 1000 are all three-digit numbers.
Total number of 3 digit numbers having at least one of their digits as 7
= (Total numbers of three-digit numbers) – (Total number of 3 digit numbers in which 7 does not appear at all)
= (9 × 10 × 10) – (8 × 9 × 9)
= 900 – 648 = 252 - Question 59 of 80
59. Question
How many 5-digit telephone numbers can be constructed using the digits 0 to 9, if each number starts with 67 and no digit appears more than once?
(A) 332
(B) 335
(C) 336
(D) 339CorrectIncorrectLet ABCDE be a five-digit number.
Given that the first two digits of each number are 6 and 7.
Therefore, the number is 67CDE.
As repetition is not allowed and 6 and 7 are already taken, the digits available for place C are 0, 1, 2, 3, 4, 5, 8, 9, i.e. eight possible digits.
Suppose one of them is taken at C, now the digits possible at place D is 7.
Similarly, at E, the possible digit is 6.
Therefore, the total five-digit numbers with given conditions = 8 × 7 × 6 = 336.UnattemptedLet ABCDE be a five-digit number.
Given that the first two digits of each number are 6 and 7.
Therefore, the number is 67CDE.
As repetition is not allowed and 6 and 7 are already taken, the digits available for place C are 0, 1, 2, 3, 4, 5, 8, 9, i.e. eight possible digits.
Suppose one of them is taken at C, now the digits possible at place D is 7.
Similarly, at E, the possible digit is 6.
Therefore, the total five-digit numbers with given conditions = 8 × 7 × 6 = 336. - Question 60 of 80
60. Question
Find the next term in the series.
632, 582, 534, 488, 444, ?
(A) 417
(B) 402
(C) 410
(D) 408CorrectIncorrectThe pattern is as follows:
252 + 7 = 632
242 + 6 = 582
232 + 5 = 534
222 + 4 = 488
212 + 3 = 444
202 + 2 = 402UnattemptedThe pattern is as follows:
252 + 7 = 632
242 + 6 = 582
232 + 5 = 534
222 + 4 = 488
212 + 3 = 444
202 + 2 = 402 - Question 61 of 80
61. Question
If a * b = 2 (a + b), then 5 * 2 is equal to :
(A) 3
(B) 10
(C) 14
(D) 20CorrectIncorrecta * b = 2 (a + b)
So, 5 * 2 = 2 (5 + 2)
= 2 × 7 = 14Unattempteda * b = 2 (a + b)
So, 5 * 2 = 2 (5 + 2)
= 2 × 7 = 14 - Question 62 of 80
62. Question
In how many of the distinct permutations of the letters in MISSISSIPPI do the four Is not come together?
(A) 33810
(B) 33812
(C) 33815
(D) 33862CorrectIncorrectGiven word – MISSISSIPPI
M – 1
I – 4
S – 4
P – 2
Number of permutations = 11!/(4! 4! 2!)
= (11 × 10 × 9 × 8 × 7 × 6 × 5 × 4!)/ (4! × 24 × 2) = 34650
We take that 4 I’s come together, and they are treated as 1 letter,
∴ Total number of letters=11 – 4 + 1 = 8
⇒ Number of permutations = 8!/(4! 2!)
= (8 × 7 × 6 × 5 × 4!)/ (4! × 2) = 840
Therefore, the total number of permutations where four Is don’t come together
= 34650 – 840 = 33810UnattemptedGiven word – MISSISSIPPI
M – 1
I – 4
S – 4
P – 2
Number of permutations = 11!/(4! 4! 2!)
= (11 × 10 × 9 × 8 × 7 × 6 × 5 × 4!)/ (4! × 24 × 2) = 34650
We take that 4 I’s come together, and they are treated as 1 letter,
∴ Total number of letters=11 – 4 + 1 = 8
⇒ Number of permutations = 8!/(4! 2!)
= (8 × 7 × 6 × 5 × 4!)/ (4! × 2) = 840
Therefore, the total number of permutations where four Is don’t come together
= 34650 – 840 = 33810 - Question 63 of 80
63. Question
DIRECTION : READ THE FOLLOWING PASSAGE AND ANSWER THE QUESTIONS THAT FOLLOW. YOUR ANSWERS TO THESE QUESTIONS SHOULD BE BASED ON THE PASSAGE ONLY.
If you’re a fitness walker, there is no need for a commute to a health club. Your neighborhood can be your health club. You don’t need a lot of fancy equipment to get a good workout either. All you need is a well-designed pair of athletic shoes.
This paragraph best supports the statement that :
(A) fitness walking is a better form of exercise than weight lifting.
(B) a membership in a health club is a poor investment.
(C) walking outdoors provides a better workout than walking indoors.
(D) fitness walking is a convenient and valuable form of exercise.CorrectIncorrectThe author stresses the convenience of fitness walking, by stating that it does not require a commute to a health club. The paragraph also implies that fitness walking will result in a good workout. Choice (A) is incorrect because no comparison to weight lifting is made. Choice (B) may seem like a logical answer, but the paragraph only refers to people who are fitness walkers, so for others, a health club might be a good investment. Choice (C) is not in the passage.
UnattemptedThe author stresses the convenience of fitness walking, by stating that it does not require a commute to a health club. The paragraph also implies that fitness walking will result in a good workout. Choice (A) is incorrect because no comparison to weight lifting is made. Choice (B) may seem like a logical answer, but the paragraph only refers to people who are fitness walkers, so for others, a health club might be a good investment. Choice (C) is not in the passage.
- Question 64 of 80
64. Question
In a small village, there are 87 families, of which 52 families have at most 2 children. In a rural development programme, 20 families are to be chosen for assistance, of which at least 18 families must have at most 2 children. In how many ways can the choice be made?
(A) 52C18 × 35C2 + 52C19 × 35C1 + 52C20
(B) 52C18 × 35C2 + 52C29 × 35C1 + 52C20
(C) 52C18 × 35C2 + 52C19 × 35C1 + 52C26
(D) 52C15 × 35C2 + 52C19 × 35C1 + 52C20CorrectIncorrectGiven,
Total number of families = 87
Number of families with at most 2 children = 52
Remaining families = 87 – 52 = 35
Also, for the rural development programme, 20 families are to be chosen for assistance, of which at least 18 families must have at most 2 children.
Thus, the following are the number of possible choices:
52C18 × 35C2 (18 families having at most 2 children and 2 selected from other types of families)
52C19 × 35C1 (19 families having at most 2 children and 1 selected from other types of families)
52C20 (All selected 20 families having at most 2 children)
Hence, the total number of possible choices = 52C18 × 35C2 + 52C19 × 35C1 + 52C20UnattemptedGiven,
Total number of families = 87
Number of families with at most 2 children = 52
Remaining families = 87 – 52 = 35
Also, for the rural development programme, 20 families are to be chosen for assistance, of which at least 18 families must have at most 2 children.
Thus, the following are the number of possible choices:
52C18 × 35C2 (18 families having at most 2 children and 2 selected from other types of families)
52C19 × 35C1 (19 families having at most 2 children and 1 selected from other types of families)
52C20 (All selected 20 families having at most 2 children)
Hence, the total number of possible choices = 52C18 × 35C2 + 52C19 × 35C1 + 52C20 - Question 65 of 80
65. Question
Find the next term in the series.
10, 18, 28, 40, 54, ?
(A) 56
(B) 68
(C) 70
(D) 71CorrectIncorrectThe pattern is as follows:
1 x 5 + 5 = 10
2 x 6 + 6 = 18
3 x 7 + 7 = 28
4 x 8 + 8 = 40
5 x 9 + 9 = 54
6 x 10 + 10 = 70UnattemptedThe pattern is as follows:
1 x 5 + 5 = 10
2 x 6 + 6 = 18
3 x 7 + 7 = 28
4 x 8 + 8 = 40
5 x 9 + 9 = 54
6 x 10 + 10 = 70 - Question 66 of 80
66. Question
DIRECTION : READ THE FOLLOWING PASSAGE AND ANSWER THE QUESTION THAT FOLLOW. YOUR ANSWER TO THIS QUESTION SHOULD BE BASED ON THE PASSAGE ONLY.
Today we can hardly conceive of ourselves without an unconscious. Yet between 1700 and1900, this notion developed as a genuinely original thought. The “unconscious” burst the shell of conventional language, coined as it had been to embody the fleeting ideas and the shifting conceptions of several generations until, finally, it became fixed and defined in specialized terms within the realm of medical psychology and Freudian psychoanalysis.
The vocabulary concerning the soul and the mind increased enormously in the course of the nineteenth century. The enrichments of literary and intellectual language led to an altered understanding of the meanings that underlie time-honored expressions and traditional catchwords. At the same time, once coined, powerful new ideas attracted to themselves a whole host of seemingly unrelated issues, practices, and experiences, creating a peculiar network of preoccupations that as a group had not existed before. The drawn-out attempt to approach and define the unconscious brought together the spiritualist and the psychical researcher of borderline phenomena (such as apparitions, spectral illusions, haunted houses, mediums, trance, automatic writing); the psychiatrist or alienist probing the nature of mental disease, of abnormal ideation, hallucination, delirium, melancholia, mania; the surgeon performing operations with the aid of hypnotism; the magnetizer claiming to correct the disequilibrium in the universal flow of magnetic fluids but who soon came to be regarded as a clever manipulator of the imagination; the physiologist and the physician who puzzled oversleep, dreams, sleepwalking, anesthesia, the influence of the mind on the body in health and disease; the neurologist concerned with the functions of the brain and the physiological basis of mental life; the philosopher interested in the will, the emotions, consciousness, knowledge, imagination and the creative genius; and, last but not least, the psychologist.
Significantly, most if not all of these practices (for example, hypnotism in surgery or psychological magnetism) originated in the waning years of the eighteenth century and during the early decades of the nineteenth century, as did some of the disciplines (such as psychology and psychical research). The majority of topics too were either new or assumed hitherto unknown colors. Thus, before 1790, few if any spoke, in medical terms, of the affinity between creative genius and the hallucinations of the insane .
Striving vaguely and independently to give expression to a latent conception, various lines of thought can be brought together by some novel term. The new concept then serves as a kind of resting place or stocktaking in the development of ideas, giving satisfaction and a stimulus for further discussion or speculation. Thus, the massive introduction of the term unconscious by Hartmann in 1869 appeared to focalize many stray thoughts, affording a temporary feeling that a crucial step had been taken forward, a comprehensive knowledge gained, a knowledge that required only further elaboration, explication, and unfolding in order to bring in a bounty of higher understanding. Ultimately, Hartmann's attempt at defining the unconscious proved fruitless because he extended its reach into every realm of organic and inorganic, spiritual, intellectual, and instinctive existence, severely diluting the precision and compromising the impact of the concept.
Which one of the following sets of words is closest to mapping the main arguments of the passage?
(A) Language; Unconscious; Psychoanalysis.
(B) Unconscious; Latent conception; Dreams.
(C) Literary language; Unconscious; Insanity.
(D) Imagination; Magnetism; Psychiatry.CorrectIncorrectThe question asks us to pick the choice that is closest to mapping the main arguments of the passage. We have to ensure that the three ideas have significant presence in the passage, precisely in the order in which they have come in the passage. Here we can mark the answer by looking at the first word of each choice. The first paragraph and the first few lines of second paragraph talk about how the word unconscious burst the shell of conventional language. This makes choice A our best bet. Literary language, in choice C, is different from language, while the latter means language in general, the former means “highly stylized language”. Thus literary language is not the author's idea; it is how the word unconscious burst the shell of conventional language, not literary language.
Latent conception in option B, and magnetism in option D make those choices irrelevant.UnattemptedThe question asks us to pick the choice that is closest to mapping the main arguments of the passage. We have to ensure that the three ideas have significant presence in the passage, precisely in the order in which they have come in the passage. Here we can mark the answer by looking at the first word of each choice. The first paragraph and the first few lines of second paragraph talk about how the word unconscious burst the shell of conventional language. This makes choice A our best bet. Literary language, in choice C, is different from language, while the latter means language in general, the former means “highly stylized language”. Thus literary language is not the author's idea; it is how the word unconscious burst the shell of conventional language, not literary language.
Latent conception in option B, and magnetism in option D make those choices irrelevant. - Question 67 of 80
67. Question
Answer the question based on the figure given below in which Rectangle represents Males, Circle represents the urbans, Square represents the educated and Triangle represents the civil servants.
The number indicating the uneducated females who are urban civil servants is
(A) 11
(B) 5
(C) 9
(D) 7CorrectIncorrectUneducated female urban civil servants: 11
UnattemptedUneducated female urban civil servants: 11
- Question 68 of 80
68. Question
DIRECTION : READ THE FOLLOWING PASSAGE AND ANSWER THE QUESTIONS THAT FOLLOW. YOUR ANSWERS TO THESE QUESTIONS SHOULD BE BASED ON THE PASSAGE ONLY.
Once people wore garlic around their necks to ward off disease. Today, most Americans would scoff at the idea of wearing a necklace of garlic cloves to enhance their well-being. However, you might find a number of Americans willing to ingest capsules of pulverized garlic or other herbal supplements in the name of health. Complementary and alternative medicine, which includes a range of practices outside of conventional medicine such as herbs, homeopathy, massage therapy, yoga, and acupuncture, hold increasing appeal for Americans. In fact, according to one estimate, 42% of Americans have used alternative therapies. In all age groups, the use of unconventional healthcare practices has steadily increased in the last 30 years, and the trend is likely to continue, although people born before 1945 are the least likely to turn to these therapies.
Why have so many patients turned to alternative therapies? Many are frustrated by the time constraints of managed care and alienated by conventional medicine’s focus on technology. Others feel that a holistic approach to healthcare better reflects their beliefs and values. Others seek therapies that relieve symptoms associated with chronic disease; symptoms that mainstream medicine cannot treat. Some alternative therapies have even crossed the line into mainstream medicine, as scientific investigation has confirmed their safety and efficacy. For example, physicians may currently prescribe acupuncture for pain management or to control the nausea associated with chemotherapy. Additionally, many U.S. medical schools teach courses in alternative therapies, and many health insurance companies offer some alternative medicine benefits.
What is the main idea of this passage?
(A) Alternative medicine is now a big business in the United States with more Americans seeking it out than ever before.
(B) Today, it is not unusual for mainstream doctors to incorporate alternative therapies into their practice.
(C) Over the last few decades, alternative medicine has become more popular, accepted, and practiced in the United States.
(D) People are tired of conventional medicine’s focus on technology.CorrectIncorrectUnattempted - Question 69 of 80
69. Question
Answer the question based on the figure given below in which Rectangle represents Males, Circle represents the urbans, Square represents the educated and Triangle represents the civil servants.
The number indicating the educated urban males who are not civil servants is
(A) 7
(B) 8
(C) 9
(D) 10CorrectIncorrectEducated Urban Males: 8 and 9
As 8 represents the civil servants, 9 is the number that we are looking for.UnattemptedEducated Urban Males: 8 and 9
As 8 represents the civil servants, 9 is the number that we are looking for. - Question 70 of 80
70. Question
Find the next term in the series.
809, 881, 945,1125, 1135, ?
(A) 1105
(B) 1150
(C) 1173
(D) 1169CorrectIncorrectThe series adds the product of the digits to the previous number:
809 + 72 = 881
881 + 64 = 945
945 + 180 = 1125
1125 + 10 = 1135
1135 + 15 = 1150UnattemptedThe series adds the product of the digits to the previous number:
809 + 72 = 881
881 + 64 = 945
945 + 180 = 1125
1125 + 10 = 1135
1135 + 15 = 1150 - Question 71 of 80
71. Question
Out of 130 students appearing in an exam, 62 failed in Science, 52 failed in English, whereas 24 failed in both Science and English. The number of students who passed is
(A) 50
(B) 20
(C) 40
(D) 30CorrectIncorrectTotal students =130
Number of failed students = (62 + 52) – 24 = 114 – 24 = 90
(24 is subtracted to avoid repetition as it is included in both 62 and 52)
Number of students who passed = 130 – 90 = 40
UnattemptedTotal students =130
Number of failed students = (62 + 52) – 24 = 114 – 24 = 90
(24 is subtracted to avoid repetition as it is included in both 62 and 52)
Number of students who passed = 130 – 90 = 40
- Question 72 of 80
72. Question
A person travels in the north direction, then turns right, then again turns right and thereafter turns left. In which direction is the person travelling now?
(A) North
(B) South
(C) East
(D) WestCorrectIncorrectThe movement of the person is indicated in the figure given below (from A to B, B to C, C to D and D to E).
The final movement is in the direction indicated by the line segment DE, which is towards the East direction.
UnattemptedThe movement of the person is indicated in the figure given below (from A to B, B to C, C to D and D to E).
The final movement is in the direction indicated by the line segment DE, which is towards the East direction.
- Question 73 of 80
73. Question
A publisher printed 3000 copies of a book for sale, the cost of each book being Rs. 7.00. He distributed 500 copies to different institutions free of cost. He allowed a book free of cost for each 24 books purchased. If the price of each book is fixed at Rs. 14.50 determine the rate of profit or loss of the publisher.
(A) 66% loss
(B) 66% profit
(C) 60% profit
(D) 60% lossCorrectIncorrectTotal cost of the books = Rs. (3000 × 7) = Rs. 21000
500 books are given free of cost.
Selling price for 25 books = 24 × 14.50 = Rs. 348
So, Total selling price = Rs. 34800
Gain = Rs. (34800 – 21000) = Rs. 13800
So, Gain% = 13800/21000 x 100 ≈ 66%UnattemptedTotal cost of the books = Rs. (3000 × 7) = Rs. 21000
500 books are given free of cost.
Selling price for 25 books = 24 × 14.50 = Rs. 348
So, Total selling price = Rs. 34800
Gain = Rs. (34800 – 21000) = Rs. 13800
So, Gain% = 13800/21000 x 100 ≈ 66% - Question 74 of 80
74. Question
DIRECTION : READ THE FOLLOWING PASSAGE AND ANSWER THE QUESTION THAT FOLLOW. YOUR ANSWER TO THIS QUESTION SHOULD BE BASED ON THE PASSAGE ONLY.
It's easy to forget that most of the world's languages are still transmitted orally with no widely established written form. While speech communities are increasingly involved in projects to protect their languages – in print, on air and online – orality is fragile and contributes to linguistic vulnerability. But indigenous languages are about much more than unusual words and intriguing grammar: They function as vehicles for the transmission of cultural traditions, environmental understandings and knowledge about medicinal plants, all at risk when elders die and livelihoods are disrupted.
Both push and pull factors lead to the decline of languages. Through war, famine and natural disasters, whole communities can be destroyed, taking their language with them to the grave, such as the indigenous populations of Tasmania who were wiped out by colonists. More commonly, speakers live on but abandon their language in favor of another vernacular, a widespread process that linguists refer to as “language shift” from which few languages are immune. Such trading up and out of a speech form occurs for complex political, cultural and economic reasons – sometimes voluntary for economic and educational reasons, although often amplified by state coercion or neglect. Welsh, long stigmatized and disparaged by the British state, has rebounded with vigor.
Many speakers of endangered, poorly documented languages have embraced new digital media with excitement. Speakers of previously exclusively oral tongues are turning to the web as a virtual space for languages to live on. Internet technology offers powerful ways for oral traditions and cultural practices to survive, even thrive, among increasingly mobile communities. I have watched as videos of traditional wedding ceremonies and songs are recorded on smartphones in London by Nepali migrants, then uploaded to YouTube and watched an hour later by relatives in remote Himalayan villages . . .Globalization is regularly, and often uncritically, pilloried as a major threat to linguistic diversity. But in fact, globalization is as much process as it is ideology, certainly when it comes to language. The real forces behind cultural homogenization are unbending beliefs, exchanged through a globalized delivery system, reinforced by the historical monolingualism prevalent in much of the West.
Monolingualism – the condition of being able to speak only one language – is regularly accompanied by a deep-seated conviction in the value of that language over all others. Across the largest economies that make up the G8, being monolingual is still often the norm, with multilingualism appearing unusual and even somewhat exotic. The monolingual mindset stands in sharp contrast to the lived reality of most the world, which throughout its history has been more multilingual than unilingual. Monolingualism, then, not globalization, should be our primary concern.
Multilingualism can help us live in a more connected and more interdependent world. By widening access to technology, globalization can support indigenous and scholarly communities engaged in documenting and protecting our shared linguistic heritage. For the last 5,000 years, the rise and fall of languages was intimately tied to the plow, sword and book. In our digital age, the keyboard, screen and web will play a decisive role in shaping the future linguistic diversity of our species.
The author lists all of the following as reasons for the decline or disappearance of a language EXCEPT:
(A) governments promoting certain languages over others.
(B) a catastrophic event that entirely eliminates a people and their culture.
(C) people shifting away from their own language to study or work in another language.
(D) the focus on only a few languages as a result of widespread internet use.CorrectIncorrectThis is a very simple question. Options A, B and C have been discussed as reasons behind decline or disappearance of a language. The author talks positively about internet in preserving languages. Thus D is the best choice.
UnattemptedThis is a very simple question. Options A, B and C have been discussed as reasons behind decline or disappearance of a language. The author talks positively about internet in preserving languages. Thus D is the best choice.
- Question 75 of 80
75. Question
Bimalbabu sells two cars each of Rs. 99,000. He makes a profit of 10% on the first car, but incurs a loss of 10% on the second. What will be his percentage of profit or loss on the whole transaction ?
(A) 1% profit
(B) 1% loss
(C) 4% profit
(D) 4% lossCorrectIncorrectC.P. of car sold at 10% profit = (100/100 + gain%) x S.P.
= 100/110 x 99000 = Rs. 90000
C.P. of car sold at 10% loss = 100/90 x 99000 = Rs. 110000
Total C.P. = Rs. (90000 + 110000)
= Rs. 200000
Total S.P. = Rs. 2 × 99000 = Rs. 198000
So, Loss = Rs. 2000
So, Loss% = 2000/200000 x 100 = 1%UnattemptedC.P. of car sold at 10% profit = (100/100 + gain%) x S.P.
= 100/110 x 99000 = Rs. 90000
C.P. of car sold at 10% loss = 100/90 x 99000 = Rs. 110000
Total C.P. = Rs. (90000 + 110000)
= Rs. 200000
Total S.P. = Rs. 2 × 99000 = Rs. 198000
So, Loss = Rs. 2000
So, Loss% = 2000/200000 x 100 = 1% - Question 76 of 80
76. Question
If a * b = 2a – 3b + ab, then 3 * 5 + 5 * 3 is equal to :
(A) 22
() 24
(C) 26
(D) 28CorrectIncorrecta * b = 2a – 3b + ab
3 * 5 = 2 × 3 – 3 × 5 + 3 × 5 = 6
5 * 3 = 2 × 5 – 3 × 3 + 3 × 5
= 10 – 9 + 15 = 16
Therefore, 3 * 5 + 5 *3
= 6 + 16 = 22Unattempteda * b = 2a – 3b + ab
3 * 5 = 2 × 3 – 3 × 5 + 3 × 5 = 6
5 * 3 = 2 × 5 – 3 × 3 + 3 × 5
= 10 – 9 + 15 = 16
Therefore, 3 * 5 + 5 *3
= 6 + 16 = 22 - Question 77 of 80
77. Question
Find the next term in the series.
2750, 2624, 2474, 2298, 2094, ?
(A) 1848
(B) 1856
(C) 1836
(D) 1815CorrectIncorrectThe series is as follows:
2750 – 9 x 14 = 2624
2624 – 10 x 15 = 2474
2474 – 11 x 16 = 2298
2298 – 12 x 17 = 2094
2094 – 13 x 18 = 1856UnattemptedThe series is as follows:
2750 – 9 x 14 = 2624
2624 – 10 x 15 = 2474
2474 – 11 x 16 = 2298
2298 – 12 x 17 = 2094
2094 – 13 x 18 = 1856 - Question 78 of 80
78. Question
DIRECTION : READ THE FOLLOWING PASSAGE AND ANSWER THE QUESTIONS THAT FOLLOW. YOUR ANSWERS TO THESE QUESTIONS SHOULD BE BASED ON THE PASSAGE ONLY.
Evolutionary psychology takes as its starting point the uncontroversial assertion that the anatomical and physiological features of the human brain have arisen as a result of adaptations to the demands of the environment over the millennia. However, from this reasonable point of departure, these psychologists make unreasonable extrapolations. They claim that the behaviour of contemporary man, in almost all its aspects is a reflection of features of the brain that acquired their present characteristics during those earliest days of our species when early man struggled to survive and multiply. These suggestions have a ready audience and the idea that Stone age man is alive in our genome and dictating aspects of our genome and dictating aspects of our behaviour has gained ground in the popular imagination.
The tabloids repeatedly run articles about ‘discoveries’ relating to ‘genes’ for aggression, depression, repression, and anything for which we need a readymade excuse. Such insistence on a genetic basis for behaviour negates the cultural influences and the social realities that separate us from our ancestors. The difficulty with pseudo science of this nature is just this popular appeal. People are eager to accept what is printed as incontrovertible, assuming quite without foundation, that anything printed has bona find antecedents. We would do well to remember that the phrenologists of the 19th century held sway for a considerable time in the absence of any evidence that behavioural tendencies could be deuced from the shape of the skull. The phrenologists are no more, but their genes would seem to be thriving.
The author mentions phrenologists as
(1) pseudo scientists who are the logical antecedents of evolutionary psychologists.
(2) a group with inherent appeal to the followers of evolutionary psychologists.
(3) a warning against blind acceptance of ideas.
Which of the statements (s) given above is/ are correct?
(A) 1 only
(B) 2 and 3 only
(C) 3 only
(D) None of theseCorrectIncorrectThe words ‘we would do well to remember’ confirm that the author is warning us about something. He is reminding us that the phrenologists gained prominence without any evidence for their beliefs and so warns us against blind acceptance of ideas.
UnattemptedThe words ‘we would do well to remember’ confirm that the author is warning us about something. He is reminding us that the phrenologists gained prominence without any evidence for their beliefs and so warns us against blind acceptance of ideas.
- Question 79 of 80
79. Question
If p × q = p + q + p/q , the value of 8 × 2 is :
(A) 6
(B) 10
(C) 14
(D) 16CorrectIncorrectpxq = p+q+p/q
So, 8×2 = 8+2 + 8/2 ́
= 10 + 4 = 14Unattemptedpxq = p+q+p/q
So, 8×2 = 8+2 + 8/2 ́
= 10 + 4 = 14 - Question 80 of 80
80. Question
If x * y = 3x +2y,
Then 2 * 3 + 3 * 4 is equal to
(A) 18
(B) 29
(C) 32
(D) 38CorrectIncorrectx * y = 3x + 2y
2 * 3 + 3 * 4
= 3 × 2 + 2 × 3 + 3 × 3 + 2 × 4
= 6 + 6 + 9 + 8 = 29Unattemptedx * y = 3x + 2y
2 * 3 + 3 * 4
= 3 × 2 + 2 × 3 + 3 × 3 + 2 × 4
= 6 + 6 + 9 + 8 = 29